105
1. What total dose delivered with conventional fractionation would yield a 5% risk of blindness 5 years after irradiation of the entire retina? A. 25 Gy B. 35 Gy C. 45 Gy D. 55 Gy Correct answer is C. REFERENCES: Joiner and van der Kogel. Basic Clinical Radiobiology. 4th edition, 2009, Chapter 13. Hall and Giaccia. Radiobiology for the Radiologist. 6th edition. 2006. Chapter 19. 2. Which of the following types of thermoluminescent dosimeter is most commonly used for dose measurements in radiation therapy? A. Lithium fluoride B. Lithium sulfate C. Calcium fluoride D. Sodium iodide Correct answer is A. RATIONALE: Lithium fluoride is almost the exclusive thermoluminescent dosimeter in radiation therapy because its effective atomic number, Zeff, is very close to that of soft tissue (8.2 versus 7.4). 3. Which of the following is a regression method used for modeling survival times? A. Fisher's exact test B. Bayesian inference C. Analysis of variance (ANOVA) D. Cox proportional hazards Correct answer is D. RATIONALE: The Cox proportional hazards model is a regression method for modeling survival times. It is also called the proportional hazards model because it estimates the ratio of the risks. 4. According to the EBCTCG meta-analysis, what was the relative risk of death from heart disease for patients who received radiation therapy for breast cancer? A. 1.05 B. 1.15 C. 1.25 D. 1.35 Correct answer is C. RATIONALE: There is a 1.27 relative risk (RR) factor of death from heart disease for patients who received radiation therapy for breast cancer. REFERENCE: Effects of radiation therapy and of differences in the extent of surgery for early breast cancer on local recurrence and 15 year survival: an overview of the randomized trials. Early Breast Cancer Trialists' Collaborative Group. Lancet. 2005;366:2087-2106.

Radiation Oncology ACR 2010 in-Service Exam Rationales

Embed Size (px)

DESCRIPTION

Answers and rationale for 2010 ACR Radiation Oncology In-Service examination

Citation preview

Page 1: Radiation Oncology ACR 2010 in-Service Exam Rationales

1. What total dose delivered with conventional fractionation would yield a 5% risk of

blindness 5 years after irradiation of the entire retina? A. 25 Gy B. 35 Gy C. 45 Gy D. 55 Gy Correct answer is C. REFERENCES: Joiner and van der Kogel. Basic Clinical Radiobiology. 4th edition, 2009, Chapter 13. Hall and Giaccia. Radiobiology for the Radiologist. 6th edition. 2006. Chapter 19. 2. Which of the following types of thermoluminescent dosimeter is most commonly used for

dose measurements in radiation therapy? A. Lithium fluoride B. Lithium sulfate C. Calcium fluoride D. Sodium iodide Correct answer is A. RATIONALE: Lithium fluoride is almost the exclusive thermoluminescent dosimeter in radiation therapy because its effective atomic number, Zeff, is very close to that of soft tissue (8.2 versus 7.4). 3. Which of the following is a regression method used for modeling survival times? A. Fisher's exact test B. Bayesian inference C. Analysis of variance (ANOVA) D. Cox proportional hazards Correct answer is D. RATIONALE: The Cox proportional hazards model is a regression method for modeling survival times. It is also called the proportional hazards model because it estimates the ratio of the risks. 4. According to the EBCTCG meta-analysis, what was the relative risk of death from heart

disease for patients who received radiation therapy for breast cancer? A. 1.05 B. 1.15 C. 1.25 D. 1.35 Correct answer is C. RATIONALE: There is a 1.27 relative risk (RR) factor of death from heart disease for patients who received radiation therapy for breast cancer. REFERENCE: Effects of radiation therapy and of differences in the extent of surgery for early breast cancer on local recurrence and 15 year survival: an overview of the randomized trials. Early Breast Cancer Trialists' Collaborative Group. Lancet. 2005;366:2087-2106.

Page 2: Radiation Oncology ACR 2010 in-Service Exam Rationales

5. Which of the following drugs was shown in a phase III trial to improve survival in patients

with symptomatic locally advanced or metastatic pancreatic cancer? A. Gemcitabine B. Capecitabine C. Bevacizumab D. Cisplatin Correct answer is A. REFERENCE: Burris, et al. Pivotal trial on gemcitabine for pancreatic cancer. Journal of Clinical Oncology (JCO). 1997. 6. According to the GOG (Homesley) study, which of the following survival outcomes was

demonstrated at 2 years for patients who have vulvar cancer with inguinal lymph node metastases and received pelvic radiation in comparison to pelvic lymphadenectomy?

A. Comparable survival rates B. Better survival rate with radiation C. Worse survival rate with radiation D. Survival was not analyzed. Correct answer is B. RATIONALE: In this GOG study, patients with vulvar cancer and positive groin lymph nodes after radical vulvectomy and bilateral groin dissection were randomized to receive pelvic-inguinal irradiation or pelvic lymphadenectomy. Patients who received irradiation had a significantly improved survival compared to patients who had surgery alone (2-year survival = 68% vs. 54%, p = 0.03). REFERENCE: Homesley, et al. Obstetrics & Gynecology. 1986;68:733-40. 7. What is the most appropriate treatment strategy for a patient who underwent a

lobectomy and mediastinal lymph node dissection with final pathology revealing a stage T2N0 SCLC?

A. Close surveillance B. Thoracic radiation therapy only C. Thoracic radiation therapy, chemotherapy, and PCI D. Chemotherapy and PCI only Correct answer is D. RATIONALE: Stage I small cell lung cancer (SCLC) is diagnosed in less than 5% of patients with SCLC. Patients who undergo complete resection (preferably by a lobectomy with either mediastinal nodal dissection or sampling) should receive postoperative chemotherapy, followed by PCI. REFERENCE: NCCN Practice Guidelines in Oncology. 2010.

Page 3: Radiation Oncology ACR 2010 in-Service Exam Rationales

8. A 2-year-old girl has had a complete resection of the right kidney for Wilms tumor

involving the kidney, perihilar lymph nodes, and renal sinus vessels. No tumor spillage occurred during surgery, and chest radiograph is normal. Which of the following treatments would be most appropriate?

A. Vincristine and dactinomycin alone B. Cyclophosphamide and etoposide alone C. 10.8 Gy of flank irradiation with chemotherapy D. 21.0 Gy of flank irradiation without chemotherapy Correct answer is A. RATIONALE: According to the current national Wilms tumor study group staging system, this child has stage II disease and would be treated with vincristine and dactinomycin alone. Stage III disease requires 10.8 Gy of local irradiation depending on the extent of involvement and tumor spill. 9. Which of the following statements about the use of the linear-quadratic (LQ) isoeffect

formula in clinical practice is true? A. The LQ formula takes into account consequential late effects. B. The LQ formula takes into account incomplete repair between two closely spaced

dose fractions. C. The LQ formula should be used as a guideline only and not as a substitute for

clinical judgment and experience. D. Stereotactic radiation treatments of >10 Gy per fraction are modeled accurately

using the LQ formula. Correct answer is C. RATIONALE: The linear-quadratic (LQ) model and formula is most useful as a conceptual framework or guide for how to approach radiation treatment planning, and it should not be used as a substitute for clinical judgment and experience. For example, the model currently has no provision to take either consequential late effects or incomplete repair between fractions into account. Further, there is much controversy over whether the model can or should be applied in cases of extreme hypofractionation, as might be the case for stereotactic or intraoperative radiation therapy. REFERENCE: Willers H and Held KD. Hematology/Oncology Clinics of North America. 2006;20:1-24. 10. Which of the following types of cancer most commonly metastasizes to the eye in

women? A. Lung B. Breast C. Colon D. Thyroid Correct answer is B. RATIONALE: The most common choroidal metastasis arises from breast cancer in women and from lung cancer in men.

Page 4: Radiation Oncology ACR 2010 in-Service Exam Rationales

11. The number of electrons in a neutral atom equals the: A. mass number. B. atomic weight. C. atomic number. D. nucleon number. Correct answer is C. RATIONALE: In a neutral atom, the number of electrons equals the number of protons. The number of protons designates the atomic number for a given atom. 12. What is the expected 10-year overall survival rate for a 45-year-old man who had a

surgical resection with negative surgical margins for a grade 1 chondrosarcoma? A. 90% B. 80% C. 70% D. 50% Correct answer is B. RATIONALE: Grade 1 chondrosarcomas are characterized as having a very low metastatic potential. The average age of presentation by a patient is in the mid-40s, and the expected 10-year overall survival rate for patients is 83%. REFERENCE: Evans HL, Ayala AG, Romsdahl MM. Prognostic factors in chondrosarcoma of bone: a clinicopathologic analysis with emphasis on histologic grading. Cancer. 1977;40:818. 13. Approximately what percentage of patients with invasive bladder cancer will develop a

superficial local recurrence after bladder-preserving therapy consisting of TURBT, radiation therapy, and chemotherapy?

A. 5% B. 25% C. 50% D. 67% Correct answer is B. RATIONALE: After bladder preservation, superficial local failure will occur in approximately 17% to 30% of cases depending on the series. Understanding the appropriate management and prognosis after local failure is a critical part of care for patients who undergo this treatment. REFERENCE: Weiss, et al. International Journal of Radiation Oncology, Biology, Physics (IJROBP). 2008;70(5):1502–1506. 14. Which of the following primary sites of stage T1 squamous cell carcinoma has a <20%

risk of bilateral lymph node involvement? A. Base of tongue B. Pyriform sinus C. Glottic larynx D. Nasopharynx Correct answer is C. RATIONALE: There is a very high incidence of bilateral lymph node involvement in many sites in the head and neck.

Page 5: Radiation Oncology ACR 2010 in-Service Exam Rationales

15. According to the International Prognostic Index, what is the expected 5-year survival rate

for a high-risk patient with diffuse large B-cell lymphoma? A. 5% B. 25% C. 50% D. 75% Correct answer is B. RATIONALE: The 5-year survival rate ranges from 75% for low-risk patients to 25% for high-risk patients with diffuse large B-cell lymphoma. 16. Which of the following groups provides the largest body of epidemiological data on

radiation-induced bone cancer? A. Uranium miners B. Young women employed as radium dial painters C. Patients who received repeated fluoroscopies for pulmonary tuberculosis D. Polynesian islanders exposed to atomic fallout Correct answer is B. RATIONALE: Young women employed as watch dial painters during the 1910s and 1920s suffered a significant increase in fatal bone tumors and other health effects, mostly due to the accidental ingestion of the radium-containing paint that allowed the hands and numbers on the watch face to glow in the dark. This public health tragedy represented a classic study in epidemiology and is acknowledged as the first mass experience with injuries (and fatalities) caused by exposure to ionizing radiation. REFERENCE: Mettler and Upton. Medical Effects of Ionizing Radiation. 3rd edition. 2008. Chapter 5. 17. A patient is treated with an abutting 9-MeV electron and a half-beam-blocked 6-MV

photon beam matched on the skin. The beam axes are parallel. What is expected of the dose distribution at a depth of 3 cm?

A. Homogeneity within +/-3% B. A cold spot on either side of the junction C. A hot spot on the electron side of the junction D. A hot spot on the photon side of the junction Correct answer is D. RATIONALE: When photons and electrons abut, a hot spot develops on the side of the photon field as the electrons scatter out of the electron field. 18. What is the endpoint of a phase I clinical trial? A. Survival rates B. Tumor response rate to treatment C. Indication for treatment D. Treatment toxicity rates Correct answer is D. RATIONALE: Phase I trials provide information about the maximum-tolerated dose(s) of treatment. These trials provide little or no information about anti-tumor activity.

Page 6: Radiation Oncology ACR 2010 in-Service Exam Rationales

19. According to the NSABP-18 (preoperative versus postoperative chemotherapy) trial, the

9-year ipsilateral breast tumor recurrence rate in patients converted from mastectomy to breast-conserving therapy was:

A. 4%. B. 8%. C. 16%. D. 24%. Correct answer is C. RATIONALE: The recurrence rate in the mastectomy BCT neoadjuvant group was 15.9% vs. 9.9% in the initial breast-conserving therapy (BCT) group (p = .04). REFERENCE: Wolmark N, et al. Preoperative chemotherapy in patients with operable breast cancer: nine year results from NSABP B-18. Journal of National Cancer Institute. Monograph. 2001;30:96-102. 20. Which of the following characteristics is associated with the RTOG 97-04 (Regine) trial

evaluating treatment for patients with resected pancreatic adenocarcinoma? A. Only patients with pancreatic head tumors were included in the study. B. The use of concurrent radiation therapy with 5-FU was compared to concurrent

radiation therapy with gemcitabine. C. Compared to patients who received 5-FU, patients who received gemcitabine had a

significantly higher rate of grade 3 or higher toxicity. D. Compared to patients who received 5-FU, patients who received gemcitabine had a

significantly higher rate of overall survival. Correct answer is C. RATIONALE: Option A is incorrect, since tumors from all parts of the pancreas were included. Option B is incorrect, since gemcitabine was not administered concurrently. Option C is correct, since 58% of patients had a higher rate of toxicity. Option D is incorrect, since only 9% of patients had a higher rate of overall survival. The survival difference was not statistically significant between the two treatment arms. REFERENCE: Regine, et al. JAMA. 2008. 21. Which of the following lymph nodes is the most common initial site of metastases in

patients with vulvar carcinoma? A. Obturator B. Internal iliac C. External iliac D. Superficial inguinal Correct answer is D. RATIONALE: As the primary drainage site for vulvar carcinomas, the superficial inguinal lymph nodes are the most commonly involved lymph node site in these patients. The superficial inguinal lymph nodes subsequently drain into the deep femoral lymph nodes and ultimately to the external iliac lymph nodes. Involvement of these sites without involvement of the superficial inguinal nodes is extremely uncommon.

Page 7: Radiation Oncology ACR 2010 in-Service Exam Rationales

22. Which of the following characteristics is associated with patients who have SCLC? A. Approximately 10% of patients are nonsmokers. B. Approximately 10% of patients present with a superior sulcus tumor. C. Approximately 30% of patients present with limited disease. D. It accounts for 30% of all lung cancers. Correct answer is C. RATIONALE: Small cell lung cancer (SCLC) accounts for 15% of all lung cancers. Nearly all cases of SCLC are attributed to cigarette smoking. About one third of patients present with limited disease confined to the chest. Less than 2% of patients with SCLC present with a superior sulcus tumor. 23. Which of the following stages is most appropriate for a patient with bilateral Wilms

tumor? A. Stage IIIb B. Stage IV C. Stage IVb D. Stage V Correct answer is D. RATIONALE: Patients with bilateral Wilms tumors have stage V disease. Each kidney is considered separately as far as radiation therapy management; however, the therapeutic approach emphasizes a renal-function-sparing approach. 24. For a tissue with an α/β ratio of 10 Gy, which of the following statements is true? A. This tissue would be spared preferentially by fractionation compared to one with a

lower α/β ratio. B. The dose response curve for this tissue would bend more gradually than if the

α/β ratio had been 3 Gy. C. This α/β ratio is characteristic of late-responding normal tissues. D. Most tumors have an α/β ratio lower than 10 Gy. Correct answer is B. RATIONALE: A tissue dose response curve with a high α/β ratio would be less "bendy" than one with a low α/β ratio. Tissues characterized by a high α/β ratio are less responsive to changes in dose per fraction than tissues with low α/β ratios. Generally, although not without exception, most late-responding normal tissues are characterized by low α/β ratios, and most early-responding normal tissues and tumors are characterized by high α/β ratios. 25. 1 mCi is equal to: A. 1.0 Bq. B. 3.7 Bq. C. 107 Bq. D. 3.7 x 107 Bq. Correct answer is D. RATIONALE: The definition of a Curie (Ci), also known as disintegrations per second (dps), is 3.7 x1010 Bq. Therefore, 1 mCi is equal to 3.7 x 107 Bq.

Page 8: Radiation Oncology ACR 2010 in-Service Exam Rationales

26. According to a randomized, placebo-controlled (Kohno) trial, Japanese women who had

bone metastases from breast cancer and received monthly infusions of 4 mg of zoledronic acid (Zometa) for 1 year had what percentage rate of reduced skeletal-related events?

A. 20% B. 40% C. 50% D. 60% Correct answer is B. RATIONALE: This important trial indicated a substantial benefit for the use of Zometa in women with metastatic breast cancer to the bone. Patients randomized to Zometa were less likely to have skeletal events including pathologic fracture, spinal cord compression, and the need for palliative radiation therapy. REFERENCES: Kohno N, Aogi K, Minami H, et al. Zoledronic acid significantly reduces skeletal complications compared with placebo in Japanese women with bone metastases from breast cancer: a randomized, placebo-controlled trial. Journal of Clinical Oncology. 2005;23:3314. 27. Choroidal melanoma has the highest risk of metastasis to the: A. liver. B. lung. C. brain. D. bone. Correct answer is A. RATIONALE: The COMS study for high-risk disease confirmed that >90% of metastases from choroidal melanoma are found in the liver. Metastasis to the lung is 28%, to the bone is 18%, and to the brain is 5%. 28. Which of the following margins around the iliac vessels is recommended when a pelvic

CTV is designed, according to the RTOG GU radiation oncology specialist consensus statement on pelvic lymph node volumes for patients with high-risk prostate cancer?

A. 4 mm B. 7 mm C. 10 mm D. 15 mm Correct answer is B. RATIONALE: A 7-mm margin is recommended in the RTOG GU radiation oncology specialist consensus statement. REFERENCE: Lawton, et. al. RTOG GU radiation oncology specialists reach consensus on pelvic lymph node volumes for high-risk prostate cancer. International Journal of Radiation Oncology, Biology, Physics (Int J Radiat Oncol Biol Phys). June 1, 2009;74(2):383-7.

Page 9: Radiation Oncology ACR 2010 in-Service Exam Rationales

29. Which of the following sites of squamous cell carcinoma has the highest risk of

retropharyngeal lymph node involvement? A. Pyriform sinuses B. Paranasal sinuses C. Rosenmüller fossa D. Tonsillar fossa Correct answer is C. RATIONALE: Nasopharyngeal primary has the highest risk of retropharyngeal and level 5 lymph node metastasis. 30. Which of the following studies reported a statistically significant improvement in overall

survival for patients receiving chemotherapy plus involved-field radiation therapy versus chemotherapy alone for early-stage, aggressive non-Hodgkin lymphoma?

A. SWOG (Miller) B. ECOG (Horning) C. GELA (Bonnet) D. EORTC (Gilman) Correct answer is A. RATIONALE: Only the initial report from the SWOG study showed an overall survival advantage for the addition of involved-field radiation therapy (RT) to chemotherapy. The ECOG study demonstrated only a disease-free survival advantage. The GELA study included patients older than 60 years with good IPI, and it showed no benefit from the use of RT. REFERENCES: Miller, et al. SWOG study. New England Journal of Medicine (NEJM). 1998. Horning, et al. ECOG study. Journal of Clinical Oncology (JCO). 2004. Bonnet, et al. GELA study. Journal of Clinical Oncology (JCO). 2007. 31. A course of fractionated radiation therapy yields a tumor cell surviving fraction of n and a

tumor control probability of 0.49. If the tumor cell surviving fraction had been 0.5n instead of n, the tumor control probability would be:

A. 0.50. B. 0.70. C. 0.85. D. 0.98. Correct answer is B. RATIONALE: Based on Poisson statistics, the tumor control probability (TCP) can be determined using the equation TCP = e-n. Substituting 0.49 for the TCP and solving for “n” gives a tumor cell surviving fraction of 0.71. If the cell surviving fraction had been 0.36 (i.e., “0.5n”), the new TCP would correspond to e-0.36 = 0.70.

Page 10: Radiation Oncology ACR 2010 in-Service Exam Rationales

32. The NCRP recommendation for annual occupational dose limits accounts for possible: A. exposures in an occupational setting only. B. exposures in an occupational setting and from background radiation. C. exposures in an occupational setting and from personal medical procedures. D. exposures in an occupational setting, from background radiation, and from personal

medical procedures. Correct answer is A. RATIONALE: The annual occupational dose limits recommended by NCRP 116 does not include exposure received from background radiation or personal medical procedures. Radiation workers should not wear their radiation badges during personal medical procedures and should not take their badges home with them. 33. Which of the following defines the probability that the observed data occurred by

chance? A. P value B. Chi square C. Odds ratio D. T-test Correct answer is A. RATIONALE: The P value gives the probability that the null hypothesis is correct. Technically, it is the probability that the observed data or more extreme outcome would have occurred by chance. 34. Which of the following characteristics is associated with the ESPAC-1 trial evaluating

treatment for patients with pancreatic cancer? A. The trial was a two-arm randomized controlled trial. B. Patients with resected ampullary cancer and resected pancreatic cancer were

included in the trial. C. Overall survival was improved in patients receiving adjuvant chemoradiation. D. Overall survival was improved in patients receiving adjuvant chemotherapy. Correct answer is D. RATIONALE: Option A is incorrect, because the trial had a 2X2 factorial design, but was really 3 separate trials. Option B is incorrect because only pancreatic cancer was included. The EORTC trial included both. Option C is incorrect because survival was worse in the chemoradiation group. Option D is correct because adjuvant chemoradiation improved survival. REFERENCE: Neoptolemos, et al. New England Journal of Medicine (NEJM). 2004.

Page 11: Radiation Oncology ACR 2010 in-Service Exam Rationales

35. According to the NSABP B-18 and B-27 trials, what were the 8-year locoregional breast

cancer recurrence rates for patients who had yPS 0 (T0N0) disease after preoperative chemotherapy and mastectomy without radiation therapy?

Chest Wall Regional Lymph Nodes A. 1.5% 4.5% B. 3.5% 3.5% C. 5% 2.5% D. 7% 1.5% Correct answer is A. RATIONALE: NSABP B-18 & B-27 studies addressed operable breast cancers treated with preoperative or postoperative systemic therapy prior to mastectomy or lumpectomy. Radiation therapy was not permitted to any regional lymphatics. In multivariate analysis, predictors of locoregional recurrence (LRR) were: 1) Clinical tumor size and nodal status (before preoperative chemotherapy) and 2) pathologic breast or nodal response (after preoperative chemotherapy). By using these independent predictors, rates of LRR in different patient subsets can be defined without the knowledge of pathologic axillary nodal status before preoperative chemotherapy. Table 60.7 Eight-Year Cumulative incidence Rates of Local-Regional Recurrence after Preoperative Chemotherapy and Mastectomy in the NSABP B-18/B-27 Trials According to Pathologic Response in the Breast and Pathologic Axillary Nodal Status at Surgery Pathologic Breast Response/Pathologic Nodal Status Type of Surgery/Radiation Node-Negative/pCR Node-Negative/No pCR Node Positive (n = 68) (n = 270) (n = 447) 8-Year Cumulative Incidence of Recurrence (%) Mastectomy/No Radiation+ Chest Wall Regional Chest Wall Regional Chest Wall Regional 1.5 4.4 5.0 3.0 11.2 3.7 NSABP, National Surgical Adjuvant Breast and Bowel Project; pCR, pathologic complete response. +No chest wall or regional nodal radiation was allowed per protocol. From Mamounas E, et al. State of the Science Conference on Preoperative Chemotherapy, Bethesda, MD, 2007 REFERENCE: Harris J., et al. (eds). Local regional therapy consideration in patients receiving preoperative chemotherapy. Diseases of the Breast. 4th edition. Philadelphia: Lippincott Williams & Wilkins Publishers. 2009;739-741. 36. What percent of patients diagnosed with SCLC present with superior vena caval

obstruction? A. <5% B. 10% C. 20% D. 30% Correct answer is B. RATIONALE: Superior vena caval obstruction is present at diagnosis in 10% of patients with small cell lung cancer (SCLC). REFERENCE: Sculier JP, et al. Superior vena caval obstruction syndrome in small cell lung cancer. Cancer. 1986;57:847.

Page 12: Radiation Oncology ACR 2010 in-Service Exam Rationales

37. Which of the following characteristics is associated with patients who have Langerhans

cell histiocytosis (LCH)? A. Ten percent of patients have cutaneous involvement. B. The mortality rate is 25% in patients with unifocal bone LCH. C. Immunohistochemical stains show a positive reaction for S-100 protein and CD1a. D. Multiorgan disease is more common in children older than 5 years of age. Correct answer is C. RATIONALE: Fifty percent to eighty percent of patients manifest cutaneous involvement. Studies of neonates and children under the age of 4 years have shown that 51% to 71% of children with Langerhans cell histiocytosis (LCH) present with multiorgan disease—much higher than that in older children and adults. Mortality for patients with unifocal bone LCH is 0.9%. REFERENCE: Satter, et al. Pediatric Dermatology. 2008;25:291-295. 38. Proton radiation therapy has the greatest theoretical advantage over photon radiation

therapy for treatment of: A. glioblastoma. B. medulloblastoma. C. non-small cell lung cancer. D. adenocarcinoma of the prostate. Correct answer is B. RATIONALE: The use of protons for craniospinal irradiation as part of medulloblastoma treatment achieves superior target dose coverage and sparing of normal tissue structures due to the lack of an exit dose. This may be especially important for children in terms of reducing the likelihood of late complications. For the other tumors listed, the role and possible advantages of protons have yet to be firmly established. REFERENCE: Lee, et al. International Journal of Radiation Oncology, Biology, Physics (Int J Radiat Oncol Biol Phys). 2005;63:362-372. 39. Compared to the total PSA value, which of the following percent-free PSA values is most

likely to indicate that a patient has prostate cancer? A. 10% B. 25% C. 40% D. 55% Correct answer is A. RATIONALE: The lower the free PSA, the higher the likelihood that a patient has prostate cancer. A percent-free PSA of ≤15% is associated with a higher Gleason grade and a poorer prognosis.

Page 13: Radiation Oncology ACR 2010 in-Service Exam Rationales

40. According to the AAPM TG40 report, treatments should be suspended if the morning

calibration check differs from the expected value by at least: A. 2.0%. B. 3.0%. C. 5.0%. D. 10.0%. Correct answer is C. RATIONALE: AAPM TG40 report on Quality Assurance sets action levels for morning calibration review. If the output is off by more than 5.0% of the expected value, treatments should not continue until corrected. 41. A 70-year-old man with prostate cancer has a painful, solitary metastasis to the thoracic

spine that is not at risk for spinal cord compression or impending pathologic fracture. What is the probability that the man will experience pain relief from administration of 12.5 to 25 Gy of stereotactic body radiation therapy in a single fraction?

A. 85% B. 75% C. 65% D. 55% Correct answer is A. RATIONALE: Stereotactic body radiation therapy (SBRT) has received increasing attention recently for the treatment of painful bony metastatic disease. Using accurate tumor localization with imaging and patient immobilization with body fixation, hypofractionated doses can be delivered with increased safety. A prospective cohort of 393 patients who had bony metastatic disease and received SBRT doses between 12.5 to 25 Gy achieved pain palliation and local control in 86% and 90%, respectively. REFERENCE: Gerszten PC, Burton SA, Ozhasoglu C, Welch WC. Radiosurgery for spinal metastases: clinical experience in 500 cases from a single institution. Spine. 2007;32:193. 42. Which of the following treatments is most appropriately performed first for a 4-year-old

patient who has an International Classification Group D (Reese-Ellsworth Group 5) unilateral retinoblastoma?

A. Enucleation B. Exenteration C. Radiation therapy D. Chemotherapy Correct answer is A. RATIONALE: International Classification Group D and Reese-Ellsworth Group 5 include very large tumors with vitreous seeding. Typically, there is a low probability of vision salvage; therefore, enucleation is recommended.

Page 14: Radiation Oncology ACR 2010 in-Service Exam Rationales

43. Which of the following factors associated with squamous cell carcinoma of the oral

cavity predicts the highest risk for lymph node metastasis? A. Proximity to the midline B. Floor of mouth involvement C. Depth of invasion D. Histologic grade Correct answer is C. RATIONALE: The depth of invasion correlates with the overall risk of lymph node metastasis. Tumor sizes, tumor location, and depth of invasion as well as proximity to midline are the predictors of the risk for contralateral lymph node involvement. This is important in designing fields for treatment with nonsurgical approaches. The influence of tobacco is not known to have an effect on lymph node involvement. REFERENCE: Perez, Brady, et al. Oral Cavity Cancer. Perez and Brady’s: Principles and Practice of Radiation Oncology. 5th edition Chapter 41. pp 896-897. 44. An otherwise healthy 50-year-old patient is diagnosed with stage IAE, extranodal

marginal zone lymphoma of the stomach. Which of the following treatments would be most appropriate for this patient?

A. Total gastrectomy B. R-CHOP x 3 and involved-field radiation therapy to 40 Gy C. R-CHOP x 6 and involved-field radiation therapy to 30 Gy D. Radiation therapy alone to 30 Gy Correct answer is D. RATIONALE: Radiation therapy alone to 30 Gy is the standard of care and yields relapse-free rates of >90%. 45. The carbon ion RBE is highest when: A. many small radiation doses are used. B. one or more large radiation doses are used. C. the irradiated cells have intact apoptotic pathways. D. the overall treatment time is less than 4 weeks. Correct answer is A. RATIONALE: The RBE is defined as a ratio of doses for low- versus high-LET types of radiation that yield an equal level of effect in cells or tissues. The RBE is higher when multiple, small radiation doses are used (e.g., conventional fractionation) compared to one or a few large radiation doses (e.g., stereotactic or intraoperative radiation therapy). The presence or absence of apoptosis as a mode of cell death and repopulation effects (that would presumably be absent for overall treatment times less than 4 weeks) have little or no bearing on the RBE. REFERENCE: Joiner and van der Kogel. Basic Clinical Radiobiology. 4th edition. 2009. Chapter 6.

Page 15: Radiation Oncology ACR 2010 in-Service Exam Rationales

46. For radiation-safety purposes, which of the following models has been adopted by

regulators to describe the risk of harmful effects from radiation doses? A. Linear no threshold B. Linear with threshold C. Linear quadratic no threshold D. Linear quadratic with threshold Correct answer is A. RATIONALE: Although the assumption of a dose-risk response increasing linearly without a dose threshold overestimates the biologic effects at low dose levels, this conservative model has been adopted by regulators in the absence of more reliable data. 47. The probability that a test will produce a significant difference at a given significance

level is called the: A. risk ratio. B. statistical power. C. negative predictive value. D. omnibus test. Correct answer is B. RATIONALE: The probability that a test will produce a significant difference at a given significance level is called the statistical power of a test (1 minus the probability of a type II error). 48. A woman has ER/PR-negative, HER2/neu-positive metastatic breast cancer with tumor

progression after anthracycline, taxane, and trastuzumab (Herceptin) therapy. What is the next most appropriate therapy?

A. Lapatinib (Tykerb) alone B. Capecitabine (Xeloda) alone C. Lapatinib and capecitabine D. Lapatinib, capecitabine, and bevacizumab (Avastin) Correct answer is C. RATIONALE: Lapatinib and capecitabine are recognized as the preferred choice in this setting, according to the 2009 Practice Guidelines by the National Comprehensive Cancer Network (NCCN). Based on a randomized phase III trial, the use of lapatinib and capecitabine in these patients improved time to progression compared to the use of capecitabine alone. The combination of lapatinib and trastuzumab is not used with cytotoxic chemotherapy in this setting. REFERENCE: Cameron, et al. Breast Cancer Research and Treatment. 2008;112:533-43.

Page 16: Radiation Oncology ACR 2010 in-Service Exam Rationales

49. A patient with liver fluke infestation has a higher risk for developing: A. hepatoblastoma. B. hepatocellular carcinoma. C. fibrolamellar carcinoma. D. cholangiocarcinoma. Correct answer is D. RATIONALE: Liver fluke infestation has been associated with the development of cholangiocarcinoma. 50. The majority of patients who have hereditary breast-ovarian cancer syndrome and

develop ovarian carcinoma have a mutation in which of the following genes? A. BRCA1 B. BRCA2 C. HER2/neu D. PTEN Correct answer is A. RATIONALE: The majority of patients who have hereditary breast-ovarian cancer (HBOC) syndrome and develop ovarian carcinomas have deletions in the BRCA1 gene, with more than 100 mutations recognized to date. HBOC is less commonly associated with mutations in the BRCA2 gene. 51. According to a prospective randomized study for patients who had limited-stage SCLC

and achieved a complete response to therapy, which of the following results was associated with high-dose (36 Gy) versus standard-dose (25 Gy) PCI?

A. Increased 2-year disease-free survival rate B. Increased 2-year overall survival rate C. Decreased isolated CNS relapse rate D. Decreased treatment compliance rate Correct answer is C. RATIONALE: The corresponding 2-year incidence rates of total brain metastases in patients with small cell lung cancer (SCLC) are 29% (95% CI 24–35) in the standard radiation dose group and 23% (18–29) in the higher radiation dose group. Comparison of the cumulative incidence of total brain metastases curves by adjusted Gray’s test yields an HR of 0.76 (0.54–1.05, p=0.10), with a 2-year difference of 6% (22% vs 16%) favoring the higher radiation dose group. When brain metastasis is considered as an isolated first site of failure, the HR is 0.48 (0.29–0.81, p=0.005), and the 2-year difference is also 6% (12% vs 6%) favoring the higher radiation dose group. REFERENCE: Lancet Oncology. 2009;10:467–74.

Page 17: Radiation Oncology ACR 2010 in-Service Exam Rationales

52. Which of the following patients with Langerhans cell histiocytosis (LCH) has the highest

mortality rate? A. 1-year-old boy with multisystem involvement B. 6-year-old boy with multifocal bone disease C. 10-year-old girl with multisystem involvement D. 35-year-old man with isolated pulmonary LCH Correct answer is A. RATIONALE: Mortality rates for patients with Langerhans cell histiocytosis (LCH) are approximately as follows: An adult with isolated pulmonary LCH = 27%. A 1-year-old boy with multisystem involvement = 60% to 70%. A 10-year-old girl with multisystem involvement = 10% to 20%. A 6-year-old boy with multifocal bone disease = 5%. REFERENCE: Satter, et al. Pediatric Dermatology. 2008;25:291-295. 53. Approximately how many DNA double-strand breaks per cell are produced after acute

exposure to 1 Gy of x-rays? A. 1 to 4 B. 20 to 40 C. 200 to 400 D. 1000 to 4000 Correct answer is B. RATIONALE: On average, 20 to 40 DNA double-strand breaks (DSBs) are produced per cell exposed to 1 Gy of x-rays. REFERENCES: Joiner and van der Kogel. Basic Clinical Radiobiology. 4th edition. 2009, Chapter 2. Hall and Giaccia. Radiobiology for the Radiologist. 6th edition. 2006. Chapter 2. 54. How do KERMA and absorbed dose quantities vary beyond the depth of maximum

dose? A. Both dose and KERMA fall off equally. B. KERMA falls off faster than dose. C. Dose falls off faster than KERMA. D. Dose falls, while KERMA rises. Correct answer is A. RATIONALE: Once electronic equilibrium is obtained for a beam-penetrating media, KERMA (kinetic energy released in the media) is the same as dose (energy released per mass).

Page 18: Radiation Oncology ACR 2010 in-Service Exam Rationales

55. Which of the following pathologic factors is most important in predicting the risk for local

recurrence of breast cancer after a patient has had breast-conserving surgery and radiation therapy?

A. Tumor size B. Tumor histology C. Extensive DCIS D. Margin status Correct answer is D. RATIONALE: Margin status appears to be the single most important prognostic factor for the risk of local recurrence after breast-conserving surgery that includes irradiation. Definitions of margin status vary between investigators. Most define a positive margin as the presence of tumor in an inked surface. There is little evidence on how the exact tumor-free margin width affects the risk of local recurrence. The amount of disease at the margins may be important in determining the risk for local recurrence. In a Joint Center for Radiation Therapy finding, the risk of recurrence with focal margin involvement (i.e., invasive or DCIS cancer across all examined slides that could be encompassed by three or fewer low-power microscopic fields) was 14%. For patients with extensive margin involvement, the rate was 27%. In the same series, the respective rates of local recurrence were 7% and 18% for patients who received systemic therapy. Experience from William Beaumont Hospital reports the volume of disease near uninvolved margins affected the rate of local recurrence. With a median follow-up of 103 months, the 12-year actuarial rates of recurrence were 6%, 18%, and 12% for increasing tumor burden at 2.1 mm from the surgical margin. REFERENCES: Recht A. Breast cancer: stages T1 and T2. Gunderson LL and Tepper JE, eds. Clinical Radiation Oncology. 2nd edition. Chapter 60. Philadelphia: Churchill Livingstone Publishers. 2007;1478-1480. Park C, Mitsumori M, Nixon A, et al: Outcome at 8 years following breast-conserving surgery and radiation therapy for invasive breast cancer: influence of margin status and systemic therapy on local recurrences. Journal of Clinical Oncology. 2000;18:1668. Goldstein, NS, Kestin L, Vicini F: Factors associated with ipsilateral breast failure and distant metastases in patients with invasive breast carcinoma treated with breast conserving therapy. A clinicopathologic study of 607 neoplasms from 583 patients. American Journal of Clinical Pathology. 2003;120:500. 56. Which of the following treatments has improved overall survival in patients who have

advanced hepatocellular carcinoma with Child’s A cirrhosis? A. Gemcitabine B. Bevacizumab C. 5-Fluorouracil D. Sorafenib Correct answer is D. RATIONALE: BACKGROUND: No effective systemic therapy exists for patients with advanced hepatocellular carcinoma. A preliminary study suggested that sorafenib, an oral multikinase inhibitor of the vascular endothelial growth factor receptor, the platelet-derived growth factor receptor, and Raf may be effective in hepatocellular carcinoma. METHODS: In this multicenter, phase 3, double-blind, placebo-controlled trial, we randomly assigned 602 patients with advanced hepatocellular carcinoma who had not received previous systemic treatment to receive either sorafenib (at a dose of 400 mg twice daily) or a placebo. Primary outcomes were overall survival and the time to symptomatic progression. Secondary outcomes included the time to radiologic progression and safety. RESULTS: At the second planned interim analysis, 321 deaths had occurred, and the study was stopped. Median overall survival was 10.7 months in the sorafenib group and 7.9 months in the placebo group (hazard ratio in the sorafenib group, 0.69; 95% confidence interval, 0.55 to 0.87; P<0.001). There was no significant difference between the two groups in the median time to symptomatic progression

Page 19: Radiation Oncology ACR 2010 in-Service Exam Rationales

(4.1 months vs. 4.9 months, respectively, P=0.77). The median time to radiologic progression was 5.5 months in the sorafenib group and 2.8 months in the placebo group (P<0.001). Seven patients in the sorafenib group (2%) and two patients in the placebo group (1%) had a partial response; no patients had a complete response. Diarrhea, weight loss, hand-foot skin reaction, and hypophosphatemia were more frequent in the sorafenib group. CONCLUSIONS: In patients with advanced hepatocellular carcinoma, median survival and the time to radiologic progression were nearly 3 months longer for patients treated with sorafenib than for those given a placebo. REFERENCES: (ClinicalTrials.gov number, NCT00105443.) AD Barcelona Clinic Liver Cancer Group, Institut d'Investigacions Biomediques August Pi i Sunyer, Centro de Investigaciones en Red de Enfermedades Hepaticas y Digestivas Hospital Clinic Barcelona, Barcelona. [email protected]. Sorafenib in advanced hepatocellular carcinoma. AULlovet JM; Ricci S; Mazzaferro V; Hilgard P; Gane E; Blanc JF; de Oliveira AC; Santoro A; Raoul JL; Forner A; Schwartz M; Porta C; Zeuzem S; Bolondi L; Greten TF; Galle PR; Seitz JF; Borbath I; Haussinger D; Giannaris T; Shan M; Moscovici M; Voliotis D; Bruix J SON, New England Journal of Medicine (NEJM). July 24, 2008;359(4):378-90. 57. The primary purpose of a phase II clinical trial of an experimental drug is to: A. estimate the patient response rate to the drug with a 95% confidence interval. B. determine the largest dose of the drug that can be administered safely to a patient. C. determine if there is sufficient evidence of the drug’s safety and efficacy to justify

further testing. D. compare the experimental drug therapy to the historical control group’s current

standard of care. Correct answer is C. RATIONALE: Phase II trials are performed to determine whether there is sufficient preliminary evidence for the safety and efficacy of a treatment to justify further testing. The appropriate dose has been established in earlier trials, usually in a phase I trial. Response is not always the desired outcome and may be poorly defined in some disease settings. Direct comparison of two therapies is based on some evidence of efficacy for each one, and it is generally the province of phase III trials. 58. Which of the following MRS findings is most consistent with prostate cancer? A. Increased choline and citrate levels B. Increased choline levels, but decreased citrate levels C. Decreased choline levels, but increased citrate levels D. Decreased choline and citrate levels Correct answer is B. RATIONALE: Normal prostate glandular tissue produces large amounts of citrate. Cancer is associated with increased choline levels (secondary to cellular proliferation) and decreased citrate levels.

Page 20: Radiation Oncology ACR 2010 in-Service Exam Rationales

59. Which of the following radiation treatment approaches is most appropriate for early-

stage squamous cell carcinoma of the glottic larynx? A. 63 Gy in 35 fractions to the larynx only B. 63 Gy in 35 fractions to the larynx and level 2-4 lymph nodes C. 63 Gy in 28 fractions to the larynx only D. 63 Gy in 28 fractions to the larynx and level 2-4 lymph nodes Correct answer is C. RATIONALE: Stage T1N0 glottic larynx cancers can be treated very well with a hypofractionated course of treatment with randomized data demonstrating better local control. Complication rates are less than 1% for stage T1 tumors, as opposed to stage T2 tumors where the complication rates for severe laryngeal edema have been noted in 4% to 5% of patients. These are all important factors for evaluating patients with larynx cancer. Stage T2 tumors have a 3% to 7% risk of lymph node involvement as opposed to 1% for stage T1 tumors. 60. Which of the following histologic subtypes of Hodgkin lymphoma has the best prognosis

after treatment with involved-field radiation therapy alone? A. Lymphocyte-depleted B. Nodular lymphocyte-predominant C. Nodular sclerosis D. Mixed cellularity Correct answer is B. RATIONALE: Nodular lymphocyte-predominant Hodgkin lymphoma is distinct from the 4 types of classical Hodgkin lymphoma (nodular sclerosis, mixed cellularity, lymphocyte-depleted, and lymphocyte-rich Hodgkin lymphoma). This distinction is important since nodular lymphocyte-predominant Hodgkin lymphoma has an excellent prognosis when treated with involved-field radiation therapy alone. 61. In mammalian cells, which of the following groups of proteins are positive regulators of

apoptosis? A. Caspases, Bcl-2, Bcl-XL B. Caspases, Bcl-2, Bax C. Caspases, Bax, Apaf-1 D. Bcl-2, Apaf-1, Bcl-XL Correct answer is C. RATIONALE: The p53, Bax, and Apaf-1 proteins are all positive regulators of apoptosis. Further, the caspases facilitate the degradation of DNA and other subcellular components during the actual process of apoptosis. Only Bcl-2 is a negative regulator of apoptosis. REFERENCE: Tannock, Hill, Bristow, and Harrington. The Basic Science of Oncology. 4th edition. 2005. Chapter 10 (Section 10.2).

Page 21: Radiation Oncology ACR 2010 in-Service Exam Rationales

62. What happens when a low-energy photon interacts with material via coherent

scattering? A. The incident photon passes near the nucleus and spontaneously transforms into an

electron/positron pair. B. The incident photon transfers a fraction of its energy to an electron and is scattered

in another direction. C. The energy of the incident photon is radiated by the atom with only a slight change

in the direction of the emitted photon. D. The energy of the incident photon is completely transferred to an atomic electron. Correct answer is C. RATIONALE: During coherent scattering, a low-energy photon interacts with the entire atom and produces a slight excitation of electrons. The electrons then radiate energy by emitting a photon of the same energy but in a slightly different direction. Option A is a description of the photoelectric effect, option B describes Compton scattering, and option D describes pair production. 63. Which of the following three factors is most important in predicting the development of

distant metastases in patients who have a local recurrence after lumpectomy and radiation therapy for stage T1-T2 breast cancer?

A. Young age, LVI, and invasive local recurrence B. Young age, LVI, and HER2/neu-positive status C. Young age, short interval to recurrence, and ER-negative status D. Short interval to recurrence, invasive local recurrence, and positive lymph nodes at

initial diagnosis Correct answer is D. RATIONALE: After isolated local recurrence, the prognostic variables across series that are consistently significant for distant recurrence and death are short interval to recurrence, invasive local recurrence, and initial lymph node positive disease. Other variables of less importance are local recurrence of the breast only versus regional lymph nodes, age, hormone receptor status, and type of salvage treatment. REFERENCES: Wapnir, et al. Journal of Clinical Oncology. 2006;24:2028-37. Shen, et al. Cancer. 2005;104:479-90. Galper, et al. International Journal of Radiation Oncology, Biology, Physics. 2005;61:348-57. 64. Which of the following complications is LEAST likely to occur after a total gastrectomy? A. Slower intestinal passage due to loss of reservoir function B. Increased susceptibility to enteric infections due to loss of bacteriostatic function C. Eventual development of pernicious anemia due to loss of erythropoietic function

(intrinsic factor) D. Demineralization of bones due to impaired calcium absorption Correct answer is A. RATIONALE: Dumping syndrome with loss of reservoir function needs to be recognized after a patient has had a total gastrectomy, and the patient should be encouraged to consume frequent small meals. REFERENCE: http://www.netterimages.com/images/vpv/000/000/006/6684-0550x0475.jpg

Page 22: Radiation Oncology ACR 2010 in-Service Exam Rationales

65. Which of the following types of ovarian tumor is associated with virilization? A. Granulosa cell B. Sertoli-Leydig cell C. Dysgerminoma D. Choriocarcinoma Correct answer is B. RATIONALE: Sertoli-Leydig cell tumors are commonly associated with virilization. Young patients with granulosa cell tumors typically present with precocious puberty, whereas older patients present with amenorrhea. 66. Thoracic radiation therapy improves the absolute local control rate in patients with

limited-stage SCLC by: A. 10%. B. 15%. C. 20%. D. 25%. Correct answer is D. RATIONALE: Thoracic radiation therapy improves local control rates by 25% in patients with limited-stage small cell lung cancer (SCLC) and is associated with improved survival. REFERENCES: Pignon, et al. New England Journal of Medicine (NEJM). 1992;327:1618-1624. Payne, et al. Journal of Clinical Oncology. 1992;10:890-896. 67. Which of the following staging classifications would apply for a 4-year-old girl with a

7-cm, unresectable, vaginal rhabdomyosarcoma with a 1-cm inguinal lymph node and no distant metastasis?

A. Stage 1, group III B. Stage 2, group II C. Stage 3, group II D. Stage 3, group III Correct answer is A. RATIONALE: The rhabdomyosarcoma (RMS) staging classification incorporates the site, size, lymph node status, and metastatic status. RMS clinical grouping classification considers the degree of resection. All primary vaginal cancers with no metastasis are considered stage I; unresectable disease is considered to be group III. 68. What is the purpose of radiation-induced cell cycle checkpoints? A. To promote cell survival B. To promote cell death C. To contribute to genomic instability D. To initiate neoplastic transformation Correct answer is A. RATIONALE: The cell cycle checkpoint activation and arrest response induced by ionizing radiation exposure halts cell cycle progression to better coordinate with DNA repair. The function of cell cycle checkpoints is to prevent genomic instability and promote cell survival.

Page 23: Radiation Oncology ACR 2010 in-Service Exam Rationales

69. The photoelectric effect mass attenuation coefficient is proportional to: A. ZE. B. Z2E2. C. Z3E3. D. Z3/E3. Correct answer is D. RATIONALE: The photon electric effect is heavily dependent on atomic number (Z) and increases with Z. As the energy (E) increases, the probability of this effect decreases significantly. 70. According to the 20-year follow-up report for the NSABP B06 (Fisher) trial, the

cumulative incidence of breast cancer recurrence in the ipsilateral breast after breast-conserving surgery and radiation therapy is:

A. 8%. B. 11%. C. 14%. D. 17%. Correct answer is C. RATIONALE: The cumulative incidence of recurrent breast cancer in the ipsilateral breast was 14.2% in the women who underwent lumpectomy and breast irradiation compared with 39.2% in the women who underwent lumpectomy without irradiation. Study design: Tumor size was 4 cm or less; patients who were lymph node positive and negative were included; surgical margins at lumpectomy were negative (no tumor at ink), patients who had a lumpectomy with tumor at the surgical margin received a total mastectomy but were followed for subsequent events; patients with positive lymph nodes received melphalan and 5-FU; patients who received 50 Gy of radiation therapy to the breast without boost; patients who had positive nodes and did not receive radiation therapy to the regional lymphatics; no adjuvant tamoxifen was used. REFERENCE: Fisher B, Anderson S, Bryant J, et al. Twenty-year follow-up of a randomized trial comparing total mastectomy, lumpectomy, and lumpectomy plus irradiation for the treatment of invasive breast cancer. New England Journal of Medicine (NEJM). 2002;347:1233. 71. A biopsy of an enhancing liver mass demonstrates adenocarcinoma, with positive

staining for CK7 and CK20 but negative results for TTF-1. These findings are most consistent with:

A. metastatic colorectal cancer. B. metastatic non-small cell lung cancer. C. primary intrahepatic cholangiocarcinoma. D. hepatocellular carcinoma. Correct answer is C. RATIONALE:

CK7+ CK20+ Urothelial tumors Mucinous ovarian cancer Pancreatic or biliary cancer

CK7+ CK20- Non-small cell lung cancer Small cell lung cancer Breast cancer Endometrial cancer Nonmucinous ovarian cancer Mesothelioma Squamous cancer of cervix

CK7- CK20+ Colorectal cancer Merkel cell cancer

CK7- CK20- Hepatocellular cancer Renal cell cancer Prostate cancer Squamous cell lung cancer Head and neck cancer

Page 24: Radiation Oncology ACR 2010 in-Service Exam Rationales

72. Which of the following squamous cell carcinomas of the larynx is most appropriate for

laryngeal-conserving chemoradiation therapy? A. Glottic primary with decreased cord mobility B. Glottic primary with extension superiorly into the ventricle and false vocal cord C. Suprahyoid epiglottic primary with invasion through the thyroid cartilage D. Supraglottic primary with a fixed cord and preepiglottic space invasion Correct answer is D. RATIONALE: Examples that were described in the question included two early-stage larynx cancers (options A and B) that would be treated with radiation therapy alone without the use of systemic therapy. Having a significant involvement of the base of tongue were excluded from larynx preservation trials due to the poor outcomes on the VA larynx study, and the tumor with thyroid cartilage invasion did not go through the cartilage so it still would be a candidate for chemotherapy and radiation therapy, per RTOG 91-11. 73. Which of the following Ann Arbor stages is most appropriate for a patient who has

Hodgkin lymphoma with enlarged lymph nodes in the preauricular, cervical, and supraclavicular regions, but no other adenopathy on workup?

A. IA B. IAE C. IIA D. IIIA Correct answer is A. RATIONALE: The preauricular, cervical, and supraclavicular lymph node regions are considered as one region, based on the Ann Arbor staging system. 74. According to the International Germ Cell Cancer Collaborative Group Consensus

Classification System, a patient who has metastatic seminoma with elevated β-hCG and LDH levels would be classified into which of the following prognostic groups?

A. Very good B. Good C. Intermediate D. Poor Correct answer is C. RATIONALE: In the International Germ Cell Cancer Collaborative Group Consensus Classification System, the poor prognosis group includes only patients with non-seminomatous germ cell tumors. REFERENCE. International Germ Cell Consensus Classification: a prognostic factor-based staging system for metastatic germ cell cancers. International Germ Cell Cancer Collaborative Group. Journal of Clinical Oncology. Feb 1997; 15(2):594-603.

Page 25: Radiation Oncology ACR 2010 in-Service Exam Rationales

75. What is the approximate risk of a local recurrence if salvage breast-conserving surgery

is attempted again after initial conservation therapy? A. 5% B. 15% C. 30% D. 50% Correct answer is C. RATIONALE: The risk of second local recurrence has been reported to be 19% (Salvadori, et al. British Journal of Surgery (Br J Surg). 1999;86:84-7), 32% (Kurtz, et al. European Journal of Cancer (Eur J Cancer). 1991;27:240-4), and 38% (Voogd, et al. Cancer. 1999;85:437-46) in the three largest published series. 76. Which of the following DNA repair mechanisms has the highest activity during the S and

G2 phases of the cell cycle? A. Nucleotide excision repair B. Homologous recombination repair C. Nonhomologous end joining D. Single-strand annealing Correct answer is B. RATIONALE: Homologous recombination repair requires a homologous DNA template to repair a DNA double-strand break. This is usually supplied by a sister chromatid that is only present during the S or G2 phase of the cell cycle. REFERENCE: Hall and Giaccia. Radiobiology for the Radiologist. 6th edition. 2006. Chapter 5. 77. The tissue phantom ratio is NOT dependent on: A. field size. B. depth. C. energy. D. SSD. Correct answer is D. RATIONALE: The tissue phantom ratio (TPR) is defined as the ratio of the dose at a given point in a phantom relative to the dose at the same point at a fixed reference depth. This ratio is nominally used for monitor unit calculations in isocentric geometries. Although the source-to-surface distance (SSD) may vary depending on the contour of the patient's surface, the source-to-axis distance (SAD) remains constant. Thus, the TPR is independent of the SSD. 78. What is the leading cause of cancer death among women in developing countries

(American Cancer Society, 2007)? A. Breast B. Cervix C. Colon D. Lung Correct answer is B. RATIONALE: The most common cause of cancer death worldwide (in developing and developed countries) is breast cancer. Cervical cancer is the most common cause of death in developing countries and the third most common cause of death in developed countries (after lung cancer). REFERENCE: American Cancer Society (ACS), 2007 released statistics. http://www.cancer.org/downloads/STT/Global_Facts_and_Figures_2007_rev2.pdf

Page 26: Radiation Oncology ACR 2010 in-Service Exam Rationales

79. The extent of SCLC metastases may be indicated by elevated serum levels of: A. CA 19-9. B. CEA. C. LDH. D. AFP. Correct answer is C. RATIONALE: Lactate dehydrogenase (LDH) serum levels have been shown to predict the extent of small cell lung cancer (SCLC) metastases. 80. Which of the following study designs would be most appropriate for investigating the

effectiveness of a radiation dose of 70 Gy versus 60 Gy in providing local disease control for patients with prostate cancer?

A. Cox proportional hazard ratio B. Receiver operator characteristic curve C. One-tailed test D. Two-tailed test Correct answer is D. RATIONALE: In designing a study, it is tempting to think that a new intervention can be only beneficial and cause no harm. Such a study would focus on the positive impact only and is a one-tailed study design. The benefits include a smaller required sample size and increased study efficiency. A two-tailed design, though, analyzes whether the intervention increases the benefits (ie, local control) as well as increases the costs (ie, side effects). While it is more time-consuming, its results may be more well-respected. A Cox proportional hazard ratio is a method analyzing multiple collected pieces of data to see which are significant and which are not. A receiver operator characteristic curve is a plot of the true-positive rate against the false-positive rate of a studied condition. Neither the Cox proportional hazard ratio nor the receiver operator characteristic curve would be the optimal first selection for this study. REFERENCE. Moye L, et al. Circulation. 2002;105:3062-3065. 81. Which of the following statements about potentially lethal damage recovery (PLDR) is

true? A. It describes the increase in cell survival if a single radiation dose is split into two

fractions. B. It describes the increase in cell survival when cells are prevented from proliferating

after irradiation. C. Changing the postirradiation environmental conditions cannot modify PLD. D. It is the manifestation of the repair of mismatched bases in DNA. Correct answer is B. RATIONALE: Both sublethal and potentially lethal damage recovery are operationally defined terms, as both were first described and characterized more than 40 years ago, well before the DNA repair mechanisms of which they are a manifestation were elucidated. PLDR is defined as an increase in cell survival noted under conditions where the cells were prevented from proliferating for several hours after irradiation; such conditions may include, for example, overcrowding, presence of drugs that directly or indirectly inhibit cell cycle progression, or lack of sufficient nutrients and oxygen. Sublethal damage, on the other hand, is defined as that increase in cell survival noted when a single radiation dose is split into two fractions with a time interval inbetween. It is too simplistic to ascribe the biochemical repair of a particular type of DNA lesion to one or both of these phenomena, although it probably is true that both are manifestations of the rejoining of chromosome breaks, most of which arise from radiation-induced DNA double-strand breaks.

Page 27: Radiation Oncology ACR 2010 in-Service Exam Rationales

82. Compared to a placebo, tamoxifen given for 5 years produced which of the following

outcomes for women with a history of LCIS, according to the NSABP P-1 (Fisher) study? A. Reduced the hazard rate for subsequent invasive breast cancer by approximately

50% B. Decreased the risk of subsequent invasive breast cancer to 10%, compared to 30%

with the placebo C. Decreased the incidence of subsequent noninvasive but not invasive breast cancer D. Lowered the risk of subsequent invasive breast cancer only for tumors that were

ER-positive Correct answer is A. RATIONALE: The incidence of subsequent breast cancer for women with lobular carcinoma in situ (LCIS) was approximately 6% at 5 years with the placebo, and the relative risk for women randomized to tamoxifen was 0.54. Estrogen receptor (ER) status was not tested for LCIS to meet eligibility for the study. The relative risk was seen for invasive breast cancer as well as noninvasive breast cancer. REFERENCE: Fisher, et al. Journal of National Cancer Institute. 2005;97:1652-62. 83. Which of the following statements about the treatment of gastric cancer is true? A. A D2 lymph node dissection is defined by the removal of more than six lymph

nodes. B. In the INT-0116 randomized intergroup (Macdonald) trial, adjuvant therapy improved

locoregional control and overall survival. C. In the MAGIC (Cunningham) trial, perioperative chemotherapy improved the

pathologic complete response rate. D. In the randomized Dutch Gastric Cancer trial, D2 versus D1 lymphadenectomy

increased overall survival. Correct answer is B. RATIONALE: D2 lymphadenectomy is not defined by the number of lymph nodes removed (TNM stage N2 refers to 7-15 lymph nodes being positive, not to be confused with D2). Perioperative chemotherapy did not improve pathological complete response (CR) rates. D2 lymphadenectomy did not improve survival over D1 lymphadenectomy for patients with gastric cancer. 84. According to randomized clinical trial data, which of the following statements about

chemoradiation for esophageal cancer is true? A. Most of the patients in the RTOG 85-01 trial had esophageal squamous cell

carcinomas. B. The CALGB 9781 trial established the equivalence of chemoradiation to surgery for

resectable tumors. C. In the randomized German Esophageal Cancer Study Group (Stahl) trial, high-dose

chemoradiation had a lower local failure rate than standard-dose chemoradiation, followed by surgery.

D. The INT 0123 (Minsky) trial showed that increasing the radiation dose for definitive chemoradiation improved survival.

Correct answer is A. RATIONALE: Know current randomized clinical trial data [RTOG 85-01 trial, CALGB 9781 trial, German Esophageal Cancer Study Group (Stahl) trial, and INT 0123 (Minsky) trial data].

Page 28: Radiation Oncology ACR 2010 in-Service Exam Rationales

85. Which of the following histological types of carcinoma most commonly originates in the

fallopian tube? A. Squamous cell B. Adenosquamous C. Adenocarcinoma D. Papillary serous Correct answer is D. RATIONALE: More than 95% of fallopian tube cancers are papillary serous carcinomas. Other histologies including adenocarcinoma are very infrequently diagnosed. 86. According to the ICRU, which of the following volumes is most appropriate when CT

images are used during radiation treatment planning for a patient with prostate cancer? A. Gross tumor volume (GTV) B. Clinical target volume (CTV) C. Planning organ at risk volume (PRV) D. Planning target volume (PTV) Correct answer is B. RATIONALE: Gross disease within the prostate capsule cannot be discerned from normal prostate tissue based on CT images. Thus, the entire prostate is typically contoured as the CTV. Per ICRU Report #50 and #62, the CTV consists of the gross disease plus microscopic growth. 87. Which of the following characteristics of historical control patients is most accurate? A. They have known outcomes independent of the study. B. They receive the same treatment as in the experimental group. C. They are selected at random from healthy volunteers. D. Their standard-of-care treatment is no longer current. Correct answer is A. RATIONALE: A historical control is one whose outcomes are known before patients receive the experimental treatment that is the subject of the trial. The historical control patients are not only treated before the patients receiving the experimental protocol, but their outcomes are known. This knowledge may affect the design and/or conduct of the experimental part of the trial. The therapy received by a historical control is necessarily earlier in time but may not be considered outdated. Registries and special purpose databases are often used to identify historical control patients, but historical control data also may be collected from among recently treated patients at a hospital or clinic.

Page 29: Radiation Oncology ACR 2010 in-Service Exam Rationales

88. According to the ANITA trial, which of the following stages of NSCLC is most likely to

have the best response to postoperative chemotherapy and radiation therapy? A. pT4N1M0 B. pT3N0M0 C. pT2N1M0 D. pT1N2M0 Correct answer is D. RATIONALE: The ANITA randomized trial showed postoperative radiation therapy (RT) had a negative effect on pN1 non-small cell lung cancer (NSCLC) disease when postoperative chemotherapy was given. Five-year survival was 40% vs. 56% with or without postoperative RT. However, postoperative RT had a positive effect on pN2 disease even when postoperative chemotherapy was given. Five-year survival was 47.4% vs. 34% with or without postoperative RT. REFERENCE: International Journal of Radiation Oncology, Biology, Physics. 2008;72:690-701. 89. Which of the following is NOT a parameningeal site in the classification of

rhabdomyosarcoma? A. Nasopharynx B. Infratemporal fossa C. Nasal cavity D. Orbit Correct answer is D. RATIONALE: Parameningeal sites include the middle ear, nasal cavity, paranasal sinuses, nasopharynx, infratemporal fossa/pterygopalatine, and parapharyngeal areas. Orbital disease is considered a favorable site and is treated with lower-dose radiation therapy and chemotherapy. 90. According to the International Germ Cell Cancer Collaborative Group Consensus

Classification System, what is the expected 5-year overall survival rate for a patient with a nonseminomatous germ cell tumor classified as having a poor prognosis?

A. 90% B. 70% C. 50% D. 30% Correct answer is C. RATIONALE: Five-year overall survival for patients included in the test set was 48%. In the validation study, the 5-year overall survival for this patient population was 57%. The best answer above is 50%. REFERENCE: International Germ Cell Consensus Classification: a prognostic factor-based staging system for metastatic germ cell cancers. International Germ Cell Cancer Collaborative Group. Journal of Clinical Oncology. Feb 1997; 15(2):594-603.

Page 30: Radiation Oncology ACR 2010 in-Service Exam Rationales

91. The cytotoxicity and radiosensitization produced by gemcitabine are affected by the loss

of function of: A. ATM. B. MLH1. C. BRCA1. D. NBS1. Correct answer is B. RATIONALE: Gemcitabine is a pyrimidine analog that functions as an antitumor agent and potent radiation sensitizer. The drug's mechanism of action is to deplete cellular deoxynucleoside triphosphate pools and incorporate into DNA, both of which interfere with DNA replication and repair. A functional mismatch repair (MMR) system is required for gemcitabine to be active. Therefore, defects in MMR, in particular the loss of MLH1, abolish both the cytotoxic and radiosensitizing effects of gemcitabine. 92. What is the effective source position for electrons from the isocenter for a 100-cm SAD

treatment machine? A. 100 cm B. 99.8 cm C. 99.0 cm D. 90.0 cm Correct answer is D. RATIONALE: The source for most of the electrons is from the scattering foils located near the photon target in the treatment head. 93. Which of the following factors is used in the Gail model to predict an individual’s risk for

the development of breast cancer? A. Number of breast biopsies B. Use of exogenous hormones C. History of atypical lobular hyperplasia D. Age at first pregnancy Correct answer is A. RATIONALE: A combination of risk factors must be considered when a generalized risk profile is determined for the development of breast cancer. A model for an individual's annual and lifetime risks of breast cancer was developed by Gail, et al. This model was developed from data derived from white females and is based on the patient's present age, number of first-degree relatives with breast cancer, age at first birth, age at menarche, number of breast biopsies, and history of atypical ductal hyperplasia. The use of exogenous hormones is not considered in this model. REFERENCES: Haffty BG, Buchholz TA, Perez CA. Early-stage breast cancer. Halperin EC, Perez CA, Brady LW, eds. Perez and Brady’s: Principles and Practice of Radiation Oncology. 5th edition. Chapter 53. Philadelphia: Williams & Wilkins Publishers. 2008;1181. Gail, M, Brinton, IA, Bayar, DP, et al. Projecting individual probabilities of developing breast cancer for white females who are being examined annually. Journal of National Cancer Institute. 1989; 81:1879-1886.

Page 31: Radiation Oncology ACR 2010 in-Service Exam Rationales

94. Based on a secondary analysis of the EORTC 22921 Rectal Study, adjuvant

chemotherapy benefited patients who had rectal cancer and: A. tumors >3 cm on pathologic assessment. B. required an abdominal-perineal resection. C. stage ypT0-T2. D. stage ypN+. Correct answer is C. REFERENCE: Journal of Clinical Oncology. 25:4379-4386. 95. What percent of neuroblastomas occur in the low thoracic or abdominal paraspinal

ganglia? A. 5% B. 30% C. 45% D. 60% Correct answer is B. RATIONALE: According to a population-based study, 30% of neuroblastomas occur in the low thoracic or abdominal paraspinal ganglia. It is important for a radiation oncologist to know the common locations of neuroblastoma. REFERENCE: Bernstein, et al. A population-based study of neuroblastoma incidence, survival, and mortality in North America. Journal of Clinical Oncology. 1992;10(2):323-329. 96. Which of the following factors is associated with the highest risk of treatment failure for

patients with squamous cell carcinoma of the head and neck of unknown primary? A. Basaloid differentiation B. Lymphoepithelial histology C. Number of involved lymph nodes D. Extranodal extension Correct answer is D. RATIONALE: Extranodal extension is associated with the highest risk of treatment failure for patients with squamous cell carcinoma of the head and neck of unknown primary. 97. According to the EORTC 20884 (Aleman) trial, what involved-field radiation dose (in Gy)

was delivered to the lymph node sites in patients who had a complete response to MOPP-ABV chemotherapy for stage III or IV Hodgkin lymphoma?

A. 20 B. 24 C. 30 D. 36 Correct answer is B and C. RATIONALE: For patients who had a complete response to chemotherapy, a radiation dose of 24 Gy or 30 Gy was delivered to the lymph node sites. REFERENCE: Aleman, et al. New England Journal of Medicine (NEJM). 2003.

Page 32: Radiation Oncology ACR 2010 in-Service Exam Rationales

98. According to the Rubin and Casarett tissue classification system, which of the following

classes includes intestinal crypt cells? A. Fixed postmitotic (FPM) B. Reverting postmitotic (RPM) C. Differentiating intermitotic (DIM) D. Vegetative intermitotic (VIM) Correct answer is D. RATIONALE: The Rubin and Casarett tissue classification system was developed in the 1960s and represented an extension and refinement of the Bergonié and Tribondeau system first introduced some 60 years earlier. Using this system, tissues are categorized as being one of four main "classes," based on the particular tissue's proliferation kinetics and cellular differentiation status. As rapidly dividing, undifferentiated stem cells supplying the gut epithelium, crypt cells would be classified as VIM or Class I, and represent the most radiosensitive of the four types (VIM > DIM > RPM > FPM, ranked from most sensitive to most resistant). 99. Based on the table shown below, what is the PDD for a 10-MV beam with a field size of

6 x 12 cm2 at a depth of 15 cm and an SSD of 110 cm? (Please note: The depth of maximum dose for a 10-MV beam is 2.5 cm.)

PDD Table*

*10-MV beam at 15-cm depth and 100-cm SSD A. 55.1% B. 57.3% C. 58.2% D. 59.0% Correct answer is C. RATIONALE: The Mayneord F Factor is an approximate method to use to calculate how percent depth dose (PDD) changes with source-to-surface distance (SSD). This approximation is based on an inverse square approach and does not account for changes in scatter. However, before using this method, one must first calculate the equivalent square of the field, which can be calculated as 4A/P = 4 x 6 x 12/ (2x6 + 2x12) = 8. PDD2 =

%2.58)15110()15100(

)5.2100()5.2110(1.57

)()(

)()(

2

2

2

2

22

21

2max1

2max2

1 =++

×++

×=++

×++

×dSSDdSSD

dSSDdSSD

PDD

Field Size (cm2) 6 x 6 8 x 8 10 x 10 12 x 12

56.2% 57.1% 57.9% 58.5%

Page 33: Radiation Oncology ACR 2010 in-Service Exam Rationales

100. A clinical trial is conducted to investigate a drug’s effect on bone loss in elderly patients.

X-rays from middle-aged and elderly participants in the study will be distributed to one of two radiologists to be interpreted. Which of the following factors is most important to consider in ensuring the validity of the x-ray results?

A. Images from an equal number of middle-aged and elderly patients should be

interpreted by each of the two radiologists. B. The patients’ x-rays should be distributed randomly to each radiologist for

interpretation. C. Both radiologists should read the same number of images. D. Both radiologists should read the images concurrently. Correct answer is A. RATIONALE: Each image will be read only once, so any systematic differences between readings by the two radiologists will be difficult to separate from other effects on bone loss. By itself, this is acceptable if the differences can be shown to be small and/or controlled by use of a standard protocol for feature identification and data recording. Age is known to be related to bone loss, however, so it is important that the age distributions of the patients whose images are read by each radiologist are approximately the same. If not, any bias due to the reader will become confounded with age, making the study results difficult to interpret. If age is related to the time of day at which patients are available to participate in the study (e.g., middle-aged working participants during evenings and on weekends, elderly or retired participants during the day), the effect may be strong enough to rob the study of usefulness. Confounding can be controlled to some extent by design; that is, by making sure that the age profiles are similar for both radiologists. Randomizing the order in which images are read also does not reduce reader bias. Having each radiologist read half of the images ensures that the characteristics of reading by each radiologist will be equally represented, but it does not reduce the bias due to the reader. Timely data collection tends to improve data quality, and it is a good goal for the study, but it does not reduce bias or confounding. 101. Which of the following features is associated with Hodgkin lymphoma in children? A. Five percent of children present with stage IV disease. B. Forty to fifty percent of children have B symptoms. C. Eighty percent of children present with cervical lymphadenopathy. D. Lymphocyte-predominant lymphoma is the most common histologic subtype. Correct answer is C. RATIONALE: Fifteen to twenty percent of children present with stage IV Hodgkin lymphoma. Twenty-five to thirty percent of children have B symptoms. Nodular sclerosing histology is the most common subtype in all age groups. 102. According to the updated Silverstein trial data, what is the 12-year local recurrence rate

in patients with DCIS and a surgical margin of ≥1 cm after breast-conserving surgery? With Radiation Therapy Without Radiation Therapy A. 1.5% 5% B. 2.5% 14% C. 3.5% 30% D. 10% 30% Correct answer is B. RATIONALE: While recurrence rates in this favorable subset of women with margins ≥1 cm in the Silverstein ductal carcinoma in situ (DCIS) experience are generally lower than reported in prospective randomized trials, there remained a large difference in local recurrence with radiation therapy in their long-term update of results. REFERENCE: MacDonald, et al. American Journal of Surgery (Am J Surg). 2006;192:420-2.

Page 34: Radiation Oncology ACR 2010 in-Service Exam Rationales

103. Based on a retrospective analysis of the ANITA trial, which of the following stages of

NSCLC is most likely to have the best response to postoperative radiation therapy alone?

A. pT3N0M0 B. pT2N1M1 C. pT2N1M0 D. pT2N0M0 Correct answer is C. RATIONALE: The ANITA randomized trial showed postoperative radiation therapy (RT) had a negative effect on pN1 non-small cell lung cancer (NSCLC) disease when postoperative chemotherapy was given, but it may have a positive effect when RT alone is used. Five-year survival is 42.6% vs 31.4% with or without postoperative RT. LCSG 773 also showed better local control in patients who received postoperative RT for pN1 NSCLC disease; the local recurrence rate was 3% vs 41% with or without RT. REFERENCE: International Journal of Radiation Oncology, Biology, Physics (Int J Radiat Oncol Biol Phys). 2008;72:690-701. 104. Which of the following statements about small bowel tumors is FALSE? A. Most small bowel tumors occur in the duodenum or proximal jejunum. B. Most GI carcinoid tumors are located in the appendix, small bowel, or rectum. C. Patients with Crohn's disease and Peutz-Jeghers syndrome have a higher incidence

of small bowel malignancy. D. Patients with primary small bowel lymphomas are more likely to have Hodgkin

lymphoma versus non-Hodgkin lymphoma. Correct answer is D. REFERENCE: Schottenfeld D, Beebe-Dimmer JL, Vigneau FD. The epidemiology and pathogenesis of neoplasia in the small intestine. Annals of Epidemiology (Ann Epidemiol). Jan. 2009;19(1):58-69. 105. The uterine arteries run to the uterine fundus via what ligament? A. Broad B. Round C. Cardinal D. Uterosacral Correct answer is A. RATIONALE: Basic anatomy question, useful for understanding the different types of hysterectomies (in part defined by the degree of parametrial resection in relation to the artery) 106. What is the function of the tumor suppressor protein p21WAF1/CIP1? A. Targets ATM for destruction B. Promotes tumor angiogenesis C. Inhibits cyclin dependent kinases D. Activates TP53 transcription Correct answer is C. RATIONALE: The tumor suppressor protein p21WAF1/CIP1 binds to and inhibits the activities of CDK2 and CDK4. It is involved in cell cycle checkpoint control, especially in the progression from G1 into the S cell cycle phase. The expression of p21 is tightly controlled by the tumor suppressor protein p53, which serves as a positive transcription factor for the WAF1/CIP1 gene.

Page 35: Radiation Oncology ACR 2010 in-Service Exam Rationales

107. A man is diagnosed with urothelial cancer in the renal pelvis. What is the risk that a

synchronous or metachronous urothelial cancer will be found elsewhere in the renal pelvis, ureter, or bladder?

A. <5% B. 5% to 10% C. 11% to 20% D. >20% Correct answer is D. RATIONALE: Urothelial cancers commonly arise in the setting of "field cancerization," as a result of exposure of the entire urothelial tract to an associated predisposing risk factor (e.g., environmental carcinogen). Therefore, any patient with a newly diagnosed primary urothelial cancer is at significant risk for a synchronous or metachronous urothelial cancer. For upper urothelial tract cancers, it is estimated that 2% to 4% of patients will present with bilateral involvement. Meanwhile, approximately 25% of patients will have multifocal disease in the renal pelvis and ureter, while 50% of patients will have synchronous or metachronous disease involving the bladder. REFERENCE: Olgac, et al. American Journal of Surgical Pathology (Am J Surg Path). 2004;28:1545-1552. 108. What is the RBE for protons used in radiation therapy beams? A. 1.00 B. 1.05 C. 1.10 D. 1.20 Correct answer is C. RATIONALE: In vivo studies have shown protons to have an RBE of 1.10 compared with photon beams. This value has been accepted into clinical use. REFERENCES: Delaney and Kooy, eds. Proton and Charged Particle Radiotherapy. Gerweck L, Paganetti H. Radiobiology of Charged Particles. Philadelphia: Lippincott, Williams, and Wilkins Publishers. 2008. 109. Based on the German Rectal Cancer Study, which of the following outcomes in patients

with rectal cancer was significantly improved by preoperative chemoradiation versus postoperative chemoradiation?

A. Pelvic control B. Overall survival C. Distant metastatic rate D. Postoperative wound complications Correct answer is A. RATIONALE: Preoperative chemoradiotherapy, as compared with

postoperative chemoradiotherapy, improved local pelvic control in patients with rectal cancer. It also was associated with reduced toxicity, but it did not improve overall survival. REFERENCE: New England Journal of Medicine (NEJM). 2004;351:1731-40.

Page 36: Radiation Oncology ACR 2010 in-Service Exam Rationales

110. What is the most common primary malignancy of the fallopian tube in the United States? A. Papillary serous adenocarcinoma B. Transitional cell carcinoma C. Endometrioid carcinoma D. Clear cell carcinoma Correct answer is A. RATIONALE: Papillary serous adenocarcinoma is the most frequent primary neoplasm of the fallopian tube, and it was previously reported to represent 90% of the 300 new cases annually occurring in the United States. 111. Which of the following statements about squamous cell carcinoma of the hypopharynx is

true? A. Dysphagia is uncommon after primary radiation therapy is administered for a

posterior pharyngeal wall tumor. B. Pyriform sinus tumors are found early due to the development of hoarseness. C. Patients with level 4 lymph node involvement have a worse prognosis. D. The posterior pharyngeal wall is the most common site. Correct answer is C. RATIONALE: Squamous cell carcinomas of the hypopharynx are rare, although pyriform sinus cancers are much more common than posterior pharyngeal wall (PPW) cancers and have a late presentation with one of the symptoms being hoarseness related to vocal cord fixation. Outcomes are poor with lower neck involvement, which is important in the era of systemic chemotherapy, possibly requiring induction chemotherapy. Swallowing function can be poor when treating posterior pharyngeal wall (PPW) cancers, including treatment of all the constrictor muscles and the larynx. 112. A 65-year-old man with stage IIB diffuse large B-cell lymphoma has a 10-cm mediastinal

mass, an LDH serum level of 300 U/L, and an erythrocyte sedimentation rate (ESR) of 50 mm/h. His ECOG performance status is 2. Which of the following factors is associated with the worst prognosis?

A. Staging, B symptoms B. High ESR, B symptoms C. Bulky disease, high ESR D. Performance status, high LDH level Correct answer is D. RATIONALE: International Non-Hodgkin’s Lymphoma Prognostic Factors include an age of >60 years, stage III/IV disease, elevated lactate dehydrogenase [LDH] serum level, Eastern Cooperative Oncology Group [ECOG] performance status ≥2, more than one extranodal site of disease. REFERENCE: Project TIN-HsLPF. A predictive model for aggressive non-Hodgkin lymphoma. The International Non-Hodgkin’s Lymphoma Prognostic Factors Project. New England Journal of Medicine (NEJM). 1993;329:987-994.

Page 37: Radiation Oncology ACR 2010 in-Service Exam Rationales

113. Which type of chromosomal damage would be observed in the peripheral blood

lymphocytes of survivors 20 years after a radiation accident? A. Dicentrics B. Reciprocal translocations C. Terminal deletions D. Micronuclei Correct answer is B. RATIONALE: So-called "stable" aberrations, such as reciprocal translocations, would be evident. Unstable aberrations (dicentrics and terminal deletions) are typically fatal to the cell; thus, they would not persist for decades. Micronuclei are not a type of chromosomal aberration per se, but instead are small, membrane-bound, cytoplasmic structures visible following cell division, which contain acentric chromosomal fragments. 114. A beam spoiler typically is used during total skin electron therapy to: A. remove contaminated photons from the beam. B. decrease the dose rate at the patient's midplane within acceptable limits. C. decrease the dose to sensitive structures such as the lens of the eye. D. degrade the energy of the incident electron beam. Correct answer is D. RATIONALE: A beam spoiler is used to improve the beam uniformity and degrade the energy of the electron beam, thereby shifting the percent depth dose (PDD) to produce a shallower depth dose. The remaining answers are incorrect for the following reasons: A. One technique to reduce x-ray contamination from striking a patient, which is typically forward directed, is to treat the patient at an extended distance and angle the beam by +/-20o above the central axis (Stanford technique). A plastic spoiler would be insufficient to attenuate contaminate photons. B. Since the skin is the area of interest and electrons are used for treatment, dose and/or dose rate at the patient's midplane is not an issue for total skin electron therapy (TSET). C. To reduce the dose to sensitive structures, lead shields are used. In particular, to protect the lens of the eye, eye shields are used during treatment. 115. Which of the following statements about bladder preservation consisting of TURBT,

radiation therapy, and chemotherapy for muscle-invasive bladder cancer is FALSE? A. Over two thirds of surviving patients at 5 years will have a functioning bladder. B. Stage-by-stage cystectomy outcomes are superior to bladder-preserving therapy. C. Good response rates of 59% to 87% in patients have been achieved, according to

different RTOG trials. D. Reevaluation can be done 4 to 6 weeks after completion of therapy or after the initial

portion of therapy. Correct answer is B. RATIONALE: Understanding the basic concepts and rationale for bladder-sparing therapy in Europe and the United States are important for appropriate patient selection and evaluation during therapy. REFERENCE: Rodel, et al. Journal of Clinical Oncology (JCO). 2006;24:5536-5544. Sweeney, et al. ASCO Educational Book. 2009. pp 200-208.

Page 38: Radiation Oncology ACR 2010 in-Service Exam Rationales

116. Approximately what percent of U.S. patients diagnosed with DCIS receive breast-

conserving surgery and radiation therapy? A. 40% B. 60% C. 85% D. 90% Correct answer is A. RATIONALE: Approximately 40% of patients with DCIS are treated with breast-conserving surgery and radiation therapy, 30% are treated with breast-conserving surgery alone, and 30% are treated with mastectomy. REFERENCE: Smith, et al. International Journal of Radiation Oncology, Biology, Physics (Int J Radiat Oncol Biol Phys). 2006;65:1397-1403. Baxter, et al. Journal of National Cancer Institute (J Natl Cancer Inst). 2004;96:443-8. 117. Chemoradiation followed by surgery for esophageal cancer is LEAST likely to produce: A. higher local relapse rates than definitive chemoradiation. B. greater pathological complete response rates than chemotherapy alone. C. improved overall survival compared to surgery alone, based on the Walsh study. D. pathological complete response rates that are correlated with overall survival. Correct answer is A. RATIONALE: Adding surgery to chemoradiation reduces the local relapse rate but does not impact overall survival (admittedly, this was for squamous cell carcinomas). REFERENCE: Stahl M, Stuschke M, Lehmann N, Meyer HJ, Walz MK, Seeber S, Klump B, Budach W, Teichmann R, Schmitt M, Schmitt G, Franke C, Wilke H. Chemoradiation with and without surgery in patients with locally advanced squamous cell carcinoma of the esophagus. Journal of Clinical Oncology (J Clin Oncol). Apr 1, 2005;23(10):2310-7. 118. According to the GOG 123 (Keys) trial, the addition of chemotherapy to preoperative

radiation therapy improved the absolute 3-year overall survival rate for patients with stage IB2 cervical cancer by what percent?

A. 2% B. 5% C. 10% D. 20% Correct answer is C. RATIONALE: Five of the six studies comparing postoperative radiation therapy versus chemoradiation showed an overall survival benefit. In the Keys study, the survival improved from 74% to 83%. REFERENCE: Keys. New England Journal of Medicine (NEJM). 1999;340:1154-1161. 119. According to the AJCC TNM staging system for NSCLC, a tumor invading the

diaphragm is classified as stage: A. T1. B. T2. C. T3. D. T4. Correct answer is C. RATIONALE: Invasion of the diaphragm by the primary tumor for patients with non-small cell lung cancer (NSCLC) is classified as stage T3.

Page 39: Radiation Oncology ACR 2010 in-Service Exam Rationales

120. Which of the following treatments would be most appropriate for a 6-month-old child with

stage 1, low-risk neuroblastoma after complete tumor resection with no organ- or life-threatening symptoms?

A. Chemoradiation therapy B. Radiation therapy only C. Chemotherapy only D. Close follow-up Correct answer is D. RATIONALE: Patients with low-risk neuroblastoma have a cure rate of higher than 90%. After complete tumor resection for stage 1 disease, close observation or follow-up is the most appropriate management. REFERENCES: NCI PDQ Neuroblastoma. (http://www.cancer.gov/cancertopics/pdq/treatment/neuroblastoma/HealthProfessional). COG 9641 protocol. Matthay KK, Haas-Kogan D, Constine l. Neuroblastoma. Pediatric Radiation Oncology. 4th edition. Chapter 6. Philadelphia: Lippincott Williams & Wilkins Publishers. pp 179-222. Perez, et al. Biologic variables in the outcome of stages I and II neuroblastoma treated with surgery as primary therapy: a children's cancer group study. Journal of Clinical Oncology. Jan 2000;18(1):18-26. 121. Which of the following proteins is required for homologous recombination repair? A. Rad51 B. Artemis C. DNA-PKcs D. DNA ligase IV Correct answer is A. RATIONALE: Radiation-induced double-strand breaks (DSBs) are repaired in mammalian cells using one of two major pathways: nonhomologous end joining (NHEJ) and homologous recombination repair (HRR). NHEJ is an error-prone process involving interplay between numerous repair proteins, including: Ku70/80, DNA-PKcs, Artemis and DNA ligase IV. HRR on the other hand, is an error-free process involving proteins such as nucleases, helicases, Rad51/52/54, DNA polymerases and ligase III. NHEJ is the preferred DSB repair mechanism during the G1/G0 phases of the cell cycle, whereas HRR predominates in the S and G2 phases, because it relies on information on an undamaged sister chromatid that would only be available after DNA replication. 122. The quality of a superficial x-ray beam is usually determined by measuring the: A. peak voltage across the tube. B. half-value layer in a specified material. C. amount of filtration used in the beam. D. effective kVp of the beam. Correct answer is B. RATIONALE: The quality of treatment beams used for superficial therapy is determined by measuring the half-value layer (HVL) in a specified material (i.e., aluminum) and is specified in millimeters. REFERENCE: Khan. Physics of Radiation Therapy. Chapter 4.

Page 40: Radiation Oncology ACR 2010 in-Service Exam Rationales

123. According to the St. Gallen International Expert Consensus, which of the following

pathologic tumor-staging characteristics in a 60-year-old patient is categorized as intermediate-risk breast cancer?

A. T1N0, grade 1, LVI-negative B. T1N0, grade 1, LVI-positive C. T1N0, grade 2, HER2-positive D. T1N1, grade 2, HER2-positive Correct answer is B. RATIONALE:Options: A is low risk; B is intermediate risk, C and D are high risk. Systemic therapy is of benefit in both lymph node negative early-stage breast cancer and in locally advanced disease. For patients with all stages of breast cancer, systemic therapy has been shown to decrease the relative risk of relapse and mortality. However, there are subsets of patients with a very favorable prognosis and extremely low rate of relapse in whom the risk reduction is only a very small absolute benefit. The St. Gallen Consensus Conference proposed an algorithm for selection of systemic therapy in early-stage breast cancer based on risk and responsiveness to endocrine therapy. The St. Gallen risk categories are as follows: Table 53.13. Definition of Risk Categories for Patients with Operated Breast Cancer

Risk category

Low risk Node negative AND all of the following features: pT ≤2 cm, AND Grade 1. AND Absence of peritumoral vascular invasion, AND HER2/neu gene neither overexpressed nor amplified, AND Age ≥35 years

Intermediate risk Node negative AND at least one of the following features: pT >2 cm, OR Grade 2-3 OR Presence of peritumoral vascular invasion, OR HER2/neu gene overexpressed or amplified, OR Age <35 years Node positive (1–3 involved nodes) AND HER2/neu gene neither overexpressed nor amplified

High risk Node positive (1–3 involved nodes) AND HER2/neu gene overexpressed or amplified Node positive (4 or more involved nodes)

pT, pathological tumor size (i.e., size of the invasive component)

Page 41: Radiation Oncology ACR 2010 in-Service Exam Rationales

Table 53.14. Expert Consensus on the Therapy of Breast Cancer

Risk Category Endocrine Responsive

Endocrine Response Uncertain

Endocrine Nonresponsive

Low risk ET ET Not applicable

Nil Nil —

Intermediate risk

ET alone, or CT → ET CT

CT → ET (CT + ET) —

(CT + ET) — —

High risk CT → ET CT → ET CT

(CT + ET) (CT + ET) —

CT, chemotherapy; ET, endocrine therapy; Nil, no adjuvant systemic therapy

REFERENCES: Haffty BG, Buchholz TA, Perez CA. Early-stage breast cancer. Halperin EC, Perez CA, Brady LW, eds. Perez and Brady’s: Principles and Practice of Radiation Oncology. 5th edition. Chapter 53. Philadelphia: Lippincott Williams & Wilkins Publishers. 2008;1206-1207. Goldhirsh A, Glick JH, Gelber RD, et al. Meeting highlights: international expert consensus on the primary therapy of early breast cancer 2005. Annals of Oncology (Ann Oncol). 2005;16:1569-1583. 124. According to the MRC CR07 randomized trial, a short course of preoperative radiation

therapy (5 Gy x 5 fractions) versus selective postoperative chemoradiation significantly reduced the local recurrence rate for patients with rectal cancer in which of the following locations?

A. Lower portion of the rectum B. Higher portion of the rectum C. Mid-portion of the rectum D. All locations Correct answer is D. RATIONALE: Preliminary results indicate that a routine short course of preoperative radiation therapy significantly reduced local recurrence and improved disease-free survival at 3 years when compared with a highly selective postoperative approach. REFERENCE: Lancet. 2009;373:811–20.

Page 42: Radiation Oncology ACR 2010 in-Service Exam Rationales

125. FIGO stage IIIB endometrial cancer involves direct extension to the: A. serosa. B. adnexa. C. vagina. D. cervix. Correct answer is C. RATIONALE: FIGO stage IIIB endometrial cancer involves metastasis or direct extension to the vagina. Stage IIIA involves the serosa and or adnexa by direct extension or metastasis. A stage II cancer involves the cervix either by endocervical glandular involvement only (IIA) or by cervical stromal invasion (IIB). 126. Which of the following statements about squamous cell carcinoma of the pyriform sinus

is true? A. It has a better prognosis than primary tonsillar carcinoma. B. Otalgia is usually due to involvement of the superior laryngeal nerve. C. Level 3 lymph node involvement in the neck is uncommon. D. The overall risk of lymph node involvement is about 40%. Correct answer is B. RATIONALE: This item is important in regard to the management of pyriform sinus cancers. The risk of nodal metastasis is 70%. Primary oropharyngeal carcinoma tends to have a better prognosis. 127. A 50-year-old woman presents with a mass in the lesser curvature of the stomach, and

biopsy results reveal that she has MALT lymphoma. Which of the following treatment options would be most appropriate?

A. Gastrectomy with D2 nodal dissection for a patient without Helicobacter pylori

infection B. Local radiation therapy to 30 Gy for a patient with or without Helicobacter pylori

infection C. Combined multiagent chemotherapy and involved-field radiation therapy to reduce

the risk that MALT lymphoma will transform into a more aggressive type of lymphoma

D. First-line therapy for Helicobacter pylori infection and monitoring of the patient’s treatment response for several months before considering use of involved-field radiation therapy

Correct answer is D. RATIONALE: Patients with gastric MALT lymphoma and Helicobacter pylori infection should receive first-line therapy for Helicobacter pylori infection, and their treatment response should be evaluated for at least 6 months before making a decision on whether to include second-line therapy of involved-field radiation therapy. REFERENCES: Bayerdorffer E, Neubauer A, Rudolph B, et al. Regression of primary gastric lymphoma of mucosa-associated lymphoid tissue type after cure of Helicobacter pylori infection. MALT Lymphoma Study Group. Lancet. 1995;345:1591–1594. Roggero E, Zucca E, Pinotti G, et al. Eradication of Helicobacter pylori infection in primary low-grade gastric lymphoma of mucosa-associated lymphoid tissue. Annals of Internal Medicine (Ann Intern Med). 1995;122:767–769.

Page 43: Radiation Oncology ACR 2010 in-Service Exam Rationales

128. Which of the following types of radiation will yield the highest surviving fraction of

Chinese hamster ovary cells when delivered as two 1.5-Gy fractions separated by 3 hours as opposed to a single 3-Gy fraction?

A. 250-kVp x-rays B. 15-MeV neutrons C. 25-MeV alpha particles D. 100-MeV carbon ions Correct answer is A. RATIONALE: Little or no repair of sublethal damage is observed following high-LET irradiation (e.g., neutrons, alpha particles, carbon ions). Split-dose recovery is only seen for low-LET types of radiation, such as x-rays, γ-rays, and electrons. REFERENCE: Hall and Giaccia. Radiobiology for the Radiologist. 6th edition. 2006. Chapter 7. 129. What is an advantage of using an AP/PA technique versus opposed laterals at an

extended distance for total-body irradiation (TBI) with photons? A. Improved skin dose B. Improved dose homogeneity C. Lower midplane dose rate D. Lower lung dose Correct answer is B. RATIONALE: The advantage of using an AP/PA technique versus opposed laterals for total-body irradiation (TBI) is improved dose homogeneity due to the reduced variability in the body's thickness in the AP/PA orientation. A. To improve/enhance the skin dose in either a lateral or AP/PA setup, a beam spoiler should

be used. C. The dose rate for TBI is intentionally maintained within a 5 to 12 Gy/min to reduce the risk of

radiation pneumonitis, independent of the treatment technique used. D. The AP/PA technique will actually result in a larger lung dose compared to the lateral

technique. In a lateral setup, the patient's arms act as attenuators to reduce the lung dose.

130. What is the positive predictive value of PET imaging for 100 patients who have the

following results?

True positives: 36 True negatives: 18 False positives: 34 False negatives: 12

A. 51% B. 60% C. 67% D. 74% Correct answer is A. RATIONALE: The positive predictive value is a measure of number of true positives/(number of true positives+ number of false positives). In this case: 36/(36+34)=51%.

Page 44: Radiation Oncology ACR 2010 in-Service Exam Rationales

131. What is the median overall survival time for patients who have anaplastic oligodendrogliomas with 1p19q codeletion treated with sequential PCV chemotherapy with radiation therapy?

A. <1 year B. 3 years C. 5 years D. >7 years Correct answer is D. RATIONALE: In patients with 1p19q codeleted tumors, the median survival has not been reached with 7 to 8 years of follow-up. Patients who have tumors with intact 1p19q chromosomes had a median survival of only 1.8 and 2.8 years, based on the EORTC and RTOG trials. REFERENCES: van den Bent MJ, Carpentier AF, Brandes AA, et al. Adjuvant procarbazine, lomustine, and vincristine improve progression-free survival but not overall survival in newly diagnosed anaplastic oligodendrogliomas and oligo-astrocytomas: a randomized European Organization for Research and Treatment of Cancer phase III trial. Journal of Clinical Oncology. 2006;24:2715-2722. Cairncross G, Berkey B, Shaw E, et al. Phase III trial of chemotherapy plus radiotherapy compared with radiotherapy alone for pure and mixed anaplastic oligodendroglioma: Intergroup Radiation Therapy Oncology Group Trial 9402. Journal of Clinical Oncology. 2006;24:2707-2714. 132. What is the estimated probability that a patient with clinical stage T1c prostate cancer

with a Gleason score of 3+3 and a PSA level of 6 ng/mL has extracapsular extension? A. <10% B. 20% C. 30% D. >50% Correct answer is A. RATIONALE: Approximately 30% of patients with early-stage prostate cancer have extracapsular extension (ECE) of only a few millimeters. The radial distance of ECE is an important measure that influences treatment strategies for patients with localized prostate carcinoma, especially for the use of brachytherapy. In this particular case, the risk of positive nodes is ~11% [using the Roach formula of 2/3 PSA + (Gleason score−6) x 10]. That is, 2/3(6) + (6−6 = 0) x 10. 2/18 + 0 = 0.11 (or 11%). The probability of +ECE in this specific case is ~30% x ~11% = ~3% to 4%. REFERENCES: Roach, M. Equations for predicting the pathologic stage of men with localized prostate cancer using the preoperative prostate specific antigen: Journal of Urology. 1993;150:1924-1924. Halperin EC, Perez CA, Brady LW, eds. Low-Risk Prostate Cancer. Perez and Brady’s: Principles and Practice of Radiation Oncology. 5th edition. Chapter 62. p 1446. 133. Compared to sequential chemoradiation therapy, concurrent chemoradiation therapy

used for patients with stage III NSCLC is more likely to: A. decrease local recurrence. B. decrease overall survival rates. C. increase the risk of distant metastasis. D. increase overall survival rates. Correct answer is D. RATIONALE: Concurrent chemoradiotherapy has been shown to improve survival in patients with stage III non-small cell lung cancer (NSCLC), but the toxicity associated with the treatment is significant. Therefore, it should be considered only for a patient with a good-performance status. Even with concurrent chemoradiotherapy, local control is still not optimal, and dose escalation is being explored.

Page 45: Radiation Oncology ACR 2010 in-Service Exam Rationales

134. What is the dose rate to point A for an optimally placed, low-dose-rate radiation implant

to treat cervical cancer? A. 80 to 100 cGy/hr B. 45 to 55 cGy/hr C. 15 to 25 cGy/hr D. <10 cGy/hr Correct answer is B. RATIONALE: This is a basic low dose rate (LDR) radiation implant question, which is necessary for understanding the prescription dose for a cervical implant. REFERENCE: Nag S, et al. The American Brachytherapy Society recommendations for low-dose-rate brachytherapy for carcinoma of the cervix. International Journal of Radiation Oncology, Biology, Physics (Int J Radiat Oncol Biol Phys). Jan 1, 2002;52(1):33-48. 135. The percent labeled mitosis technique is used to determine the: A. potential doubling time (Tpot) of a tumor. B. likelihood of a normal tissue undergoing compensatory proliferation. C. latency period before radiation injury manifests in a tissue. D. cell cycle phase durations of proliferating cells in vivo. Correct answer is D. RATIONALE: The percent labeled mitosis technique is used to determine the individual cell cycle phase durations and overall cell cycle time of a proliferating cell population, either in vitro or in vivo. REFERENCE: Hall and Giaccia. Radiobiology for the Radiologist. 6th edition. Chapter 21. 136. The label "6 MV" used for linear accelerators during radiation therapy means that: A. the average beam energy is 6 MV. B. photons have an initial energy of 6 MV. C. 6 MV is applied between the filament and target. D. the kinetic energy of the accelerated electrons is approximately 6 MeV. Correct answer is D. RATIONALE: Linear accelerators beam energy labels are specified by the energy that electrons acquire in the waveguide. When the electrons strike the target, the resulting x-rays that are produced will have a range of energies upwards of approximately 6 MeV. It is a convention to drop off the "e" and designate the spectrum of energies by the term "6 MV." This is discussed in Chapter 4 of Khan's Physics of Radiation Therapy and Karzmark's Linear Accelerator Primer. 137. According to the Danish 82b (Overgaard) and British Columbia (Ragaz) trials, the

addition of postmastectomy radiation therapy to the chest wall and regional lymph nodes improved overall survival in premenopausal women with one to three positive lymph nodes by what percent?

A. 3% B. 5% C. 10% D. 13% Correct answer is C. RATIONALE: Both studies found that the overall survival was improved by 10% for patients receiving postmastectomy radiation therapy. Note: There were study design differences in the choice of systemic therapies, radiation doses/fractionation as well as

Page 46: Radiation Oncology ACR 2010 in-Service Exam Rationales

differences in systemic therapy and radiation sequencing schedules. None of these factors appeared to affect the improved overall survival finding reported in both studies. REFERENCES: Overgaard M, Hansen PS, Overgaard J, et al. Postoperative radiotherapy in high risk premenopausal women with breast cancer who receive adjuvant chemotherapy. New England Journal of Medicine (NEJM). 1997;337:949-955. Ragaz J, Jackson SM, Le N, et al. Adjuvant radiotherapy and chemotherapy in node-positive premenopausal women with breast cancer. New England Journal of Medicine (NEJM). 1997;337:956-962. Ragaz J, Olivotto IA, Spinelli JJ, et al. Locoregional radiation therapy in patients with patients with high-risk breast cancer receiving adjuvant chemotherapy: 20-year results of the British Columbia randomized trial. Journal of the National Cancer Institute (J Natl. Cancer Inst). 1997;97:116-126. 138. Which of the following treatments (listed in order of administration) would be most

appropriate after incomplete resection of a high-risk neuroblastoma? A. Local radiation therapy, surgery, followed by chemotherapy B. Chemotherapy, local radiation therapy, followed by cis-retinoic acid C. Chemotherapy, surgery, myeloablative therapy, followed by local radiation therapy D. Chemotherapy, surgery, myeloablative therapy, local radiation therapy, followed by

cis-retinoic acid Correct answer is D. RATIONALE: Myeloablative therapy has been shown to be superior to maintenance chemotherapy in the treatment of high-risk neuroblastoma, according to two randomized trials. In one of those trials, the addition of cis-retinoic acid has been shown to improve treatment outcome. REFERENCES: Matthay KK, Haas-Kogan D, Constine l. Neuroblastoma. Pediatric Radiation Oncology. 4 edition. Chapter 6. Philadelphia: Lippincott Williams & Wilkins Publishers. pp 179-222. Berthold F, Boos J, Burdach S, et al. Myeloablative megatherapy with autologous stem-cell rescue versus oral maintenance chemotherapy as consolidation treatment in patients with high-risk neuroblastoma: a randomized controlled trial. Lancet Oncology. 2005;6(9):649-58. Matthay KK, Villablanca JG, Seeger RC, et al. Treatment of high-risk neuroblastoma with intensive chemotherapy, radiotherapy, autologous bone marrow transplantation, and 13-cis-retinoic acid. Children's Cancer Group. New England Journal of Medicine (NEJM). 1999;341(16):1165-73. NCI PDQ Neuroblastoma. (http://www.cancer.gov/cancertopics/pdq/treatment/neuroblastoma/HealthProfessional) 139. Based on the CALGB 8984 phase II study, what was the local recurrence rate at

10 years for patients with stage T2 rectal tumors after full-thickness wide local excision with or without chemoradiation?

A. <10% with chemoradiation B. <10% without chemoradiation C. Approximately 20% with chemoradiation D. Approximately 20% without chemoradiation Correct answer is C. RATIONALE: In this study, all patients with stage T2 rectal cancer disease received 54 Gy of irradiation with 5-FU postoperatively. In spite of this treatment regimen, 10-year results showed a local recurrence rate of 18%. For this reason, fewer centers offer wide local excision (WLE) for stage T2 rectal tumors. REFERENCE: Dis Colon Rectum. Aug 2008;51(8):1185-91; discussion 1191-4.

Page 47: Radiation Oncology ACR 2010 in-Service Exam Rationales

140. What is the FIGO stage for an endometrial cancer with pelvic lymph node involvement? A. IIB B. IIIC C. IVA D. IVB Correct answer is B. RATIONALE: FIGO stage IIIC represents metastases to the pelvic and or para-aortic lymph nodes. FIGO stage IIB includes cervical stromal invasion. FIGO stage IVA involves invasion of the bladder and or bowel mucosa. FIGO stage IVB involves distant metastases. Pelvic lymph nodes in the TNM classification are represented by an N1 designation. 141. Which of the following histologies of stage T1N0 skin cancer of the scalp has the worst

prognosis? A. Angiosarcoma B. Squamous cell carcinoma C. Merkel cell carcinoma D. Melanoma Correct answer is A. RATIONALE: Angiosarcomas of the scalp have an extremely poor outcome due to field cancerization affecting the whole scalp. 142. A 70-year-old woman has grade 1 follicular lymphoma involving the pelvic lymph nodes.

Which of the following management options would be most appropriate for this patient? A. Rituximab for 6 months B. Radiation therapy to 40 Gy C. Either observation or radiation therapy of 20 to 30 Gy D. Combined multiagent chemotherapy and involved-field radiation therapy Correct answer is C. RATIONALE: Patients with low-grade follicular lymphoma have several options for disease management: observation, radiation therapy, and chemotherapy provide an equal chance of survival for this patient. REFERENCE: Horning ST. Natural history of and therapy for the indolent non-Hodgkin lymphomas. Seminars in Oncology (Semin Oncol).

1993;20:75-80. 143. Most radiation-induced HPRT gene mutations in human cell lines involve: A. transitions. B. transversions. C. frame shifts. D. large deletions. Correct answer is D. RATIONALE: For mammalian cells grown in culture, the HPRT mutagenesis assay is used to detect mutations in the HPRT gene, which codes for the enzyme hypoxanthine-guanine phosphoribosyl transferase. This enzyme is involved in the purine nucleotide salvage pathway, and as such, its loss is not a lethal event. Transitions, transversions, and frame shifts are all small, point mutations; however, most ionizing radiation-induced mutations are large, often involving the deletion of most or all of the gene.

Page 48: Radiation Oncology ACR 2010 in-Service Exam Rationales

144. A radiation survey is taken 1 m from a patient who received a brachytherapy implant.

How many half-value layers (HVLs) of lead would be required to reduce the exposure rate at 1 m from 6.7 mR/hr to 2 mR/hr?

A. 0.7 cm B. 1.0 cm C. 1.7 cm D. 2.0 cm

Correct answer is C. RATIONALE: ⎟⎠⎞

⎜⎝⎛ ×−

=HVL

tII

o

)2ln(exp , if t = x HVL, then

))2ln(exp()2ln(exp7.6

2 xHVL

xHVL×−=⎟

⎠⎞

⎜⎝⎛ ×−

= , and solving for x,

74.1693.0209.1

)2ln(1

7.62ln =

−−

=−

×⎟⎠⎞

⎜⎝⎛=x cm

145. What is the negative predictive value of PET imaging for 100 patients who have the

following results?

True positives: 36 True negatives: 18 False positives: 34 False negatives: 12

A. 74% B. 67% C. 60% D. 51% Correct answer is C. RATIONALE: The negative predictive value is defined as the number of true negatives/(number of true negatives + number of false negatives). In this case, 18/(18+12) = 60%. 146. Which of the following tumor marker serum levels is most common in a patient with a

pure germinomatous intracranial germ cell tumor? A. AFP of 100 ng/mL B. β-hCG of 30 IU/dL C. β-hCG of 3000 IU/dL D. AFP of 100 ng/mL and β-hCG of 30 IU/dL Correct answer is B. RATIONALE: β-hCG is produced by syncytiotrophoblastic giant cells that are often present in germinoma tissue; therefore, a pure germinomatous germ cell tumor may have a modest elevation of β-hCG (typically <50 IU/dL). Higher β-hCG levels are consistent with a non-germinomatous germ cell tumor, such as embryonal carcinoma or choriocarcinoma. Elevated serum AFP levels are diagnostic for a non-germinomatous germ cell tumor.

Page 49: Radiation Oncology ACR 2010 in-Service Exam Rationales

147. Which of the following whole-body diagnostic tests is most appropriate to evaluate a

patient with high-risk prostate cancer for metastasis? A. Bone scan B. PET/CT scan C. ProstaScint scan D. MRI Correct answer is A. RATIONALE: Patients with high-risk prostate cancer need to have a bone scan to evaluate or rule out metastatic disease. 148. What is the risk that a patient with FIGO stage IIIB cervical cancer will have paraaortic

lymph node involvement? A. 5% B. 15% C. 20% D. 30% Correct answer is D. RATIONALE: Patients with FIGO stage IB cervical cancer have a 5.6% risk for paraaortic lymph node involvement; patients with FIGO stage II-IV have a 29.3% risk for paraaortic lymph node involvement. REFERENCES: Perez. Principles and Practice of Gynecologic Oncology. 1st edition. 1992. Lagasse LD, et al. Results and complications of operative staging in cervical cancer: experience of the Gynecologic Oncology Group. Gynecologic Oncology. Feb 1980;9(1):90-8. 149. Which of the following statements about the 2009 AJCC TNM staging criteria for lung

cancer is true? A. Tumor size of >5 cm has been reclassified from stage T2 to stage T3. B. Multiple tumor nodules in the same lobe have been reclassified from stage T4 to

stage T3. C. The N stage classification has been changed to reflect the number of lymph nodes

involved. D. The M stage classification has not changed. Correct answer is B. RATIONALE: This item tests the summary changes of the 2009 AJCC lung cancer staging classifications: The T-stage classifications have been redefined: Stage T1 has been subclassified into stage T1a (≤2 cm in size) and stage T1b (2 cm to 3 cm in size). Stage T2 has been subclassified into stage T2a (3 cm to 5 cm in size) and stage T2b (5 cm to 7 cm in size). Stage T2 (>7 cm in size) has been reclassified as stage T3. Multiple tumor nodules in the same lobe have been reclassified from stage T4 to stage T3. Multiple tumor nodules in the same lung but a different lobe have been reclassified from stage M1 to stage T4. The N stage classifications have remained unchanged. The M stage classifications have been redefined. Stage M1 has been subdivided into stage M1a and stage M1b. Malignant pleural and pericardial effusions have been reclassified from stage T4 to stage M1a. Separate tumor nodules in the contralateral lung are considered stage M1a. Stage M1b designates distant metastases.

Page 50: Radiation Oncology ACR 2010 in-Service Exam Rationales

150. Which of the following factors increases the risk for CNS dissemination in a patient with

retinoblastoma? A. Extension into the orbital fat B. Invasion of >50% of the choroid C. Presence of multiple intraocular tumors D. Involvement of the optic nerve beyond the lamina cribrosa Correct answer is D. RATIONALE: Involvement of the optic nerve beyond the lamina cribrosa is a known risk factor for CSF spread because of the access of retinoblastoma cells to the subarachnoid space (the optic nerve is surrounded by meninges). REFERENCE: Halperin EC, Kirkpatrick JP. Retinoblastoma. Pediatric Radiation Oncology. 4th edition. Chapter 5. Philadelphia: Lippincott Williams & Wilkins Publishers. pp 135-176. 151. How do a tissue’s progenitor cells compare to its stem cells? A. Only progenitor cells can migrate to the site of a normal tissue injury. B. Tumors are thought to contain stem cells but not progenitor cells. C. Like stem cells, progenitor cells can proliferate indefinitely. D. Progenitor cells are totipotent, and stem cells are not. Correct answer is A. RATIONALE: Progenitor cells, sometimes referred to as "transit amplifying cells," arise originally from stem cells and undergo further differentiation; however, unlike true stem cells, they are not totipotent (although they may be pluripotent). Progenitor cells are also capable of further cell division on the path to terminal differentiation; however, they cannot proliferate indefinitely. Progenitor cells in normal tissues can be mobilized in response to injury by cytokine or growth factor activation. Tumors are thought to contain a small fraction of stem cells; however, progenitor cells make up the bulk of a tumor and are most responsive to cancer therapies. 152. When measuring the output of a linear accelerator, the temperature correction term

utilized must correct for temperature dependence of the: A. density of water. B. density of air in an ion chamber. C. coefficient of expansion of the ion chamber. D. electrometer sensitivity. Correct answer is B. RATIONALE: If an ion chamber is unsealed, it will be affected by changes in air temperature and pressure. According to the ideal gas law, P = nRT/V = ρRT/M (where ρ is density and M is the molar mass). If you look at this equation carefully, you will notice that P/T is proportional to the density, demonstrating that changes in these environmental conditions will affect the density of the air in the ion chamber.

Page 51: Radiation Oncology ACR 2010 in-Service Exam Rationales

153. Which of the following treatment outcomes is most likely to occur in a patient who has

occult primary breast cancer with axillary lymph node metastases? A. Fifteen percent of primary sites will be identified by performing an upper-outer

quadrantectomy. B. A local recurrence rate of 25% is seen with breast irradiation after breast-conserving

surgery. C. A local recurrence rate of 50% to 55% is noted in patients who do not receive breast

irradiation or a localized mastectomy. D. Overall survival rates are lower for patients with pathologic stage II (TXN1) disease

versus pathologic stage II (T1N2) disease. Correct answer is C. RATIONALE: After the diagnosis of adenocarcinoma has been established by surgical excision of an isolated axillary mass, extensive evaluation is not necessary. Evaluation of a patient who has occult primary breast cancer should include a thorough clinical examination, chest x-ray, bilateral mammograms, and tumor markers. Breast MRI should be used in instances where MRI-guided biopsy is feasible; otherwise contrast CT or ultrasound imaging may help evaluate for the breast primary site. Axillary dissection should be done to provide prognostic indicators (number of involved lymph nodes) and sufficient material for biomarkers. The breast should be treated to a dose of 50-55 Gy if mastectomy is not completed. The supraclavicular area and upper axilla should be treated if there is a Level I/II axillary lymph node dissection. The axilla should be treated if there is no lymph node dissection after excision of the axillary mass. The local recurrence rate with no breast therapy is ~50-55%. Radiation therapy to the breast reduces the rate of recurrence to ranges of 8-20% in reported series. The overall survival of patients without the finding of a primary site is superior to those of stage II patients in which a primary site is identified. REFERENCE: Fourquet A, Meunier M, Campana F. Occult primary cancer with axillary metastases. Harris JR, Lippman ME, Morrow M, Osborne CK, eds. Diseases of the Breast. 3rd edition. Chapter 65. Philadelphia: Williams & Wilkins Publishers. 2004;1047-1052. 154. Which of the following organs is an intraperitoneal structure? A. Rectum B. Ascending colon C. Descending colon D. Transverse colon Correct answer is D. RATIONALE: The transverse colon is an intraperitoneal structure, while the other three structures are retroperitoneal. 155. Medroxyprogesterone acetate is an appropriate systemic therapy for which of the

following uterine tumors? A. Carcinosarcoma B. Leiomyosarcoma C. Low-grade endometrial stromal sarcoma D. High-grade undifferentiated sarcoma Correct answer is C. RATIONALE: Hormonal therapy is only appropriate to treat low-grade endometrial stromal sarcoma when systemic therapy is chosen for uterine sarcomas. When a primary endometrial carcinoma is being treated, hormonal therapy is considered for endometrioid histologies only; therefore, it is not indicated for papillary serous carcinomas, clear cell carcinomas, or carcinosarcomas. Note that a carcinosarcoma is also called a malignant mixed Müllerian tumor (MMMT).

Page 52: Radiation Oncology ACR 2010 in-Service Exam Rationales

156. Which of the following statements about the use of a FDG-PET scan for patients with

NSCLC is true? A. It is useful in measuring the exact tumor size. B. It improves the accuracy of lymph node staging by 10% to 20% compared to a CT

scan. C. It has a high specificity and low sensitivity in detecting lymph node disease. D. It is not as sensitive as a bone scan in detecting metastasis of the bone. Correct answer is B. RATIONALE: A PET scan is very sensitive in detecting a high metabolic state, such as in tumors, but provides poorer spatial resolution in cross-sectional anatomy as compared to CT or MRI scans. A PET scan has been shown to be as sensitive as a bone scan in detecting bone metastasis. 157. A Type II error is defined as the probability that the: A. null hypothesis is true. B. alternative hypothesis to the null hypothesis is false. C. null hypothesis is accepted when the alternative hypothesis is true. D. alternative hypothesis is accepted when the null hypothesis is true. Correct answer is C. RATIONALE: This is the definition of a Type II error. 158. Which of the following management options would be best to recommend for a patient

with conjunctival follicular lymphoma? A. Rituximab for 6 months B. Involved-field radiation therapy of 25 to 35 Gy C. Combined multiagent chemotherapy and involved-field radiation therapy D. Orbital radiation therapy of 35 to 45 Gy Correct answer is B. RATIONALE: Conjunctival follicular lymphoma is a highly curable disease and can be treated effectively with conformal techniques. There is no dose response above 30 Gy. Rituximab can yield an initial good response; however, it may not be durable. Proper imaging with MRI is reliable in ruling out disease involvement beyond anterior structures. REFERENCE: Halperin EC, Perez CA, Brady LW, eds. Perez and Brady’s: Principles and Practice of Radiation Oncology. 5th edition. pp 793, 1752. 159. Approximately how many autosomal-recessive genetic disorders would be expected (per

Sievert per 106 births) for the first-generation progeny of irradiated parents? A. 3000 B. 750 C. 50 D. 0 Correct answer is D. RATIONALE: The expression of an autosomal-recessive trait would require that both parental alleles be mutated. According to the 2001 UNSCEAR report on hereditary effects of radiation, the risk of this occurring in first-generation progeny after a 1 Sv parental exposure is essentially zero.

Page 53: Radiation Oncology ACR 2010 in-Service Exam Rationales

160. What wedge angle would be most appropriate to minimize a hot spot during a treatment

using two oblique radiation beams with a hinge angle of 85°? A. 15o B. 30o C. 45o D. 60o

Correct answer is C. RATIONALE: Wedge angle = 90° – hinge angle/2 = 90° – 85°/2 = 90° – 42.5° = 47.5o, which is closest to a 45o wedge angle. 161. Which of the following management options is most appropriate for an intracranial non-

germinomatous germ cell tumor after surgical resection? A. Irradiation of the whole brain B. Irradiation of the tumor plus the surgical margin C. Platinum-based chemotherapy, followed by craniospinal irradiation D. Surveillance if postoperative imaging confirms gross total resection Correct answer is C. RATIONALE: Compared to intracranial germinomas, non-germinomatous germ cell tumors (NGGCT) have a worse prognosis. NGGCT are more resistant to radiation and should not be treated with radiation therapy alone. Several reports indicate that complete and partial responses can be achieved in 80% of the patients who have NGGCT and receive platinum-based chemotherapy. However, chemotherapy alone results in high rates of recurrence. Additionally, relapse rates are higher in patients undergoing local field radiation therapy (RT) only. Therefore, adjuvant craniospinal irradiation is recommended for all patients, followed by a boost to the primary site. REFERENCES: Balmaceda C, Heller G, Rosenblum M, et al. Chemotherapy without irradiation−a novel approach for newly diagnosed CNS germ cell tumors: results of an international cooperative trial. The First International Central Nervous System Germ Cell Tumor Study. Journal of Clinical Oncology (J Clin Oncol). 1996;14(11):2908-15. Kellie SJ, Boyce H, Dunkel IJ, et al. Primary chemotherapy for intracranial non-germinomatous germ cell tumors: results of the second international CNS germ cell study group protocol. Journal of Clinical Oncology (J Clin Oncol). 2004;22(5):846-53. 162. Which of the following statements about the use of radiation therapy for renal cell

carcinoma is true? A. Radiosurgery is effective in controlling more than 85% of brain metastases from

renal cell carcinoma. B. There is no role for radiation therapy since renal cell carcinoma is very

radioresistant. C. Combined radiation therapy and chemotherapy offer the best approach for

overcoming radioresistance in renal cell carcinoma. D. After nephrectomy is performed, radiation therapy combined with targeted therapy

has been shown to improve survival. Correct answer is A. RATIONALE: Radiosurgery is a very effective treatment for brain metastases from renal cell carcinoma. REFERENCE: Doh LS, Amato RJ, Paulino AC, Teh BS. Radiation therapy in the management of brain metastases from renal cell carcinoma. Oncology. May 2006;20(6):603-613.

Page 54: Radiation Oncology ACR 2010 in-Service Exam Rationales

163. Which of the following statements about the HPV vaccination is true? A. It targets eight HPV serotypes. B. It could reduce up to 70% of cervical cancers worldwide. C. It has been proven to be effective in reducing oropharyngeal cancer. D. It is recommended for all sexually active females. Correct answer is B. RATIONALE: HPV has been found to be associated with 70% of cervical cases worldwide. There are currently two different vaccines available: Cevarix, Glaxo Smith Kline, a bivalent vaccine against serotypes 16 and 18 and Gardasil, Merck and Co, a quadrivalent vaccine against serotypes 6, 11, 16, and 18. Current data suggests that it is effective in reducing cervical, vulvar, vaginal, and anogenital cancers with 93% efficacy. The data has not yet proven its efficacy in reducing oropharyngeal cancer. Current recommendations suggest vaccination of sexually inactive females only, though there is evidence to vaccinate sexually active women who are seronegative for HPV. REFERENCE: International Journal of Gynecological Cancer. October 2009;19(7):1166-76. 164. Which of the following treatments would be most appropriate for a patient with poor

pulmonary function and stage I NSCLC involving the right hilum? A. Radiofrequency ablation B. Conventional fractionated radiation therapy C. Stereotactic body radiation therapy D. Wedge resection, followed by brachytherapy Correct answer is B. RATIONALE: The 2009 Practice Guidelines by the National Comprehensive Cancer Network (NCCN) recommends conventional fractionated radiation therapy for medically inoperable early-stage non-small cell lung cancer (NSCLC). Hilar involvement precludes stereotactic body radiation therapy. 165. Approximately what percent of patients with hereditary retinoblastoma develop

secondary malignancies within 50 years of follow-up? A. 15% B. 30% C. 50% D. 65% Correct answer is C. RATIONALE: The incidence of secondary malignant neoplasm (SMN) in patients with hereditary retinoblastoma is very high (reported to be 51% in 50 years in one large study). REFERENCE: Wong FL, et al. Cancer incidence after retinoblastoma. Radiation dose and sarcoma risk. JAMA. Oct 15, 1997;278(15):1262-7.

Page 55: Radiation Oncology ACR 2010 in-Service Exam Rationales

166. The limiting dilution assay has been used to assess the radiosensitivity of murine: A. embryonic fibroblasts. B. bone marrow stem cells. C. lymphocytic leukemia cells. D. lung adenocarcinoma cells. Correct answer is C. RATIONALE: Hewitt and Wilson first developed the limiting dilution assay in the late 1950s as a means of determining the radiation dose response curve for murine lymphocytic leukemia in vivo. (The method has since been adapted for use with several other experimental tumors.) Since the assay endpoint is transmission of the leukemia to recipient mice and not the counting of colonies, it is necessarily a non-clonogenic assay. One notable finding of the original study was that it took the injection of, on average, only two (previously unirradiated) leukemia cells to transmit the disease to 50% of syngeneic mice. REFERENCES: Joiner and van der Kogel. Basic Clinical Radiobiology. 4th edition. 2009. Chapter 4. Hall and Giaccia. Radiobiology for the Radiologist. 6th edition. Chapter 20. 167. Charged particle equilibrium exists in a small mass when: A. absorption of photon radiation becomes exponential. B. the number of electrons entering the mass is equal to the number of electrons

leaving it. C. the number of x-rays entering the mass is equal to the number of x-rays leaving it. D. the build-up of dose begins to slow down. Correct answer is B. RATIONALE: Charged particle equilibrium is discussed in chapter 6 of Khan's Physics of Radiation Therapy. To measure dose correctly to a small volume of tissue – mini-phantom − the charged particle equilibrium needs to be attained, and this is an important, yet basic, concept of radiation dose measurements. 168. What is the recommended V20 value for patients who receive IMRT after extrapleural

pneumonectomy for mesothelioma? A. <7% B. 17% C. 23% D. 30% Correct answer is A. RATIONALE: The standard recommendation of keeping V20 below 35% for patients who have lung cancer (when both lungs are counted as the total lung volume) and are receiving thoracic radiation therapy (RT) does not apply to patients with mesothelioma following pneumonectomy. The remaining lung has to be protected as much as possible from RT, and several institutions (Beth Israel/Harvard; Duke; MD Anderson) described severe pulmonary toxicity if the V20 was not kept below 7%. In addition, V5 is emerging as an important parameter to watch, since IMRT frequently “floods” the remaining lung with low-dose RT.

Page 56: Radiation Oncology ACR 2010 in-Service Exam Rationales

169. Which of the following features is included in the Nottingham histological grading system

for breast cancer? A. Cell size B. Cell shape C. Stromal pattern D. Tubule formation Correct answer is D. RATIONALE: The Nottingham histological grading system is based on the Scarff-Bloom-Richardson grading system. The Nottingham system assigns a grade for tubule formation, nuclear pleomorphism, and mitotic cell count. These three features are selected because they are more quantitative than subjective and thereby provide better inter-observer agreement. These features also have a strong correlation with prognosis. All invasive breast cancers should be graded except medullary carcinomas. REFERENCES: Harris JR. Staging of breast cancer. Harris JR, Lippman ME, Morrow M, Osborne CK, eds. Diseases of the Breast. 3rd edition. Chapter 38. Philadelphia: Lippincott Williams & Wilkins Publishers. 2004;659-660. Elston CW, Ellis IO. Pathological prognostic factors in breast cancer: The value of histological grade in breast cancer: experience form a large study with long-term follow up. Histopathology. 1991;19:403-410. Fitzgibbons PL, Page DL, Weaver D, et.al. Prognostic factors in breast cancer. College of American Pathologists consensus statement 1999. Archives of Pathology & Laboratory Medicine (Arch Pathol. Lab Med). 2000;124:966-978. 170. Which of the following characteristics is associated with the Intergroup 0130 randomized

trial that evaluated postoperative chemoradiation for patients with high-risk colon cancer?

A. Only patients with stage T4 disease were eligible. B. Patients with negative surgical margins were excluded. C. Radiation therapy had no disease-free survival benefit. D. Radiation therapy improved overall survival. Correct answer is C. RATIONALE: The Intergroup 0130 was a randomized trial that evaluated the addition of adjuvant radiation therapy to chemotherapy in patients with resected colon cancer. Patients were eligible if they had stage T3N+ or T4 disease. The trial was closed due to poor accrual, but there was no significant difference in either disease-free or overall survival. 171. What postoperative management option would be most appropriate for a pathologic

stage II low-grade endometrial stromal sarcoma? A. Observation B. Pelvic radiation therapy only C. Vaginal brachytherapy only D. Pelvic radiation therapy with vaginal brachytherapy Correct answer is A. RATIONALE: Series of low-grade endometrial stomal sarcomas suggest long-term disease-free intervals in the absence of specific therapy and offer less support for the use of adjuvant radiation therapy. Adjuvant radiation therapy has been demonstrated to reduce local recurrence rates but again with limited effect on survival. REFERENCES: 2009 Practice Guidelines by the National Comprehensive Cancer Network (NCCN). Gynecologic Oncology. 1990;36:60-65 and 113-118. International Journal of Radiation Oncology, Biology, Physics (Int J Radiat Oncol Bio Phys). 2001;49:739-748.

Page 57: Radiation Oncology ACR 2010 in-Service Exam Rationales

172. Cranial radiation therapy is most appropriate for adults who have: A. CNS-positive leukemia and are receiving CNS-directed therapy in conjunction with

intrathecal chemotherapy. B. acute lymphoid leukemia with any presentation and did not receive intrathecal

chemotherapy or CNS-directed therapy. C. acute lymphoid leukemia due to their high risk for disease-related CNS relapse. D. acute myeloid leukemia due to their high risk for disease-related CNS relapse. Correct answer is A. RATIONALE: CNS prophylaxis, although effective at reducing the incidence of CNS relapse, has no significant effect on systemic relapse or overall survival, and cranial irradiation is used in conjunction with intrathecal chemotherapy for CNS-positive leukemia. REFERENCE: De Vita, et al. Principles and Practice of Oncology. 173. Which of the following outcomes is most likely to occur if a pregnant woman receives

2.5 Sv of radiation during the preimplantation stage of gestation? A. Microcephaly B. Spina bifida C. Embryonic death D. Mental retardation Correct answer is C. RATIONALE: Animal studies indicate a lower frequency of births when irradiation occurs very early in gestation, presumably due to death of the embryo. Based on related studies of Japanese A-bomb survivors, excess cases of congenital malformations, microcephaly, and mental retardation were more characteristic of irradiation later in pregnancy, during the organogenesis and early fetal stages (approximately 4-25 weeks gestation). REFERENCE: Hall and Giaccia. Radiobiology for the Radiologist. 6th edition. 2006. Chapter 12. 174. The electrons produced by a linear accelerator operated in electron mode will pass

through which of the following structures before exiting the linear accelerator? A. Scattering foil B. Flattening filter C. Target D. Klystron Correct answer is A. RATIONALE: Of the items listed, the generated electrons that can be used clinically will only pass through the scattering foil before exiting the linear accelerator (linac). The target and flattening filter are only in place when the linac is operated in photon mode. The target is used to convert electrons to bremsstrahlung x-rays, and the flattening filter is used to flatten the forward-directed x-rays at a specified depth in phantom (typically 10 cm). Lastly, a klystron is a microwave amplifier used to power and accelerate the electrons through the accelerator waveguide. 175. Sensitivity can be defined as the probability of: A. correctly classifying a person with a disease. B. correctly classifying a person without a disease. C. falsely classifying a person with a disease. D. falsely classifying a person without a disease. Correct answer is A. RATIONALE: This is the definition of sensitivity.

Page 58: Radiation Oncology ACR 2010 in-Service Exam Rationales

176. What is the 5-year overall survival rate for patients with glioblastoma treated with

temozolomide (TMZ) and radiation therapy compared to radiation therapy (RT) alone? TMZ + RT RT alone A. 40% 10% B. 27% 11% C. 10% 2% D. 2% 1% Correct answer is C. RATIONALE: The addition of TMZ to RT increases overall survival for patients with glioblastoma multiforme. The 2-year overall survival rate with RT plus temozolomide is 27%, and the 5-year overall survival rate is 10%. REFERENCE: Stupp R, Hegi ME, Mason WP, et al. Effects of radiotherapy with concomitant and adjuvant temozolomide versus radiotherapy alone on survival in glioblastoma in a randomized phase III study: 5-year analysis of the EORTC-NCIC trial. Lancet Oncology. 2009;10:459-466. 177. What is the most common site of lymph node metastasis in a patient with penile cancer? A. Inguinal B. Obturator C. Paraaortic D. Hypogastric Correct answer is A. RATIONALE: Approximately 20% of patients with clinically nonpalpable inguinal lymph nodes will have micrometastases. Pathologic evidence of lymph node metastases is reported in approximately 35% of all patients with penile cancer and in approximately 50% of those patients with palpable lymph nodes. 178. Oral cavity cancer most commonly occurs in which of the following sites in both

American men and women? A. Lip B. Oral tongue C. Floor of mouth D. Buccal mucosa Correct answer is B. RATIONALE: According to SEER 2007 data: Option B, the oral tongue, is the most common site for cancer of the oral cavity (38%). Option C, the floor of mouth, is the second most common site (26%). These two sites are followed by all other sites of the mouth: Option D, buccal mucosa (18%); and Option A, the lip (18%).

Page 59: Radiation Oncology ACR 2010 in-Service Exam Rationales

179. Which of the following treatment strategies is most appropriate for a patient who has

stage IIIB NSCLC with KPS of >70? A. Concurrent chemotherapy and fractionated radiation therapy B. Neoadjuvant chemotherapy, followed by concurrent chemoradiation C. Conventional fractionated radiation therapy, followed by chemotherapy D. Oligo fractionated stereotactic radiation therapy, followed by chemotherapy Correct answer is A. RATIONALE: The standard treatment for a patient with stage IIIB non-small cell lung cancer (NSCLC) is concurrent chemoradiation with fractionated radiation therapy. This is supported by at least three randomized phase III trials: The West Japanese study, Czech Republic study, and RTOG 9410. The CALGB 39801 study did not show a significant advantage of survival in patients treated with neoadjuvant carboplatin and Taxol before concurrent chemoradiation. The HOG Phase III study failed to show a significant benefit of adjuvant docetaxel (Taxotere) chemotherapy, while the Locally Advanced Multimodality Project (LAMP) did report longer median survival in patients treated with fractionated radiation therapy with concurrent and adjuvant carboplatin and Taxol. REFERENCES: Zatloukal, et al. Concurrent versus sequential chemoradiotherapy with cisplatin and vinorelbine in locally advanced non-small cell lung cancer: a randomized study. Lung Cancer. Oct 2004;46(1):87-98. Furuse, et al. Phase III study of concurrent versus sequential thoracic radiotherapy in combination with mitomycin, vindesine, and cisplatin in unresectable stage III non-small cell lung cancer (NSCLC). Journal of Clinical Oncology (J Clin Oncol). Sep 1999;17(9):2692-9. RTOG 9410 is to be published soon. Fournel, et al. Randomized phase III trial of sequential chemoradiotherapy compared with concurrent chemoradiotherapy in locally advanced non-small cell lung cancer: Groupe Lyon-Saint-Etienne d'Oncologie Thoracique-Groupe Français de Pneumo-Cancérologie NPC 95-01 Study. Journal of Clinical Oncology (J Clin Oncol). Sep 2005 1;23(25):5910-7. Hanna, et al. Phase III study of cisplatin, etoposide, and concurrent chest radiation with or without consolidation docetaxel in patients with inoperable stage III non-small cell lung cancer: the Hoosier Oncology Group and U.S. Oncology. Journal of Clinical Oncology (J Clin Oncol). Dec 10, 2008;26(35):5755-60. Belani, et al. Combined chemoradiotherapy regimens of paclitaxel and carboplatin for locally advanced non-small cell lung cancer: a randomized phase II locally advanced multi-modality protocol. Journal of Clinical Oncology (J Clin Oncol). Sep 1, 2005;23(25):5883-91. Epub 2005 Aug 8. Erratum in: Journal of Clinical Oncology (J Clin Oncol). Apr 20, 2006;24(12):1966. 180. Which of the following radiation treatments is most appropriate for acute lymphoblastic

leukemia involving the testes? A. 18 Gy in 10 fractions B. 24 Gy in 12 fractions C. 30 Gy in 15 fractions D. 36 Gy in 18 fractions Correct answer is B. RATIONALE: Because the testicles are regarded as a sanctuary site, testicular radiation therapy to a dose of 20-24 Gy in 2-Gy fractions is given as a consolidative therapy after chemotherapy. REFERENCES: Hustu HO, et al. Extramedullary leukemia. Clinical Haematology (Clin Haematol). 1978;7(2):313-337. Bowman WP, et al. Isolated testicular relapse in acute lymphocytic leukemia of childhood: categories and influence on survival. Journal of Clinical Oncology. Aug 1984;2(8):924-9. Kun L. Leukemias in children. Pediatric Radiation Oncology. 4th edition. Chapter 2. Philadelphia: Lippincott Williams & Wilkins Publishers. pp 15-39.

Page 60: Radiation Oncology ACR 2010 in-Service Exam Rationales

181. Which of the following statements about radiation carcinogenesis from diagnostic

imaging procedures is FALSE? A. The overall risk of radiation carcinogenesis increases with a patient’s age at

exposure. B. An increase in cancer mortality has been observed in Japanese A-bomb survivors

exposed to doses similar to those delivered by today's CT scanners. C. The average annual effective dose in the United States has nearly doubled since

1980, mostly due to the dramatic increase in the use of CT scanning. D. Medical exposure now surpasses natural background radiation exposure as the

leading contributor to the average annual effective dose in the United States. Correct answer is A. RATIONALE: The major sources of radiation exposure to the U.S. population are natural background and medical exposure, with the contribution of the latter having increased dramatically in the last 30 years due to the proliferation of relatively high-dose medical imaging procedures, particularly CT scanning. Medical radiation exposure now surpasses natural background radiation exposure as the leading contributor to the average annual effective radiation dose in the United States (51% versus 48%, respectively), currently estimated to be approximately 6.3 mSv. Concern about the possible carcinogenic risk of imaging procedures has heightened in recent years due to epidemiological findings among the Japanese A-bomb survivors of a statistically significant increased risk of cancer mortality after exposure to radiation doses comparable to those delivered by CT scanners. This is especially concerning given that CT scanning has increased disproportionately in the pediatric population, who are both inherently more sensitive to radiation and have the longest potential lifespan to manifest a cancer. REFERENCE: Mettler and Upton. Medical Effects of Ionizing Radiation, 3rd edition. 2008. Chapter 2. 182. For 100-cm isocentric linear accelerators, the decrease in dose deposition due to the

inverse square factor for 1 cm depth in tissue is approximately: A. 1.0%. B. 1.5%. C. 2.0%. D. 2.5%. Correct answer is C. RATIONALE: For 100-cm isocentric machines, the loss of photon fluence per 1 cm of distance is approximately (100/101)2 = 0.98. This relationship holds for a relatively large range of distances around 100 cm (i.e. (90/91)2 = 0.978 and (110/111)2 = 0.982.

Page 61: Radiation Oncology ACR 2010 in-Service Exam Rationales

183. An MRI is most likely indicated for breast cancer screening or staging when a patient: A. has silicone implant augmentation. B. has breast pain with no findings on mammography. C. is pregnant and has a newly diagnosed breast cancer. D. needs to be evaluated for the extent of DCIS involvement. Correct answer is D. RATIONALE: MRI is useful in evaluating the extent of DCIS involvement. In the case of extensive intraductal component (EIC), MRI has a 95% accuracy rate vs. 36% for mammography and ultrasound. MRI is not indicated for evaluating breast pain; clinical evaluation and ultrasound are the primary studies to be completed. MRI is not indicated for screening patients with silicone augmentation. Mammography with implant imaging protocols is used for screening purposes. The safety of gadolinium-enhanced contrast agents during pregnancy is not widely established at this time. REFERENCE: Berg WA, Birdwell, RL, et al, eds. Imaging modalities – magnetic resonance imaging. Diagnostic Imaging. Salt Lake City: Amirsys Publishers. 2006; part II-0: 36-37. 184. Which of the following features is associated with acute lymphoid leukemia? A. Adult patients treated with an intense chemotherapy regimen can achieve cure rates

as high as those observed in children. B. A patient’s ability to achieve a complete response after induction chemotherapy is

one of the strongest predictors of disease outcome. C. High-risk patients who are positive for the Philadelphia chromosome should receive

allogeneic bone marrow transplantation. D. It is more prevalent than acute myeloid leukemia in the U.S. population. Correct answer is B. RATIONALE: Acute myeloid leukemia (AML) is more prevalent in the U.S. population, with 9000 cases per year, when compared with acute lymphoid leukemia (ALL), with 4000 cases per year. Achieving a complete response after induction chemotherapy is one of the strongest prognosticators in predicting a patient’s outcome. High-risk patients may benefit from myeloablative treatment with allogeneic bone marrow transplantation, and TBI is the preferred conditioning regimen for a superior outcome. REFERENCE: De Vita, et al. Principles and Practice of Oncology. 185. Which of the following stages is most appropriate for a patient who has colon cancer that

invades the subserosa with five positive lymph nodes? A. T2N1 (IIIA) B. T3N1 (IIIB) C. T3N2 (IIIC) D. T4N2 (IIIC) Correct answer is C. RATIONALE: A tumor that has invaded the subserosa is a stage T3 lesion, and involvement of four or more lymph nodes is stage N2 disease.

Page 62: Radiation Oncology ACR 2010 in-Service Exam Rationales

186. According to the ASTEC surgical trial, women who had a pelvic lymphadenectomy to

treat stage I endometrial carcinoma experienced: A. improved overall survival and recurrence-free survival. B. improved overall survival but no benefit in recurrence-free survival. C. improved recurrence-free survival but no benefit in overall survival. D. no benefit in overall survival or in recurrence-free survival. Correct answer is D. RATIONALE: The ASTEC surgical trial included more than 1400 women with stage I endometrial cancer, randomized to receive a hysterectomy and bilateral salpingo-oophorectomy (BSO) with or without pelvic lymphadenectomy. A median of 12 lymph nodes were removed in patients who received lymphadenectomy. Results showed no evidence of benefit in overall or recurrence-free survival for pelvic lymphadenectomy in women with early-stage endometrial cancer. REFERENCE: The Lancet. January 10, 2009;373(9658):125-136. 187. Which of the following conditions is a contraindication for extrapleural pneumonectomy

in patients with mesothelioma? A. Pericardial pleural involvement B. Presence of multiple ipsilateral pleural implants C. Tumor extension through the diaphragm D. Epithelial histologic subtype Correct answer is C. RATIONALE: Practically all cases of mesothelioma will have multiple pleural implants along the chest wall, all of which will be removed during pleurectomy and as such would not constitute a contraindication to surgery. Pericardial pleura can be resected. Extension through the diaphragm is a classic contraindication to complete resection. Epithelial histologic subtype has a better prognosis than the sarcomatoid subtype and is more likely to have prolonged survival after surgery. 188. Which of the following is an anti-angiogenic factor? A. Interleukin-8 (IL-8) B. Thrombospondin-1 (TSP-1) C. Vascular endothelial growth factor (VEGF) D. Basic fibroblast growth factor (bFGF) Correct answer is B. RATIONALE: Thrombospondin-1 (TSP-1) behaves as an anti-angiogenic factor. It is an extracellular matrix glycoprotein shown to inhibit matrix metalloproteinase 9 (MMP-9), which is responsible for the release of VEGF sequestered in the extracellular matrix. TSP-1 also decreases vascular endothelial cell motility, adhesion, and growth. IL-8, VEGF, and bFGF have been shown to be pro-angiogenic factors. 189. What is the primary photon interaction when a megavoltage beam strikes a patient? A. Photoelectric effect B. Compton scattering C. Electron scattering D. Bremsstrahlung Correct answer is B. RATIONALE: Compton scattering is the dominant photon interaction when a megavoltage beam strikes a patient.

Page 63: Radiation Oncology ACR 2010 in-Service Exam Rationales

190. A test is performed to determine the presence of a malignancy, and its results are

compared with the actual incidence of the disease. What is the specificity of the test based on the findings shown below?

Positive Test (105 patients) Negative Test (475 patients) Presence of malignancy 56 14 Absence of malignancy 49 461

A. 53.3% B. 80.0% C. 90.4% D. 97.1% Correct answer is C. RATIONALE: The specificity of a test is defined as the percentage of individuals without a disease who are correctly classified as not having the disease. In this case, there were 49 + 461 = 510 subjects without the lesion. Of these subjects, 461 were correctly predicted. Thus, the specificity is 100 x 461/510 = 90.4%. 191. An MRI scan performed 1 month after completion of chemoradiation with temozolomide

for glioblastoma reveals enlargement of the contrast-enhanced lesion. Which of the following management options would be most appropriate?

A. Second-line chemotherapy B. Continuation of temozolomide C. Resection of the lesion D. SRS to the lesion Correct answer is B. RATIONALE: Enlargement of the contrast-enhanced tumor 1 month after chemoradiotherapy with temozolomide (TMZ) may occur in approximately 50% of patients. However, with repeat imaging 2 months later, 64% displayed stable or reduced enhancement, indicative of pseudo-progression rather than true early progressive disease. Median overall survival for patients with pseudo-progression was 38 months versus 10 months for true progression. Given the incidence of pseudo-progression, the standard planned treatment with adjuvant TMZ is typically continued until further imaging is obtained several months later. REFERENCE: Brandes AA, Franceschi E, Tosoni A, et al. MGMT promoter methylation status can predict the incidence and outcome of pseudo-progression after concomitant radiochemotherapy in newly diagnosed glioblastoma patients. Journal of Clinical Oncology (J Clin Oncoll). 2008;26:2192-2197. 192. Which of the following histologic subtypes of urethral carcinoma occurs most commonly

in men? A. Squamous cell carcinoma B. Transitional cell carcinoma C. Sarcomatoid carcinoma D. Adenocarcinoma Correct answer is A. RATIONALE: Approximately 80% of urethral carcinomas in men are diagnosed as having a histologic subtype of squamous cell carcinoma, followed by transitional cell carcinoma (15%) and adenocarcinoma (5%).

Page 64: Radiation Oncology ACR 2010 in-Service Exam Rationales

193. Which of the following oral cavity cancers has the worst prognosis? A. Oral tongue B. Floor of mouth C. Alveolar ridge D. Buccal gingiva

Correct answer is A. RATIONALE: According to SEER data, a lip primary has the best prognosis with a 10-year survival rate of 87%, followed by the gums (70%) and the floor of mouth (48%). The alveolar ridge and buccal gingiva are both portions of the gums. Oral tongue cancer has the worst 10-year survival rate of 45%. 194. Which of the following adverse effects most commonly occurs in a patient with lung

cancer after administration of concurrent chemoradiation? A. Pneumonitis in 2 to 4 weeks during radiation treatment B. Esophagitis in 3 to 4 weeks during radiation treatment C. Pericarditis in 1 to 2 months after radiation treatment D. Myelitis in 5 to 6 months after radiation treatment Correct answer is B. RATIONALE: Esophagitis is the most common side effect (about 40%-50%) during treatment in patients treated with concurrent chemoradiation, according to all of the phase III trials of patients treated with concurrent chemoradiation therapy. Esophagitis often occurs at 3-4 weeks from the commencement of radiation therapy. 195. Myeloablative therapy for childhood leukemia: A. is used as consolidative therapy after initial remission in children with acute myeloid

leukemia. B. is indicated for children who have acute lymphoblastic leukemia without the

Philadelphia chromosome. C. does not include total body irradiation as part of the bone marrow transplant

conditioning regimen. D. requires an allogeneic bone marrow transplant. Correct answer is A. RATIONALE: Myeloablative therapy is indicated in children who have acute lymphoblastic leukemia (ALL) with the Philadelphia chromosome (BCR-ABL or MLL-AF4). Cyclophosphamide/TBI as a conditioning regimen is better than busulfan cyclophosphamide for leukemia control. Both allogeneic and autologous bone marrow transplantations are effective in treating leukemias. REFERENCE: Kun L. Leukemias in children. Pediatric Radiation Oncology. 4th edition. Chapter 2. Philadelphia: Lippincott Williams & Wilkins Publishers. pp 15-39.

Page 65: Radiation Oncology ACR 2010 in-Service Exam Rationales

196. Which of the following free radical interactions secondary to the radiolysis of water would

protect a cell from further DNA damage? A. e- + H2O → e-

aq B. [H2O+] → HO• + H+ C. e-

aq + H+ → H• D. H• + H• → H2 Correct answer is D. RATIONALE: The free radicals set into motion by the radiolysis of water are believed to be responsible for about 70% of the total DNA damage produced by ionizing radiation. On occasion, two free radicals will interact with each other instead of the DNA (or other important bio-molecules), effectively "detoxifying" themselves. This action removes potentially damaging free radicals from the cell. 197. Compared to kilovoltage CT imaging, megavoltage CT imaging has which of the

following advantages? A. Reduced low-Z material artifacts B. Reduced high-Z material artifacts C. Improved soft tissue delineation D. Improved spatial integrity Correct answer is B. RATIONALE: Due to reduced dependence on photoelectric interactions for image creation and mainly Compton interactions, megavoltage (MV) CT imaging is largely unaffected by the Z number of imaged tissues. For kilovoltage (kV) CT imaging, the major mode of radiation interaction is the photoelectric effect, where Z dependence is significant and high-Z objects can cause significant image artifacts. 198. Which of the following statements about phyllodes tumors arising in the breast is true? A. Mastectomy is the treatment of choice. B. Initial management should include radiation therapy. C. They tend to occur in women about 10 years earlier than is typical for palpable

fibroadenomas. D. Mammographic findings demonstrate a circumscribed mass without calcifications. Correct answer is D. RATIONALE: Phyllodes tumors clinically resemble fibroadenomas with the following exceptions: (a) They tend to present in women about 10 years later than typical palpable fibroadenomas, (b) they are capable of rapid growth, and (c) in advanced cases, skin ulceration and open non-healing wounds can occur. Phyllodes tumors should be suspected based on clinical criteria (e.g., older age, rapid growth, or large tumor size). Imaging and tissue sampling have serious limitations with this tumor. Mammographic appearance is primarily a large, circumscribed mass without calcifications. The mean size is 4-5 cm, but the tumor can occupy the entire breast. Wide excision appears to be the most important aspect of therapy. When phyllodes tumors are resected with a narrow margin, reexcision should be performed. Distant metastases are rarely seen, even with malignant phyllodes tumors, but are more common among tumors exhibiting stromal overgrowth. Histologic differentiation between benign, borderline, and malignant phyllodes tumors only loosely correlates with distant metastatic rates. Radiation therapy has no role in the initial management of primary phyllodes tumors. However, phyllodes tumors that are initially excised with wide margins or mastectomy and that then recur locally may warrant adjuvant chest wall radiation after reexcision because of the high morbidity of additional recurrences. REFERENCES: Anderson BO, Lawton TJ,

Page 66: Radiation Oncology ACR 2010 in-Service Exam Rationales

Lehman CD, Moe RE. Phyllodes tumors. Harris JR, Lippman ME, Morrow M, Osborne CK, eds. Diseases of the Breast. 3rd edition. Chapter 61. Philadelphia: Lippincott Williams & Wilkins Publishers. 2004;991-1006. Berg WA, Birdwell, RL, et al., eds. Histopathologic diagnoses–phyllodes tumor. Diagnostic Imaging. Salt Lake City: Amirsys Publishers. 2006;part IV-2:96-102. 199. Which of the following treatments would be most appropriate for a patient who has anal

canal cancer, is HIV positive, and has a CD4 count of 800 cells/mm3? A. Radiation therapy alone to a reduced field B. Chemoradiation with 5-FU and mitomycin C C. Chemoradiation with 5-FU alone D. Chemotherapy alone Correct answer is B. RATIONALE: Standard chemoradiation is an appropriate treatment for anal canal cancer in a patient who is HIV positive and has an adequate CD4 count. 200. What is the incidence of pelvic lymph node metastasis for a grade 1 adenocarcinoma of

the endometrium with inner one-third myometrial invasion? A. ≤5% B. 10% C. 15% D. 20% Correct answer is A. RATIONALE: Based on a prospective evaluation (by the GOG and others) of surgicopathologic patterns of spread in patients with endometrial cancer, it is recognized that much of the adverse prognosis associated with intrauterine risk factors is mediated through lymph node involvement. The incidence of pelvic lymph node metastases is 5% or less for grade 1 or grade 2 tumors with inner one-third myometrial invasion. For tumors with outer one-third myometrial invasion, lymph node disease was found in 19% of grade 2 cancers and in 34% of grade 3 cancers. REFERENCES: Cancer. 1987;60:2035-2041. International Journal of Gynecologic Cancer. 2008;18(2):269-273. 201. According to the TNM mesothelioma staging system, a stage T3 tumor involves the: A. ipsilateral visceral pleura. B. ipsilateral parietal pleura. C. chest wall in one focal area. D. vertebral body. Correct answer is C. RATIONALE: Mesothelioma evolves from the parietal pleura; therefore, involvement of either parietal or visceral pleura denotes an early-stage tumor. A focal (in contrast to diffuse) invasion of the chest wall is still potentially resectable; therefore, it is a stage T3 tumor. Spinal invasion is not resectable and is a stage T4 tumor.

Page 67: Radiation Oncology ACR 2010 in-Service Exam Rationales

202. A 3½-year-old patient who has average-risk medulloblastoma should receive which of

the following craniospinal radiation therapy doses as part of a total radiation dose of 54 Gy before chemotherapy?

A. 13.2 Gy B. 23.4 Gy C. 30.6 Gy D. 36.0 Gy Correct answer is B. RATIONALE: The current North American standard of craniospinal irradiation (CSI) in patients older than 3 years of age is 23.4 Gy with a boost to 54 to 55.8 Gy to the posterior fossa, followed by chemotherapy. Studies are underway to evaluate the avoidance of radiation therapy (RT) vs. the use of a lower dose of 18 Gy CSI. Some studies use weekly vincristine during RT, others do not. All studies use post-RT chemotherapy. 203. Which of the following statements about hypoxia-inducible factor-1 (HIF-1) is true? A. It is a membrane-bound receptor tyrosine kinase. B. It is composed of three protein subunits: α, β, and γ. C. The β subunit of HIF-1 is upregulated under hypoxic conditions. D. Stabilization of the α subunit activates the HIF-1 transcription factor. Correct answer is D. RATIONALE: HIF-1 is a protein transcription factor consisting of the α and β subunit. The β subunit is constitutively active in cells; however, it is the α subunit that is upregulated under hypoxic conditions. Under normoxic conditions, the α subunit is hydroxylated on prolyl residues, allowing it to bind to the von Hippel Lindau (VHL) protein, which leads to its degradation. As the oxygen tension drops, however, the α subunit is no longer hydroxylated, does not bind to VHL, and is not degraded. This process allows it to enter the nucleus and act (in concert with the β subunit) as a transcription factor that activates dozens of other genes involved in the hypoxic response. 204. Which of the following American Association of Physicists in Medicine Task Group

(AAPM TG) clinical protocols is currently recommended for calibration dosimetry of external photon and electron beams in the United States?

A. AAPM TG21 B. AAPM TG43 C. AAPM TG51 D. AAPM TG64 Correct answer is C. RATIONALE: The AAPM TG-51 absorbed-dose based calibration protocol is currently used. The AAPM TG-21 protocol is an older, air-kerma based calibration protocol, which is no longer recommended. AAPM TG-43 and AAPM TG-64 are brachytherapy protocols.

Page 68: Radiation Oncology ACR 2010 in-Service Exam Rationales

205. Which of the following findings is most likely to support a diagnosis of inflammatory

breast cancer? A. Skin erythema affecting at least one third of the breast B. Non-blanchable erythematous skin nodules C. Tumor emboli within the dermal lymph nodes D. Breast enlargement Correct answer is A. RATIONALE: Inflammatory breast cancer (IBC) is characterized by the following features: skin discoloration affecting at least one-third of the breast; thickening or fine dimpling (peau d'orange) of at least one third of the breast; edema or warmth; a palpable ridge present at the margin of induration; and rapid onset of symptoms (several weeks to 6 mos.). Common features seen with IBC but not part of its definition include erythematous, nonblanchable nodules; breast pain; ecchymosis; and tumor emboli with dermal lymphatics. REFERENCE: Harris J., et al, eds. Inflammatory breast cancer. Diseases of the Breast. 4th edition. Philadelphia: Lippincott Williams & Wilkins Publishers. 2009;762. 206. Which of the following characteristics are associated with the best overall survival in a

patient with glioblastoma? A. Age of <30 years; unmethylated MGMT B. Age of <50 years; methylated MGMT C. Age of >50 years; unmethylated MGMT D. Age of >70 years; methylated MGMT Correct answer is B. RATIONALE: Epigenetic silencing of the MGMT DNA repair gene by promoter methylation compromises DNA repair and has been associated with longer survival in patients with glioblastoma who receive alkylating agents. Younger patients have a better survival rate than older patients. REFERENCES: Hegi ME, Diserens AC, Gorlia T, et al. MGMT gene silencing and benefit from temozolomide in glioblastoma. New England Journal of Medicine (NEJM). 2005;352:997-1003. Stupp R, Hegi ME, Mason WP, et al. Effects of radiotherapy with concomitant and adjuvant temozolomide versus radiotherapy alone on survival in glioblastoma in a randomized phase III study: 5-year analysis of the EORTC-NCIC trial. Lancet Oncology. 2009;10:459-466. 207. Penile carcinoma invading the urethra or prostate should be classified as stage: A. T1. B. T2. C. T3. D. T4. Correct answer is C. RATIONALE: According to AJCC staging, a T1 tumor invades subepithelial connective tissues, a T2 tumor invades the corpus spongiosum or cavernosum, a T3 tumor invades the urethra or prostate, and a T4 tumor invades other adjacent structures.

Page 69: Radiation Oncology ACR 2010 in-Service Exam Rationales

208. What is the expected 5-year survival rate for a patient who has oral cavity cancer with

distant metastasis? A. 50% B. 30% C. 10% D. 5% Correct answer is B. RATIONALE: According to SEER data, regional lymph node metastasis decreases the 5-year survival rate by 40%. Distant metastasis further reduces the survival rate to about 30%, ranging from 27% for oral tongue to 37% for lip cancer. 209. Which of the following factors is typically required when stereotactic body radiation

therapy is used to treat lung tumors? A. Peripheral tumor location B. Negative mediastinoscopic results C. Photon energy of >15 MV D. CTV expansion Correct answer is A. RATIONALE: While 20% of patients with clinical early-stage lung cancer may have involved lymph nodes, patterns of failure from stereotactic body radiation therapy (SBRT) series do not demonstrate a high lymph node failure rate. Currently, CTV expansions are NOT allowed on the ongoing RTOG 0618 study. Also, a photon energy of >15 MV is specifically discouraged due to the typically small field used and lack of electronic equilibrium from highly energetic laterally scattered electrons. Generally, peripheral tumor locations (defined as tumors that are >2 cm from the bronchotracheal tree) are favored for SBRT, due to the higher toxicity associated with the treatment of central lung lesions. 210. Which of the following findings is associated with a poor prognosis in children who

present with acute lymphoblastic leukemia? A. White blood cell count of <50,000/mm3 B. B-cell immunophenotype C. Chromosomal hyperdiploidy D. Presence of t(9;22) Correct answer is D. RATIONALE: Options A-C are favorable features for acute lymphoblastic leukemia (ALL). The presence of translocation t(9;22) is a high-risk feature associated with a worse treatment outcome. REFERENCE: Kun L. Leukemias in children. Pediatric Radiation Oncology. 4th edition. Chapter 2. Philadelphia: Lippincott Williams & Wilkins Publishers. pp 15-39.

Page 70: Radiation Oncology ACR 2010 in-Service Exam Rationales

211. Which of the following statements about anatomically undefined functional subunits

(FSUs) is true? A. The kidney and liver are examples of tissues with anatomically undefined FSUs. B. Tissues with anatomically undefined FSUs can be rescued by migration of surviving

clonogens from outside the radiation field. C. Tissues with anatomically undefined FSUs have a parallel arrangement. D. Tolerance doses for tissues with anatomically undefined FSUs are lower than for

tissues with anatomically defined FSUs. Correct answer is B. RATIONALE: It is important to remember that FSUs have two different characteristics: 1) whether they are contained within an obvious anatomical or histological boundary or not (i.e., anatomically defined or anatomically undefined) and 2) how they are arranged functionally within the tissue (i.e., "in parallel" or "in series"). Both are considerations when attempting to model the volume dependence of tissue tolerance to radiation. For example, the liver is a tissue with putative functional subunits that are anatomically defined (i.e., the lobule), but are organized in a parallel arrangement, whereas the spinal cord behaves as if its FSUs were arranged in series. Yet, what constitutes an FSU in the anatomical sense remains unclear (anatomically undefined). Tissues with anatomically undefined FSUs have an advantage in that they can be rescued from potential failure by the migration of surviving clonogens from outside the radiation field, so-called "tissue rescuing units." Anatomic or histological boundaries presumably make this impossible for tissues with anatomically defined FSUs. 212. Which of the following modes of intensity modulation is most reliable and capable of

delivering highly conformal dose distributions but is relatively labor intensive? A. Compensating filter B. Dynamic jaws C. Dynamic MLC D. Static MLC Correct answer is A. RATIONALE: Compensating filters are very reliable as they do not depend on electronic hardware and software interfaces on a linear accelerator, and they are also capable of delivering highly conformal dose distributions. The major shortcoming of a compensating filter is labor associated with production and daily treatments with multiple trips to the treatment room required by therapists to exchange the filters. 213. What is the most appropriate dose range of radiation therapy for a patient with a

completely resected Masaoka stage III thymoma? A. 36 to 40 Gy B. 41 to 45 Gy C. 50 to 55 Gy D. 60 to 66 Gy Correct answer is C. RATIONALE: While the optimal postoperative radiation therapy (RT) dose for patients with a resected thymoma is not fully established, many authors recommend a dose of at least 50 Gy. Since local control rates achieved with doses between 50 to 54 Gy are high (80-97%), it seems unnecessary to administer radiation doses of ≥60 Gy.

Page 71: Radiation Oncology ACR 2010 in-Service Exam Rationales

214. What is the 95% confidence interval for a specificity of 0.6 if the standard error is 0.08? A. 0.40 to 0.80 B. 0.44 to 0.76 C. 0.48 to 0.72 D. 0.52 to 0.68 Correct answer is B. RATIONALE: The correct answer is B since 95% confidence bounds are computed as the value of interest +2 times the standard error. 215. Which of the following conclusions about the addition of chemotherapy to radiation

therapy for anal cancer is supported by randomized trial data? A. Mitomycin C/5-FU improves overall survival versus radiation therapy alone. B. Mitomycin C/5-FU improves overall survival versus 5-FU alone. C. Mitomycin C/5-FU decreases the colostomy rate versus 5-FU alone. D. Cisplatin/5-FU improves colostomy-free survival versus mitomycin C/5-FU. Correct answer is C. RATIONALE: The RTOG 8704 study compared concurrent radiation therapy (RT) with 5-FU with or without mitomycin C. There was no significant difference in overall survival, but there was a significant decrease in colostomy rates in the mitomycin C arm (use of mitomycin C and 5-FU). Cisplatin-based chemotherapy was evaluated in RTOG 98-11, which randomized patients to induction cisplatin/5-FU, followed by concurrent RT with cisplatin/5-FU versus the standard concurrent 5-FU/ mitomycin C with RT. The cisplatin arm had a significantly worse colostomy rate. Both the EORTC of the UKCCR showed an improvement in colostomy rates with chemoradiation (5-FU/mitomycin C) versus RT alone, but no significant difference in overall survival. 216. Which of the following conditions would be most likely to reduce the chance for survival

in a patient with squamous cell carcinoma of the vagina? A. Age of less than 60 years B. A lesion in the middle third of the vagina C. Asymptomatic presentation at diagnosis D. A moderately differentiated tumor Correct answer is B. RATIONALE: The prognosis for a patient with squamous cell carcinoma of the vagina depends primarily on the stage of disease, but survival is reduced in patients who are older than 60 years of age, are symptomatic at the time of diagnosis, have lesions in the middle or distal third of the vagina, or have poorly differentiated tumors. REFERENCE: Gynecologic Oncology. 1991;40(1):12.

Page 72: Radiation Oncology ACR 2010 in-Service Exam Rationales

217. Which of the following is a small molecule tyrosine kinase inhibitor that targets the

EGFR? A. Erlotinib B. Temsirolimus C. Cetuximab D. Imatinib Correct answer is A. RATIONALE: Erlotinib (Tarceva) is a small molecule inhibitor that selectively targets the EGFR tyrosine kinase, which tends to be overexpressed and/or mutated in several malignancies, including non-small cell lung cancer. Cetuximab (Erbitux) is a monoclonal antibody that targets the extracellular domain of EGFR, not the intracellular tyrosine kinase, and is used in the treatment of head and neck and metastatic colorectal cancer. Temsirolimus (Torisel) is a small molecule inhibitor of mTOR, a kinase enzyme that transduces various growth and survival signals received by tumor cells, and is used in the treatment of renal cell carcinoma. Imatinib (Gleevec) targets the bcr-abl fusion protein, and is used in the treatment of chronic myelogenous leukemia and GI stromal tumors. 218. What is the advantage of tomotherapy over IMRT that uses less than five fields for

treatment plans? A. It has smaller beamlets. B. It results in better conformity. C. It results in a lower integral dose. D. Treatment planning is performed faster. Correct answer is B. RATIONALE: Multiple-field deliveries give better dose conformity. The differences between tomotherapy and IMRT becomes negligible when more than nine field angles are used. 219. Osteosarcoma most commonly develops in the: A. mandible. B. thoracic spine. C. iliac wing of the pelvis. D. diaphysis of the long bone. Correct answer is D. RATIONALE: Ninety percent of all osteosarcomas present in the diaphysis of the long bone (femur or humerus). The mandible is the most common site in the head and neck region. Osteosarcoma occurs in the pelvis in less than 10% of cases, but the pelvis is the most common non-extremity site. Osteosarcoma is seen in the spine in less than 2% of cases. REFERENCES: Larrier NA. Osteosarcoma. Halperin EC, Perez CA, Brady LW, eds. Perez and Brady’s: Principles and Practice of Radiation Oncology. 5th edition. Chapter 80. Philadelphia: Lippincott Williams & Wilkins Publishers. 2008;1801.

Page 73: Radiation Oncology ACR 2010 in-Service Exam Rationales

220. A 6-year-old patient undergoes complete resection of a fourth ventricular nonmetastatic

grade II ependymoma. What is the most appropriate next management step? A. EBRT to the primary site B. Multiagent systemic chemotherapy C. Craniospinal radiation therapy with a boost D. Observation with serial MRI scans Correct answer is A. RATIONALE: Complete surgical resection has been found to be the most important treatment for patients with ependymoma. Following maximal surgery, local radiation therapy is recommended. Routine chemotherapy and craniospinal radiation therapy are no longer recommended because of the lack of efficacy. 221. What was the local recurrence rate of a single brain metastasis after resection and

observation, according to the randomized trial that evaluated WBRT versus observation after resection of a single brain metastasis as reported by Patchel?

A. 16% B. 26% C. 36% D. 46% Correct answer is D. RATIONALE: A 46% rate of local recurrence at the original site was reported after surgery alone, which was 10% in the group receiving WBRT. Distant brain metastasis occurred in 37% versus 7% in the observation vs. WBRT groups, respectively. REFERENCE: Patchel. JAMA. 1988;280:1485-1489. 222. Circumcision is performed before radiation therapy in patients with penile carcinoma to: A. decrease radiation-associated morbidity. B. decrease tumor recurrences. C. improve chemotherapy delivery. D. improve lymphatic flow. Correct answer is A. RATIONALE: Circumcision, if indicated, must be performed before irradiation is commenced to minimize radiation-associated morbidity−including swelling, skin irritation, moist desquamation, and secondary infection. 223. A patient who receives radiation therapy to the primary site only for clinical stage N0 oral

cavity cancer is most likely to have a neck failure rate of: A. 70%. B. 50%. C. 30%. D. 10%. Correct answer is C. RATIONALE: Turner, et al, (1996) analyzed a series of 268 patients who received radiation therapy to the primary site only for oral cavity cancer with clinically negative lymph nodes (stage N0) and found an overall neck failure rate of 31%. The risk of subsequent neck failure varied between the sites of oral cavity and was as high as 40% for oral tongue cancers.

Page 74: Radiation Oncology ACR 2010 in-Service Exam Rationales

224. Which of the following factors is associated with a poor prognosis for patients with

chronic lymphocytic leukemia (CLL)? A. Age of <65 years B. Age of >65 years C. Trisomy 13 D. Trisomy 11 Correct answer is B. RATIONALE: It is important to be aware of the risk factors for chronic lymphocytic leukemia (CLL) to discuss the prognosis and outcomes of therapy with patients. 225. Which of the following statements about tirapazamine is true? A. It has shown promise for patients with certain tumors when combined with radiation

and/or cisplatin, but it has not met overall clinical expectations. B. In the presence of oxygen, it is rapidly ubiquitinated and targeted for proteasomal

degradation. C. A high incidence of peripheral neuropathy in patients receiving it significantly limits

its maximum-tolerated dose. D. The enzymes responsible for reducing it to its toxic metabolite under hypoxic

conditions are carbonic anhydrase and lysyl oxidase. Correct answer is A. RATIONALE: Tirapazamine is a bioreductive drug that is selectively toxic to hypoxic cells. It is only reduced to its toxic intermediate, an oxidizing free radical capable of both causing DNA damage and interfering with DNA repair, in the relative absence of oxygen. Although carbonic anhydrase and lysyl oxidase have been identified as oxygen-regulated proteins and endogenous hypoxia markers, they are not the enzymes responsible for the bio-reduction of tirapazamine. Rather, cytochrome P450, DT diaphorase, and nitric oxide synthase are the enzymes responsible for the bio-reduction of tirapazamine. To date, tirapazamine has been evaluated in Phase II and III clinical trials in combination with radiation therapy with or without cisplatin in patients with advanced head and neck and non-small cell lung cancer. Although the drug has shown promise in some situations, it has not met overall clinical expectations. Toxicities associated with tirapazamine include temporary hearing loss, muscle cramping, and nausea and vomiting. 226. What is a significant concern with the use of proton radiation therapy for pediatric

patients? A. Inability to control the exit dose deposition in small separations B. Longer times required to deliver treatments and daily anesthesia C. Increased whole-body dose due to neutron production in the treatment room D. Increased whole-body dose due to lateral proton scattering Correct answer is C. RATIONALE: Proton machines can be a significant source of neutrons, which can result in an elevated whole-body dose for pediatric patients. While there is no consensus on this issue, it is something that is routinely considered when discussing pediatric proton beam treatments.

Page 75: Radiation Oncology ACR 2010 in-Service Exam Rationales

227. Which of the following lymph node stages is most appropriate for a patient who has

breast cancer with palpable, mobile axillary adenopathy and both infraclavicular and internal mammary lymph nodes present on a staging CT scan?

A. N2A B. N3A C. N3B D. N3C Correct answer is C. RATIONALE: Stage N3B breast cancer = metastases in the ipsilateral internal mammary node(s) and axillary lymph node(s). Stage N2A breast cancer = metastases in the ipsilateral axillary lymph nodes fixed to one another (matted) or to other structures. Stage N3A breast cancer = metastases in the ipsilateral infraclavicular lymph node(s). Stage N3C breast cancer = metastases in the ipsilateral supraclavicular lymph node(s). Clinically apparent is defined as detected by imaging studies (excluding lymphoscintigraphy) or by clinical examination or grossly visible pathology. REFERENCE: AJCC Cancer Staging Manual, 6th edition. 2002;227. 228. Which of the following conditions most likely would require a treatment break for a

patient receiving chemoradiation for anal canal cancer? A. Moist desquamation B. Secondary infection of desquamated skin C. Two to three episodes of diarrhea daily D. Rectal irritation during defecation Correct answer is B. RATIONALE: Treatment breaks, while often necessary, should be limited as much as possible when patients are receiving chemoradiation for anal canal cancer. Moist desquamation in itself is not an indication for a break, unless there are signs of an infection present. Rectal irritation should be managed conservatively. Diarrhea should only result in a break if it cannot be controlled with medications. 229. What is the recommended dose to the vaginal mucosa when low-dose-rate intracavitary

brachytherapy is used to treat carcinoma in situ of the vagina? A. 30 to 40 Gy B. 45 to 55 Gy C. 60 to 70 Gy D. 80 to 90 Gy Correct answer is C. RATIONALE: Use intracavitary radiation delivering 60 Gy to 70 Gy to the vaginal mucosa to treat carcinoma in situ (CIS) of the vagina. Note that the entire vaginal mucosa needs to be treated. REFERENCES: International Journal of Radiation Oncology, Biology, Physics (Int J Radiat Oncol Biol Phys). 1988;15(6):1283-90. British Journal of Obstetrics and Gynaecology (Br J Obstet Gynaecol). 1988;95(10):976-9.

Page 76: Radiation Oncology ACR 2010 in-Service Exam Rationales

230. Which of the following treatment approaches is most appropriate for a 75-year-old man

who underwent a resection for thymic carcinoma with a focal residual tumor at the mediastinal margin, which was marked with surgical clips?

A. 54 Gy to the entire tumor bed only B. 60 Gy to the area marked with the clips only, with concurrent chemotherapy C. 60 Gy to the entire tumor bed, with concurrent chemotherapy D. 60 Gy to the entire tumor bed after chemotherapy Correct answer is C. RATIONALE: Thymic carcinomas are highly malignant tumors with a high risk of both local and distant failure despite aggressive therapy. Although no universally accepted standard treatment exists, best results seem to be reported with a maximum safe resection, followed by immediate postoperative radiation therapy (RT) given concurrently with cisplatin-based combination chemotherapy. Due to the tumor’s infiltrative nature, most studies use RT given to the entire tumor bed and mediastinum rather than to the area of suspected residual disease only. Radiation doses in the range of 60 to 70 Gy may be associated with a better outcome. 231. Which of the following hereditary syndromes is most likely to be found in a 1-year-old

patient who has an enlarged abnormally enhancing optic chiasm? A. Neurofibromatosis B. Tuberous sclerosis C. von Hippel-Lindau D. Li-Fraumeni Correct answer is A. RATIONALE: An enlarged, abnormally enhancing optic chiasm in a 1-year-old patient is suggestive of an optic nerve glioma, which is associated with neurofibromatosis, type 1. Tuberous sclerosis is associated with subependymal giant cell tumors; von Hippel Lindau disease is associated with hemangioblastomas; Li-Fraumeni is associated with brain tumors and other tumors. 232. Which of the following organs has a TD5/5 of approximately 30 Gy for whole-organ

irradiation using conventional fractionation? A. Liver B. Lung C. Brain D. Kidney Correct answer is A. RATIONALE: The TD5/5 (total dose associated with a 5% probability of a complication within 5 years after radiation therapy) for whole-liver irradiation is approximately 30 Gy for conventional fractionation with x-rays. The TD5/5 is 17.5 Gy for the whole lung, 23 Gy for the whole kidney, and 45 Gy for the whole brain. REFERENCE: Hall and Giaccia. Radiobiology for the Radiologist. 6th edition. 2006. Chapter 19 (and Table 19.2).

Page 77: Radiation Oncology ACR 2010 in-Service Exam Rationales

233. CT-based image-guided radiation therapy (IGRT) refers to the use of CT imaging: A. for treatment planning. B. before treatment delivery. C. during treatment delivery. D. after treatment delivery. Correct answer is B. RATIONALE: CT-based image-guided radiation therapy (IGRT) refers to the use of CT imaging prior to treatment delivery to minimize set-up errors. CT for treatment planning is not referred to as IGRT. CT during treatment delivery cannot be used because of high exposure. CT after treatment delivery is not considered IGRT. 234. Which of the following radiographic findings is characteristic of osteosarcoma? A. Sunburst pattern B. Onion skin effect C. Multifocal sclerotic lesions D. Lytic lesion in the diaphysis Correct answer is A. RATIONALE: A classic radiographic appearance of osteosarcoma is a sclerotic lesion most commonly seen in the metaphysis of bone. There can be a starburst pattern that is consistent with periosteal new bone formation. Onion skin effect is seen with Ewing’s sarcoma. Lytic lesions in the diaphysis are common findings in metastatic disease. REFERENCE: Hansen EK, Roach M., eds. Handbook of Evidence-based Radiation Oncology. Springer. 2007;417. 235. According to the RTOG 9508 (Andrews) trial, which of the following should be the

standard treatment for patients with one to three brain metastases? A. Surgery, followed by WBRT and an SRS boost B. WBRT, followed by an SRS boost C. WBRT alone D. SRS alone Correct answer is B. RATIONALE: Whole-brain radiation therapy (WBRT) and stereotactic radiosurgery (SRS) boost improved functional autonomy (KPS) for all patients, and these treatments improved survival for patients with a single unresectable brain metastasis. Therefore, WBRT and SRS boost should be standard treatment for patients with a single unresectable brain metastasis, and they should be considered for patients with two or three brain metastases. REFERENCE: Andrews, et al. Lancet. 2004;363(9422):1665-72. 236. What is the primary treatment for male urethral carcinoma? A. EBRT B. Brachytherapy C. Chemoradiation D. Surgery Correct answer is D. RATIONALE: The primary mode of therapy for male urethral carcinoma is surgical excision. However, because of the rarity of this disease, comparison of cure rates with surgery or radiation therapy is difficult.

Page 78: Radiation Oncology ACR 2010 in-Service Exam Rationales

237. Intergroup Study 0099 for nasopharyngeal cancer (NPC) concluded that: A. adjuvant 5-FU and cisplatin chemotherapy were well tolerated with high treatment

completion rates. B. the results of the radiation therapy alone arm were worse than expected compared

to other published results. C. chemotherapy improved 3-year event-free survival only. D. chemotherapy improved 3-year overall survival only. Correct answer is B. RATIONALE: When compared with radiation therapy (RT) alone, chemotherapy resulted in a significant benefit for both event-free survival (EFS) and overall survival (OS) at 3 years and 5 years. However, controversies remain, particularly regarding the magnitude of benefit because the results of the RT alone arm were grossly inferior to those achieved by most centers. Adjuvant chemotherapy was poorly tolerated. 238. Which of the following rearrangements is most often associated with follicular

lymphoma? A. Bcl-2 B. Bcl-16 C. VEGF D. MALT Correct answer is A. RATIONALE: Rearrangements of Bcl-2 are often associated with follicular lymphoma and testing should be considered for diagnosis. REFERENCE: 2009 Practice Guidelines by the National Comprehensive Cancer Network (NCCN). p 22. 239. Which of the following cellular features best differentiates necrosis from apoptosis? A. Apoptotic cells produce an inflammatory response. B. Apoptosis triggers a "chain reaction" of cell death in surrounding tissue. C. Necrotic cells explode, and apoptotic cells implode. D. Necrosis involves DNA laddering. Correct answer is C. RATIONALE: Distinguishing features of apoptosis include DNA laddering, elimination of cellular remnants by phagocytosis, and cellular implosion. Apoptosis is critically important during embryonic development and for the maintenance of normal tissue homeostasis, so it is very strictly controlled, in part to avoid "chain reactions." Features of necrosis include membrane swelling, culminating in rupture and the spilling of toxic cellular products into the microenvironment. This not only causes a local inflammatory response, but also runs the risk of causing a chain reaction of death in neighboring cells. 240. Which of the following groups of paraneoplastic syndromes is associated with thymoma? A. Myasthenia gravis, hypocalcemia, and a monoclonal gamma-globulin spike in serum

level B. Myasthenia gravis, hyponatremia, and a monoclonal gamma-globulin spike in serum

level C. Myasthenia gravis, erythrocytosis, and hypogammaglobulinemia D. Myasthenia gravis, red cell aplasia, and hypogammaglobulinemia Correct answer is D. RATIONALE: Paraneoplastic syndromes listed in option D have been associated with thymoma.

Page 79: Radiation Oncology ACR 2010 in-Service Exam Rationales

241. Compared to conventional treatment, IMRT may be associated with a greater risk to

patients because it requires: A. longer treatment times. B. computer control for delivery. C. a larger number of monitor units for delivery. D. a higher number of individual beams. Correct answer is C. RATIONALE: IMRT requires several times more monitor units than conventional radiation treatment delivery. An increased number of monitor units can result in higher doses delivered to patients if something goes wrong during treatment delivery. This type of damage is typically not possible with conventional radiation therapy. 242. Which of the following sites of metastases is most common in patients with ER-negative

breast cancer? A. Liver B. Lung C. Bone D. Soft tissue Correct answer is ABCD. RATIONALE: The liver is 17%. The lung is 28%. The bone is 33%. Soft tissue is 51%. REFERENCES: Clark GM, Sledge GW Jr., Osborne CK, et al. Survival from first recurrence: relative importance of prognostic factors in 1,015 breast cancer patients. Journal of Clinical Oncology. 1987;5(1):55-61. 243. Which of the following features is associated with anal cancer? A. The most common histology is adenocarcinoma. B. The perirectal lymph nodes are the first echelon lymph nodes. C. The inguinal lymph nodes are the first echelon lymph nodes. D. The majority are associated with EBV infection. Correct answer is B. RATIONALE: The perirectal lymph nodes are the first echelon lymph nodes. The most common histology is squamous cell carcinoma. Inguinal lymph nodes are always included in the radiation treatment. The distinction is that positive inguinal lymph nodes are given a higher radiation dose. 244. A vulvar cancer arising on the labia minora with involvement of the upper urethra is

classified as stage: A. T1. B. T2. C. T3. D. T4. Correct answer is D. RATIONALE: According to the 1997 AJCC staging manual, vulvar tumors involving the lower urethra and/or vagina and anus are classified as T3. Upper urethral involvement is classified as T4.

Page 80: Radiation Oncology ACR 2010 in-Service Exam Rationales

245. A 15-year-old girl with a large, unresectable, nonmetastatic iliac wing Ewing’s sarcoma

has an excellent response to 12 weeks of chemotherapy. Which of the following chemoradiation therapy approaches should be recommended?

A. 30 Gy to the prechemotherapy bone and soft tissue volume B. 45 Gy to the prechemotherapy bone and soft tissue volume C. 45 Gy to the prechemotherapy bone volume and postchemotherapy non-infiltrative

soft tissue volume D. 55.8 Gy to the prechemotherapy bone volume and postchemotherapy non-infiltrative

soft tissue volume Correct answer is D. RATIONALE: Gross unresectable disease is treated with a radiation dose of 55.8 Gy. A full dose of radiation therapy is administered to the infiltrative disease before chemotherapy, and it is delivered to the non-infiltrative “pushing” tumor bulk after chemotherapy. 246. A person is most likely to die after exposure to 10 Sv of total-body irradiation within: A. 3 days due to cerebral edema and/or cardiovascular shock. B. 1 week due to diarrhea, dehydration, and/or sepsis. C. 2 months due to infection and/or hemorrhage. D. 6 months due to cardiorespiratory failure. Correct answer is B. RATIONALE: A total-body dose in excess of approximately 7 Sv will elicit the gastrointestinal syndrome, which, for sufficiently high doses, would be fatal within about 1 week secondary to diarrhea, dehydration, or sepsis due to destruction of the bowel epithelium. A total-body dose in excess of approximately 2.5 Sv will illicit the hematopoietic syndrome, which, for sufficiently high doses, would be fatal within about 2 months secondary to infection or hemorrhage due to destruction of the bone marrow. A total body dose in excess of approximately 50 Sv will illicit the cerebrovascular syndrome, which is invariably fatal within about 3 days secondary to cerebral edema or cardiovascular shock. For total-body doses above about 8 Sv, pneumonitis of the lung – followed by progressive fibrosis – can occur, either of which can culminate in death within about 6 months due to cardiorespiratory failure. 247. The statement that “less than 20% of lung should receive more than 30 Gy” represents

a: A. hard dose constraint. B. soft dose constraint. C. volumetric constraint. D. DVH constraint. Correct answer is D. RATIONALE: Since both the maximum dose (= more than 30 Gy) and maximum volume (= less that 20% of lung) are prescribed, this statement represents a dose volume histogram (DVH) constraint. 248. What is the most common anatomical location for a melanoma involving the vulva? A. Clitoris B. Mons pubis C. Labia majora D. Vaginal vestibule Correct answer is C. RATIONALE: Most vulvar cancers (including melanomas) arise on the labia majora or minora.

Page 81: Radiation Oncology ACR 2010 in-Service Exam Rationales

249. Which of the following radiation treatment schedules is recommended for management

of desmoid tumors? A. 55 Gy for an unresectable tumor B. 60 Gy after complete gross resection with negative surgical margins C. 65 Gy after complete gross resection with microscopically positive surgical margins D. 65 Gy for a recurrent, unresectable tumor Correct answer is D. RATIONALE: Radiation therapy is indicated for inoperable tumors, after incomplete microscopic resection and resection for recurrent tumor with negative surgical margins. A total dose of 50-55 Gy is recommended postoperatively. For inoperable or recurrent desmoids, the recommended total dose is 60-65 Gy. After primary radiation therapy, the control rates do not differ from those after adjuvant radiation therapy. In most studies, tumor size had no prognostic influence on local control. According to a metaanalysis (698 cases in 13 studies) local control after complete resection with radiation therapy improved by 17% compared with surgery alone. With macroscopic (R2) and microscopic (R1) tumor rests, patients with adjuvant radiation therapy had better results than this. According to the patterns of care study in Germany, the long-term local control rate was 81.4% after primary radiation therapy and 79.6% after resection and postoperative radiation therapy. REFERENCES: Seegenschmiedt MH. Radiotherapy of nonmalignant diseases. Halperin EC, Perez CA, Brady LW, eds. Perez and Brady’s: Principles and Practice of Radiation Oncology. 5th edition. Chapter 89. Philadelphia: Williams & Wilkins Publishers. 2008;1949-1950. 250. Which of the following factors is associated with an increased risk for the development of

meningioma? A. Cranial irradiation B. Testosterone therapy C. Neurofibromatosis type 1 D. Ovarian cancer Correct answer is A. RATIONALE: An increased risk for the development of meningioma is associated with women, patients with neurofibromatosis (NF) type 2 or breast cancer, pregnant patients, and patients who have received cranial irradiation. 251. Before receiving treatment, a 69-year-old man with stage T2c adenocarcinoma of the

prostate has a PSA serum level of 18 ng/mL and Gleason score of 4+4. According to RTOG 9202 randomized trial data, the optimal amount of time for administration of androgen deprivation therapy in conjunction with radiation therapy is:

A. 4 months. B. 6 months. C. 2 years, 4 months. D. indefinitely. Correct answer is C. RATIONALE: RTOG 9202 was a randomized trial including 1554 men with clinical T2c-T4 disease. Patients were treated with 65 to 70 Gy and randomized to receive 4 months or 2 years, 4 months of androgen deprivation therapy (ADT). ADT consisted of flutamide and goserelin for 2 months before and 2 months during radiation therapy. Men randomized to receive adjuvant ADT were treated with goserelin for an additional 2 years. For the subset of men with a Gleason score of ≥8, long-term ADT was associated with an improvement in all endpoints measured, including overall survival. REFERENCE: Horwitz, et al. RTOG 9202. Journal of Clinical Oncology (JCO). 2008;26:2497-2504.

Page 82: Radiation Oncology ACR 2010 in-Service Exam Rationales

252. Which of the following statements about oropharyngeal squamous cell carcinoma is

true? A. The soft palate is the most common site. B. The incidence continues to increase in the United States. C. HPV infection is least likely to be associated with the condition. D. HPV-positive tumors are more likely to be well differentiated. Correct answer is B. RATIONALE: The tonsil is the most common site of oropharyngeal squamous cell carcinoma, followed by the base of tongue. The incidence of oropharyngeal squamous cell carcinoma in the tonsil and base of tongue continued to increase by 3.9% and 2.1% per year through the late 1990s in the United States. HPV-positive tumors are more likely to be undifferentiated, have basaloid features, and are more frequently associated with lymph node metastasis. 253. Which of the following histologic subtypes of non-Hodgkin lymphoma is CD5 antigen

positive? A. Mantle B. Diffuse large cell C. Marginal D. Follicular Correct answer is A. RATIONALE: Mantle non-Hodgkin lymphoma (NHL) is CD5 positive. Mantle cell lymphoma is thought to represent a variant of follicular lymphoma and was classified under the Working Formulation as a low-grade lymphoma. Most of the cases of a small-cleaved cell lymphoma in the Working Formulation terminology were mantle cell lymphomas (MLC). MLC is a small- to medium-sized B cell lymphoma with irregular nuclei that resemble the cleaved cells of germinal centers. The morphologic pattern can be diffuse, nodular, or mantle (mantles of malignant cells around normal-appearing germinal centers) or combinations of any. mantle cell tumor cells are typically CD5 positive. REFERENCES: Devita, et al. Cancer. 6th edition. 2001;2220. Halperin EC, Perez CA, Brady LW, eds. Non-Hogkin's lymphoma. Perez and Brady’s: Principles and Practice of Radiation Oncology. 5th edition. Chapter 76, p. 1742. 254. The radical competition model provides a simple explanation for: A. autophagy. B. cytokine signaling. C. the oxygen effect. D. the bystander effect. Correct answer is C. RATIONALE: The radical competition model provides a simple explanation for the oxygen effect, and the action of hypoxic cell radiosensitizers and normal tissue radioprotectors. When, for example, DNA is converted to a free radical by the direct or indirect action of ionizing radiation, it is viewed as having two possible fates. In the presence of oxygen, the DNA damage would be "fixed" by oxygen interacting with the DNA radical to form a peroxide. In the absence of oxygen, the DNA damage would be reversed or "restituted" by endogenous radical scavengers such as glutathione. This competition between fixation and restitution exists in a dynamic equilibrium that shifts with the relative concentrations of oxygen and endogenous radical scavenging compounds.

Page 83: Radiation Oncology ACR 2010 in-Service Exam Rationales

255. What is the range of displayed Hounsfield units (HU) if the window level is set at

−500 HU and the window width is set at 1500 HU? A. 0 and +1500 B. −250 and +1250 C. −500 and +1500 D. −1250 and +250 Correct answer is D. RATIONALE: If the window level is set at −500 HU and the window width is set at 1500 HU, the range of displayed Hounsfield units (HUs) is −500 ± 1500/2 = −500 ± 750 = −1250 HU and +250 HU. 256. According to Truong, et al (British Columbia Cancer Agency), which of the following two

factors were the best predictors of the actuarial 10-year risk of locoregional recurrence for patients undergoing mastectomy without radiation therapy for stage T1N0 or T2N0 breast cancer?

A. Tumor grade and surgical margin status B. Tumor grade and lymphovascular space invasion C. Tumor size and use of systemic chemotherapy D. Tumor size and surgical margin status Correct answer is B. RATIONALE: On logistic regression analysis, grade, and lymphovascular space invasion (LVI), T-stage and use of systemic therapy were statistically independent predictors of locoregional recurrence. On recursive partitioning analysis, the first split was at histologic grade 3 (12% vs. 6%). The concurrent presence of LVI increased the Kaplan-Meier 10-year locoregional recurrence risk to 21%, compared to 9% for grade 3 alone. REFERENCES: Truong P, et al. Patient subsets with T1-T2 node negative breast cancer at high local recurrence risk after mastectomy. International Journal of Radiation Oncology, Biology, Physics (Int J Radiat Oncol Biol Phys). 2005;62:175-182. 257. Which of the following tumor stages is most appropriate for a 2-cm pancreatic

adenocarcinoma that involves the duodenum but does not involve the celiac axis or superior mesenteric artery?

A. T1 B. T2 C. T3 D. T4 Correct answer is C. RATIONALE: Tumor that extends beyond the pancreas but without celiac axis or superior mesenteric artery (SMA) involvement is stage T3.

Page 84: Radiation Oncology ACR 2010 in-Service Exam Rationales

258. Which of the following statements about germ cell tumors is true? A. Mediastinal germ cell tumors in men are benign. B. Non-teratomatous germ cell tumors in the mediastinum are more common in

women. C. The most common site for extragonadal germ cell tumors is the mediastinum. D. Approximately 25% of all germ cell tumors are found in the mediastinum. Correct answer is C. RATIONALE: Mediastinal seminomas occur primarily in men. Non-teratomatous mediastinal germ cell tumors are virtually nonexistent in women. Myxoma is an intracardiac tumor. Mature teratoma is the most likely diagnosis. About 5% to 10% of all germ cell tumors are found in the mediastinum. They can be benign or malignant. Benign germ cell tumors are referred to as mature teratoma or desmoids. 259. Which of the following genetic aberrations is most commonly found in patients with

Ewing’s sarcoma? A. t(2;13) B. t(11;22) C. 1p deletion D. 11p13 deletion Correct answer is B. RATIONALE: The genetic aberration t(11;22)(q24;q12) is found in 85% of patients with the Ewing’s sarcoma family of tumors (ESFT) and is considered pathognomonic for the disease. Genetic abnormalities noted on 11p13 are associated with Wilms tumor; t(2;13) or t(1;13) is noted in alveolar rhabdomyosarcoma; 1p deletion or loss of heterozygosity (LOH) at 1p is noted in neuroblastoma. 260. Which of the following statements about amifostine is true? A. It contains a phosphate group essential for its activity as a radioprotector. B. It received FDA approval for the prevention of radiation-induced esophagitis. C. It has a dose-reduction factor of 5.0 for preventing death in mice due to the GI

syndrome. D. A common, significant side effect is hypotension. Correct answer is D. RATIONALE: Hypotension is a common, significant side effect of treatment with amifostine. It is administered to patients as a pro-drug that requires dephosphorylation by alkaline phosphatase to form its active metabolite (WR-1065). The "unmasked" thiol group (not the phosphate group) on this active metabolite is a free-radical scavenger responsible for the activity of amifostine as a radioprotector. The only FDA-approved indication for amifostine as a radioprotector is to prevent radiation-induced xerostomia, not esophagitis. The dose-reduction factor (DRF) for preventing death in mice from the gastrointestinal syndrome is approximately 1.8. 261. In contrast to CT scans, MRIs are used in CNS treatment planning because they: A. are less susceptible to motion artifacts. B. have better soft tissue discrimination. C. have less high-Z artifacts. D. have less noise. Correct answer is B. RATIONALE: Compared to CT scans, magnetic resonance images have better soft tissue contrast (discrimination).

Page 85: Radiation Oncology ACR 2010 in-Service Exam Rationales

262. Which of the following types of therapy is most likely to improve overall survival in a

patient with osteosarcoma? A. Surgery alone B. Adjuvant chemotherapy C. Neoadjuvant chemotherapy D. Neoadjuvant irradiation Correct answer is C. RATIONALE: The percent of necrosis after neoadjuvant chemotherapy in nonmetastatic cases is associated with improved overall survival. Patients having tumor with >90% necrosis have an overall survival near 70% compared with 50% in those with <90% necrosis. Recommended treatment is neoadjuvant chemotherapy ----> surgery ------> adjuvant chemotherapy for 4-6 months. Radiation therapy is indicated whenever surgical margins are positive. Surgical resection of lung metastases improves overall survival. REFERENCES: Larrier NA. Osteosarcoma. Halperin EC, Perez CA, Brady LW, eds. Principles and Practice of Radiation Oncology. 5th edition. Chapter 80. Philadelphia: Lippincott Williams & Wilkins Publishers. 2008;1801-1805. Hansen, EK and Roach M, eds. Handbook of Evidence-based Radiation Oncology. Springer. 2007; 416-421. 263. What is the minimum number of mitoses per 10 high-power fields required to diagnose

an anaplastic (malignant) meningioma, according to the WHO criteria? A. 5 B. 10 C. 15 D. 20 Correct answer is D. RATIONALE: Anaplastic (malignant) meningioma by the 2007 WHO criteria requires either the presence of at least 20 mitoses per 10 high-power fields (HPF) or cytology resembling that of carcinoma, melanoma, or a high-grade sarcoma. 264. According to a planned subset analysis of the Swedish randomized (Bill-Axelson) trial,

radical prostatectomy versus watchful waiting resulted in the greatest improvement of overall survival in men with prostate cancer and:

A. an age of <65 years. B. a Gleason score of ≥7. C. a PSA serum level of ≥20 ng/mL. D. negative surgical margins. Correct answer is A. RATIONALE: In this study, a planned subset analysis revealed that only men who are younger than 65 years of age appeared to derive a disease-specific survival benefit with prostatectomy. This randomized trial comparing radical prostatectomy to watchful waiting in men with early-stage disease in the pre-PSA screening era (clinical stages T1b, T1c, or T2) showed a statistically significant difference in overall survival (OS) at 10 years. After 10 years, the difference in OS was approximately 73% versus 68%; absolute difference 5.0%; relative risk of death 0.74 (95% confidence interval, 0.56–0.99). This benefit was restricted to men younger than 65 years at the time of surgery (P = .01 in a planned subset analysis of the effect of age on treatment efficacy). This highlights the importance of life expectancy in selecting men most likely to benefit from local therapy for localized prostate cancer. Older men are more likely to die from a competing risk of mortality, thereby minimizing the potential benefit of treating a prostate cancer that could otherwise take several years to progress. REFERENCE: Bill-Axelson, et al. New England Journal of Medicine (NEJM) 2005;357:1977-1984.

Page 86: Radiation Oncology ACR 2010 in-Service Exam Rationales

265. What is the contralateral neck failure rate in patients with stage T1-T2N0 tonsillar

carcinoma treated with ipsilateral neck irradiation alone? A. 15% B. 10% C. 5% D. <1% Correct answer is C and D. RATIONALE: O'Sullivan, et al, reported a 3-year local regional control (LRC) of 77% and a contralateral neck failure rate of only 3.5% in 228 patients who have tonsillar carcinoma treated with ipsilateral neck radiation therapy (RT) alone. Specifically, there was no contralateral neck failure in 118 patients with stage T1-T2N0 tumor in that series (O'Sullivan, et al, 2001). 266. A 43-year-old man presents with stage IA mantle cell lymphoma. Which of the following

radiation doses (in Gy) would be most appropriate? A. 46 B. 42 C. 36 D. 28 Correct answer is C. RATIONALE: According to the 2009 Practice Guidelines by the National Comprehensive Cancer Network (NCCN), a radiation dose of 30 to 36 Gy is most appropriate for early-stage mantle cell lymphoma. 267. According to the NCRP, what is the annual permissible dose limit for occupational

exposure to the lens of the eye? A. 10 mSv B. 50 mSv C. 100 mSv D. 150 mSv Correct answer is D. RATIONALE: NCRP Report 116, published in 1993, is the latest report on general radiation protection criteria and includes annual radiation exposure limits for occupational radiation workers and the general public. Cataract formation, a deterministic radiation effect, is the basis for exposure limits to the lens of the eye and is currently set at 150 mSv per year. 268. Which of the following diagnostic imaging tests can acquire images in arbitrary planes or

orientations? A. CT B. MRI C. PET D. SPECT Correct answer is B. RATIONALE: MRI does not depend on axial image acquisition and images can be acquired in arbitrary planes in the patient.

Page 87: Radiation Oncology ACR 2010 in-Service Exam Rationales

269. What is the most significant predictor of locoregional recurrence of breast cancer in a

patient after a mastectomy? A. Clinical tumor size B. Pathologic tumor size C. Surgical margin status D. Axillary lymph node status Correct answer is D. REFERENCE: Harris J., et al, eds. Patient selection for postmastectomy radiation therapy. Diseases of the Breast. 4th edition. Philadelphia: Williams & Wilkins Publishers. 2009;602-604. 270. Prophylaxis for tumor lysis syndrome should be considered before initial treatment of: A. chronic lymphocytic leukemia. B. diffuse large B-cell lymphoma. C. marginal zone B-cell lymphoma. D. hairy cell leukemia. Correct answer is A. RATIONALE: Prophylaxis for tumor lysis syndrome should be considered before treatment of chronic lymphocytic leukemia (CLL). REFERENCE: 2009 Practice Guidelines by the National Comprehensive Cancer Network (NCCN). p 9. 271. Which of the following regions of tumor involvement is classified as stage T3 ampulla of

Vater adenocarcinoma? A. Extension beyond the ampulla B. Invasion of the pancreas C. Invasion of the duodenum D. Invasion of adjacent organs Correct answer is B. RATIONALE: Ampullary adenocarcinoma that invades the pancreas is regarded as stage T3. 272. What is the most common histology of vulvar cancer? A. Melanoma B. Adenocarcinoma C. Clear cell carcinoma D. Squamous cell carcinoma Correct answer is D. RATIONALE: The most common tumor histology for vulvar cancers is squamous cell carcinoma (85%). The second most common histology is melanoma, comprising about 10% of all cases. Other histologies are significantly less common (<5%).

Page 88: Radiation Oncology ACR 2010 in-Service Exam Rationales

273. According to the Lung Cancer Study Group randomized trial, which of the following

results was demonstrated in patients who had a limited resection versus a lobectomy for stage T1N0M0 NSCLC?

A. 75% increase in local recurrence B. 75% increase in cancer-specific mortality C. 30% increase in distant metastasis D. 20% decrease in overall survival Correct answer is A. RATIONALE: There was a 75% increase in local recurrence rate (p=0.02) in patients with stage T1N0M0 non-small cell lung cancer (NSCLC), 30% increase in overall mortality (p=0.08), and 50% increase in cancer-specific mortality (p=0.09). REFERENCE: Lung Cancer Study Group. Annals of Thoracic Surgery (Ann Thorac Surg). 1995;60:615-22. 274. A 16-year-old patient presents with a painful, enlarging mass in the distal thigh. Plain

film radiography reveals a permeative diaphyseal femoral lesion with periosteal new bone formation lifting the bone cortex. What is the most likely diagnosis?

A. Aneurysmal bone cyst B. Synovial sarcoma C. Ewing’s sarcoma D. Osteosarcoma Correct answer is D. RATIONALE: Osteosarcoma is the most likely diagnosis for a patient who is 16 years of age and presents with this disease location. The plain film with the cortical abnormality (Codman’s triangle) is suggestive of a malignant process. Ewing’s sarcoma tends to have metaphyseal rather than diaphyseal involvement. 275. Which of the following statements about the interaction between hyperthermia and low-

LET ionizing radiation is true? A. Ionizing radiation inhibits the repair of hyperthermia-induced DNA damage. B. Ionizing radiation blocks hyperthermia-induced heat-shocked protein synthesis. C. Hyperthermia inhibits the repair of radiation-induced DNA damage. D. Hyperthermia eliminates radiation-induced cell cycle blocks and delays. Correct answer is C. RATIONALE: Hyperthermia inhibits the repair of radiation-induced DNA damage, and accordingly, both sublethal and potentially lethal damage recovery. The heat-induced radiosensitization takes the form of a steepening of the final slope of the radiation survival curve for concomitant exposure to high (about 45°C) temperatures and a removal of the shoulder of the radiation survival curve for exposure to lower (40°C to 43°C) temperatures. REFERENCE: Hall and Giaccia. Radiobiology for the Radiologist. 6th edition. 2006. Chapter 28. 276. Which of the following radionuclides has the LEAST penetrating radiation? A. Palladium-103 (HVLPb = 0.0008 cm) B. Iodine-125 (HVLPb =0.0025 cm) C. Cesium-131 (HVLPb = 0.002 cm) D. Ytterbium-169 (HVLPb = 0.18 cm) Correct answer is A. RATIONALE: The more penetrating or the higher the radiation energy, the higher the half-value layer (HVL) in millimeters of lead that is required for radiation shielding. Hence, Palladium-103, with the smallest HVL, has the lowest radiation energy.

Page 89: Radiation Oncology ACR 2010 in-Service Exam Rationales

277. Which of the following statements about radiation-induced soft tissue sarcomas is true? A. Patients who have an Rb (retinoblastoma) gene mutation are at increased risk for

development of the condition. B. Soft tissue sarcomas of the breast are most commonly associated with ipsilateral

irradiation from breast-conserving therapy. C. Radiation-induced soft tissue sarcomas can develop outside the area in which high-

dose therapy was administered. D. Angiosarcoma is the most common histological subtype. Correct answer is A. RATIONALE: The Rb (retinoblastoma) gene mutation plays a significant role in the development of radiation-induced soft tissue sarcomas. For patients with genetic retinoblastoma, the actuarial risk of developing a sarcoma in the radiation field within the next 18 years has been 6.6% (Trott). The most common radiation-induced malignancy after breast irradiation as part of conservation management is breast cancer in the contralateral breast. A cautious estimate of breast cancer risk suggests the lifetime risk for one breast exposed to 1 Gy is approximately 5% if irradiated at an age younger than 35 years, <3% at the age of 35-45 years, and much less or zero at an older age. The most common histology of soft tissue sarcoma associated with radiation is malignant fibrous histiocytoma. Radiation-induced soft tissue sarcoma tends to occur within the region of high-dose therapy, with very rare appearance in tissue receiving a dose of <10 Gy. REFERENCES: Halperin EC, Perez CA, Brady LW. The discipline of radiation oncology. Halperin EC, Perez CA, Brady LW, eds. Principles and Practice of Radiation Oncology. 5th edition. Chapter 1. Philadelphia: Lippincott Williams & Wilkins Publishers. 2008;54-55. Trott K-S, Kamprad, F. Estimation of cancer risks from radiotherapy of benign diseases. Strahlenther Oncol. 2006;8:431-436. 278. What is the most likely diagnosis for a 45-year-old man with impotence, infertility, and

gynecomastia? A. Germ cell tumor B. Adrenal cortical cancer C. Pheochromocytoma D. Prolactinoma Correct answer is D. RATIONALE: A prolactin-secreting tumor can cause these symptoms in men. Germ cell tumors are not usually associated with endocrinopathies. An adrenal cortical tumor would be associated with elevated cortisol levels. Pheochromocytomas secrete catecholamines and do not affect this pathway.

Page 90: Radiation Oncology ACR 2010 in-Service Exam Rationales

279. A woman with a localized renal mass and compromised renal function undergoes partial

nephrectomy. Final pathology reveals a 4-cm clear cell carcinoma with a focal positive surgical margin. Staging workup was negative. Which of the following management options would be most appropriate?

A. Observation B. Reexcision C. Radiation therapy D. Immunotherapy Correct answer is A. RATIONALE: Although the goal of resection is to achieve negative surgical margins, patients with positive surgical margins after partial nephrectomy do not necessarily have a poor prognosis. In one large retrospective study of 1390 partial nephrectomies treated at one of two high-volume institutions, patients with a positive surgical margin (5.5%) on final pathology did not appear to have an increased risk of local or distant failure when compared to patients with negative margins. Therefore, it is appropriate to manage select patients expectantly when weighed against the potential risks associated with re-resection or radical nephrectomy. Radiation therapy is uncommonly administered after nephrectomy. Retrospective data may indicate an improvement in local control for those at highest risk of local failure, but any benefit in local control is typically unlikely to improve survival. Other systemic therapy, including small molecule inhibitors or immunotherapy, have not improved disease control in prospective randomized studies and would be best administered in the context of a clinical trial. REFERENCE: Yossepowitch, et al. Journal of Urology (J Urol). 2008;179:2158-2163. 280. What is the expected 5-year local control rate for a patient with stage T1-T2N0 tonsillar

carcinoma treated with definitive radiation therapy? A. 80% B. 60% C. 40% D. 20% Correct answer is A. REFERENCE: Journal of Clinical Oncology. Jun 2000;18(11):2219-25. 281. Heat-shock proteins reduce both heat-induced cell killing and radiosensitization because

they: A. enhance heat-induced DNA damage. B. enhance radiation-induced membrane damage. C. eliminate radiation-induced gene expression. D. reduce heat-induced protein damage. Correct answer is D. RATIONALE: Heat-shock proteins (HSPs) are members of a family of related proteins expressed in cells that have been exposed to elevated temperatures or other stressors. They were first identified in Drosophila, but are present in most other species as well, including humans. HSPs function as intracellular molecular chaperones for other proteins, and as such, play important roles in protein-protein interactions and the prevention of potentially damaging protein aggregation. By helping to stabilize partially unfolded or aggregated proteins, HSPs reduce the damage caused by hyperthermia, and in turn, reduce both heat-induced cell killing and radiosensitization. REFERENCE: Hall and Giaccia. Radiobiology for the Radiologist. 6th edition. 2006. Chapter 28.

Page 91: Radiation Oncology ACR 2010 in-Service Exam Rationales

282. Which of the following radionuclides used for brachytherapy has the largest fractional

decay? A. Palladium-103 (T1/2 = 16.97 days) B. Iodine-125 (T1/2 = 59.4 days) C. Cesium-131 (T1/2 = 9.7 days) D. Ytterbium-169 (T1/2 = 32.02 days) Correct answer is C. RATIONALE: T1/2 = 0.693/λ; hence, the radionuclide with the shortest half life, Cesium-131 (T1/2 = 9.7 days), exhibits the largest decay per day, or the largest daily fractional decay. 283. Which of the following statements about the AJCC staging system for soft tissue

sarcomas is true? A. MRI findings are not used for T staging. B. Kaposi’s sarcoma is staged according to AJCC criteria. C. Regional lymph node involvement is classified as stage IV disease. D. The site of primary disease is included. Correct answer is C. RATIONALE: All clinical examination and radiological findings are used to define the soft tissue sarcoma stage. The site of disease is an important prognostic factor that is not incorporated into the current AJCC staging system. Kaposi’s sarcoma is not staged according to the rules of soft tissue sarcomas. Regional lymph node involvement is assigned stage IV status, regardless of grade, tumor size, and absence of diffuse metastatic disease. REFERENCE: Kotilingam D, et al. Staging soft tissue sarcoma: evolution and change. CA: A Cancer Journal for Clinicians. 2006;56:282-291. 284. What is the most appropriate management for an unresectable, progressive,

nonfunctioning pituitary macroadenoma with a mass effect on the optic chiasm? A. Single-fraction SRS B. Permanent interstitial brachytherapy with 125I seeds C. Hypofractionated stereotactic radiation therapy D. Conventionally fractionated EBRT Correct answer is D. RATIONALE: Conventional fractionated external-beam radiation therapy (EBRT) would be recommended with a dose of 45 to 50 Gy in 5 to 5.5 weeks. A larger daily fraction size is not recommended for tumors that are abutting the optic chiasm. Brachytherapy is not recommended because of the proximity to the optic chiasm.

Page 92: Radiation Oncology ACR 2010 in-Service Exam Rationales

285. According to the NSABP B24 (Fisher) trial, the use of tamoxifen reduced the 7-year

actuarial recurrence rate of ipsilateral breast cancer in patients who received breast-conserving surgery and radiation therapy for DCIS and invasion by what percent?

A. 2.5% B. 3.5% C. 5.5% D. 8.5% Correct answer is B. RATIONALE: The 7-year actuarial rate of ipsilateral breast cancer recurrence for both DCIS and invasion was 7.7% for patients receiving tamoxifen and 11.1% for patients who did not receive tamoxifen. The difference is 3.4%. Tamoxifen therapy resulted in a 44% reduction in the risk of subsequent invasive tumor recurrence but had no significant effect on ipsilateral noninvasive breast recurrence. Positive tumor margins were significantly associated with breast recurrence. Tamoxifen reduced ipsilateral breast failure by 22% with negative margins and 44% in cases with positive or unknown margins. REFERENCES: Wazer DE, Arthur DW. Breast: stage Tis. Halperin EC, Perez CA, Brady LW, eds. Perez and Brady’s: Principles and Practice of Radiation Oncology. 5th edition. Chapter 52. Philadelphia: Lippincott Williams & Wilkins Publishers. 2008;1171. Fisher B, Dignnam J, Wolmark N, et al. Prevention of invasive breast cancer in women with ductal carcinoma in situ: An update of the National Surgical Adjuvant Breast and Bowel Project Experience. Seminars in Oncology. 2001;28:400-418. 286. Which of the following carcinomas of the parotid gland has the highest risk for regional

spread? A. Ductal B. High-grade mucoepidermoid C. Adenoid cystic D. Adenocarcinoma Correct answer is ABCD. REFERENCE: Izandro Re´gis de Brito Santos, MD, Luiz P. Kowalski, MD. Multivariate analysis of risk factors for neck metastases in surgically treated parotid carcinomas. Archives of Otolaryngology-Head & Neck Surgery (Arch Otolaryngol Head Neck Surg). 2001;127:56-60. 287. A worse prognosis is associated with patients who have chronic lymphocytic leukemia

and deletion of which of the following short-arm chromosomes? A. 4 B. 8 C. 17 D. 22 Correct answer is C. RATIONALE: Deletion of 17p is an unfavorable feature in chronic lymphocytic leukemia (CLL). REFERENCE: 2009 Practice Guidelines by the National Comprehensive Cancer Network (NCCN). p 14.

Page 93: Radiation Oncology ACR 2010 in-Service Exam Rationales

288. A 31-year-old man presents with a painless left testicular mass. Which of the following

diagnostic tests would be most appropriate? A. Urinalysis and urine culture B. CT scan of the abdomen and pelvis C. Transscrotal needle biopsy of the testicle D. Testicular ultrasound Correct answer is D. RATIONALE: Ultrasound can confirm the presence of a solid testicular mass, and of the listed options, it is the most appropriate next step in evaluation. If a solid (rather than cystic) mass is confirmed on ultrasound, a testicular neoplasm must be considered. 289. Which of the following survival curve parameters is associated with the greatest capacity

for sublethal damage recovery? A. High α/β ratio B. Low α/β ratio C. Large D0 D. Small n Correct answer is B. RATIONALE: A cell or tissue characterized by a low α/β ratio will exhibit more sublethal damage recovery than one with a high α/β ratio. A lower α/β ratio implies that a larger fraction of the overall amount of cell killing is due to a two-hit process (i.e., a larger β component), and it is the interaction between these two different lesions in space and time that defines sublethal damage and its repair. In terms of the target theory model, the parameters most associated with the capacity for sublethal damage recovery are n, the extrapolation number, and Dq, the quasi-threshold dose. Large, not small, values for these parameters are characteristic of a survival curve with a broad shoulder, which in turn implies a large capacity for sublethal damage recovery. 290. The energy of an electron beam with a range of about 6 cm is approximately: A. 6 MeV. B. 9 MeV. C. 12 MeV. D. 16 MeV. Correct answer is C. RATIONALE: As a “rule–of-thumb,” the range of an electron can be approximated as R (cm) ≈ E/2 (MeV). Therefore, an electron beam with a range of about 6 cm has approximately 12 MeV of energy.

Page 94: Radiation Oncology ACR 2010 in-Service Exam Rationales

291. Which of the following statements about cutaneous angiosarcoma is true? A. The presenting lesion appears as a nodular mass. B. Histological grade is considered in the staging system for angiosarcomas. C. This tumor most commonly develops in women who have lymphedema after therapy

for breast cancer. D. The standard treatment consists of complete resection followed by irradiation. Correct answer is D. RATIONALE: Cutaneous angiosarcoma is a rare malignancy of vascular origin that usually arises in the scalp and face of elderly white males. Sixty percent of all angiosarcomas arise in the skin and superficial soft tissues, and 50% of cutaneous angiosarcomas are found in the head and neck area. The primary tumor typically presents as a "spreading bruise" that varies from blue to red in color. A nodular component develops as the tumor progresses. Typically, the primary tumor is more extensive than is apparent on physical examination. Many patients present with multifocal disease. Optimal treatment consists of wide local excision followed by radiation therapy to the primary site and to the regional lymph nodes. There is no staging system for angiosarcomas. REFERENCE: Mendenhall W, Mendenhall CM, Werning JW, et al. Cutaneous Angiosarcoma. American Journal of Clinical Oncology. 2006;29:524-528. 292. Which of the following test findings should be used to determine the gross tumor volume

delineation during radiation therapy planning for a patient with a fibrillary astrocytoma? A. CT scan abnormality only B. MRI FLAIR abnormality only C. MRI T1-weighted contrast-enhanced abnormality only D. Both MRI T1-weighted contrast-enhanced and FLAIR abnormalities Correct answer is D. RATIONALE: Fibrillary astrocytomas are WHO grade 2 and typically do not have significant enhancement. However, for delineation of the gross tumor volume (GTV), both the FLAIR and T1-weighted contrast-enhanced MRI images should be evaluated. These tumors are not able to be seen very well on CT scans with or without contrast. 293. A 65-year-old Caucasian man with a 30-pack-year smoking history presents with gross

hematuria. Cystoscopy reveals no abnormalities in the bladder, but urine cytology reveals cells consistent with urothelial carcinoma. What is the most appropriate next step in management?

A. IV pyelogram B. Total-body bone scan C. Nephroureterectomy D. Systematic biopsies of the bladder Correct answer is A. RATIONALE: Gross hematuria with positive cytology and a normal cystoscopy should raise the suspicion for upper tract disease (ureters or renal pelvis). The best reasonable next step is an IV pyelogram, which will help visualize the ureters and renal pelvis for filling defects. CT and/or ureteroscopy also can be done.

Page 95: Radiation Oncology ACR 2010 in-Service Exam Rationales

294. Which of the following maxillary sinus carcinomas is most likely to benefit from elective

lymph node irradiation? A. Stage T2 undifferentiated carcinoma involving the maxillary tuberosity B. Stage T2 squamous cell carcinoma originating above Ohngren's line C. Stage T3 adenoid cystic carcinoma with perineural invasion D. Stage T4 squamous cell carcinoma invading the orbit Correct answer is D. REFERENCES: Based on studies from Le. International Journal of Radiation Oncology, Biology, Physics (IJROBP). 2000;46(3):541-549. Jiang SB. Radiotherapy Oncology. 1991;21(3):193-200. 295. Which of the following tests would be best to monitor late effects in a patient after

treatment for Hodgkin lymphoma? A. PSA serum level and digital rectal exam B. Aggressive management of cardiovascular risk factors C. Liver function tests D. Colonoscopy Correct answer is B. RATIONALE: Patients treated for Hodgkin lymphoma may be at a higher risk for cardiovascular events at an earlier onset than patients with non-Hodgkin lymphoma. REFERENCE: 2009 Practice Guidelines by the National Comprehensive Cancer Network (NCCN). p 11. 296. As chromatin compaction increases, the amount of radiation-induced DNA damage is: A. unaffected. B. increased. C. decreased. D. variable. Correct answer is C. RATIONALE: The complexing of DNA with histone proteins and the resulting condensation of the chromatin serve important roles in terms of regulating DNA's "accessibility" for the purposes of transcription, replication, and repair. By the same token, the DNA associated with highly compacted chromatin is less accessible to the radiation-induced free radicals that might otherwise damage it. REFERENCES: Chiu S and Oleinick NL. Radiation Research (Radiat. Res). 1996;148:188-192. Warters RL, et al. Radiation Research (Radiat. Res). 1999;151:354-362. 297. Which of the following beam energies will produce the largest absorbed dose to the skull

bone for a patient with a brain lesion? A. 120 kVp B. 60Co C. 6 MV D. 20 MV Correct answer is A. RATIONALE: The “f” factor for the absorbed dose to bone is much greater for superficial treatments (120 kVp) than for the other megavoltage energies.

Page 96: Radiation Oncology ACR 2010 in-Service Exam Rationales

298. Which of the following tumor sites in patients with Ewing’s sarcoma is associated with

the worst prognosis? A. Rib B. Shoulder girdle C. Femur D. Pelvis Correct answer is D. RATIONALE: The duration of time from initial onset of symptoms to diagnosis for Ewing's sarcoma is one of the longest of all pediatric malignancies. Although overt metastases are present in approximately 25% of patients at diagnosis, subclinical metastases are suspected in 80-90% of patients. Patients with tumors in the extremity generally have a better prognosis than those in the axial skeleton, such as the pelvis. 299. VEGF-based therapies are most likely to be beneficial for patients with which of the

following types of tumor? A. Pilocytic astrocytoma B. Fibrillary astrocytoma C. Ganglioglioma D. Glioblastoma Correct answer is D. RATIONALE: VEGF is upregulated in patients with glioblastoma. Bevacizumab (Avastin) was FDA-approved in 2009 for recurrent glioblastoma (GBM). 300. Which of the following radiation fractionation schedules used to treat squamous cell

carcinoma of the skin involving the nasolabial fold is associated with the worst cosmesis?

A. 7.0 Gy x 5 B. 4.5 Gy x 10 C. 3.0 Gy x 17 D. 2.5 Gy x 20 Correct answer is A. RATIONALE: Fraction sizes of >5 Gy are associated with worse cosmesis. REFERENCES: Silva JJ. International Journal of Radiation Oncology, Biology, Physics (IJROBP). 2000;47(2):451-459 (PMH experience). Lim JT. Clinical Oncology (R Coll Radiol) 1992;4(4):236-239. Petrovich Z. American Journal of Surgery (Am J Surg). 1987;1549(4):447-450. 301. What percent of weight loss constitutes a B symptom? A. 5% B. 10% C. 15% D. 20% Correct answer is B. RATIONALE: The B symptom of weight loss is defined as 10%. REFERENCE: 2009 Practice Guidelines by the National Comprehensive Cancer Network (NCCN). p. 20.

Page 97: Radiation Oncology ACR 2010 in-Service Exam Rationales

302. What is one advantage of the clonogenic assay compared to an apoptosis or membrane

integrity-based assay? A. The clonogenic assay is the least expensive. B. The clonogenic assay takes less time to complete. C. The clonogenic assay takes into account multiple modes of cell death. D. All mammalian cell types are amenable to the clonogenic assay. Correct answer is C. RATIONALE: Apoptotic and dye exclusion assays only provide a "snapshot" of the fraction of cells dead or dying at the time of the assay. The clonogenic assay, however, is the gold standard for the determination of cellular radiosensitivity, even though it takes longer to complete and is more expensive and labor intensive overall. Further, it takes into account multiple modes of cell death (e.g., apoptosis, mitotic catastrophe, permanent growth delay, etc.) that occur over the entire colony formation period, typically a week or more. Not all cell types are amenable to the clonogenic survival assay however. 303. How many half-value layers (HVL) of shielding are required to reduce transmitted

radiation from 1.6% to 0.2%? A. 2 HVL B. 3 HVL C. 4 HVL D. 5 HVL Correct answer is B. RATIONALE: Each half-value layer (HVL) will decrease transmission of radiation by one half. Therefore, 3 HVL (1.6% x 0.5 x 0.5 x 0.5 = 0.2%) must be added to reduce the amount of transmitted radiation to 0.2%. 304. Which of the following findings is associated with the best prognosis for patients with

Ewing's sarcoma? A. Elevated alkaline phosphatase B. Elevated creatinine phosphokinase (CPK) C. Reduced polymorphonuclear leukocyte counts (PMN) D. Presence of certain chromosomal translocations Correct answer is D. RATIONALE: Systemic symptoms such as fever and elevated CPK are considered poor prognostic factors. Patients with metastatic tumors in the axial skeleton tend to have a poorer prognosis compared to those with metastatic disease in the extremities. At least 18 chromosomal translocations have been identified in the Ewing's sarcoma family (EWSF). For example, the presence of the EWSF exon 7 fusion to FLI1 exon 6 occurs in approximately 60% of patients and is a favorable prognosticator. 305. Which of the following chemotherapy drugs is mostly likely to decrease the contrast-

enhanced tumor volume in patients with glioblastoma? A. BCNU B. Irinotecan C. Bevacizumab D. Temozolomide Correct answer is C. RATIONALE: Bevacizumab is an anti-VEGF monoclonal antibody that can decrease tumor enhancement even if the T2-weighted or FLAIR MRI signal abnormality is increasing.

Page 98: Radiation Oncology ACR 2010 in-Service Exam Rationales

306. What is the most appropriate management for a 32-year-old man with a confirmed left

testicular solid mass? A. Inguinal orchiectomy B. Transscrotal orchiectomy C. Transscrotal needle biopsy D. Radiation therapy Correct answer is A. RATIONALE: A solid testicular mass should be treated initially with an inguinal orchiectomy. Subsequent treatments will be determined on the basis of the pathology. Biopsy may be considered if ultrasound shows a hypoechoic mass with calcifications, but inguinal orchiectomy is generally the preferred method of surgery for a solid testicular mass. Transscrotal approaches are not favored due the perceived risk of scrotal recurrences; however, studies have not confirmed this risk. AFP and β-hCG should also be obtained prior to surgery. 307. Which of the following sites of basal cell carcinoma has the best prognosis? A. Lower lip B. Columella C. Periauricular D. Paranasal Correct answer is A. RATIONALE: Basal cell carcinoma (BCC) alone in the H-zone has a higher local recurrence rate. The lower lip is not in the H zone. 308. Radiation-induced senescence is best characterized by increased activity of: A. caspase 3. B. caspase 9. C. lysosomes. D. SA β-galactosidase. Correct answer is D. RATIONALE: So-called "permanent growth delay" following irradiation was first noted in cultured cells decades ago, but only more recently has this phenomenon been identified as a form of senescence, not growth delay per se. Senescent cells display increases in cell size, increased activity of senescence-associated beta galactosidase (SA β-Gal), and altered patterns of gene expression. Increased lysosomal activity is a hallmark of autophagy, and increased activity of caspase occurs during apoptosis. REFERENCE: Verheij M. Cancer Metastasis. Rev. 2008;27:471-480.

Page 99: Radiation Oncology ACR 2010 in-Service Exam Rationales

309. The radius of the nucleus of an atom with atomic mass, A, is approximately

0.13•A1/3 • 10-12 cm. Which of the following radionuclides has the smallest nucleus?

A. Cs13755

B. Te12852

C. I12553

D. Xe12154

Correct answer is D. RATIONALE: This question requires knowledge of the atomic symbol ( bolElementSymA

Z ), and a distinction between the atomic number, Z, and the atomic mass number, A. Based on the equation provided, the atom with the smallest mass would have the smallest nucleus; hence, D is the answer. 310. Which of the following treatments is most appropriate for patients with chondrosarcoma? A. Adjuvant chemotherapy for low-grade tumors B. Wide local excision for intermediate- to high-grade tumors C. Radiation therapy for both low- and intermediate-grade tumors D. Radiation doses of 50 to 60 Gy for localized tumors Correct answer is B. RATIONALE: Grade and histology are key in the decision-making process for definitive management of chondrosarcomas. While intralesional curettage with adjuvant therapy can be performed in low-grade tumors, wide local excision is recommended for intermediate- to high-grade tumors. Low- and intermediate-grade tumors are thought to be radioresistant due to a slower proliferation rate, but radiation can be used for high-grade tumors, which are more radiosensitive. It is recommended that the radiation dose exceed 60 Gy, but this can be hard to achieve due to dose constraints of adjacent organs. Chemotherapy is not recommended after resection of low-grade chondrosarcomas. 311. Which of the following outcomes is most likely to be observed in patients who receive an

SRS boost in combination with fractionated radiation therapy and chemotherapy for glioblastoma?

A. Improved progression-free and overall survival B. Improved progression-free survival but not overall survival C. Improved local tumor control and quality of life D. No change in overall survival or quality of life Correct answer is D. RATIONALE: RTOG 9305 showed no benefit for any endpoint to upfront radiosurgery in the treatment of glioblastoma.

Page 100: Radiation Oncology ACR 2010 in-Service Exam Rationales

312. A 29-year-old man is found to have a 6.5-cm pure seminoma of the right testicle with

rete testis invasion after appropriate surgery is performed. His serum tumor markers are within normal range, and his staging workup is otherwise negative. Which of the following treatments is most appropriate for this patient?

A. Radiation dose of 20 to 30 Gy to the paraaortic lymph nodes alone B. Radiation dose of 20 to 30 Gy to the paraaortic lymph nodes and mediastinum C. One cycle of bleomycin, etoposide, and cisplatin D. Two cycles of single-agent cisplatin Correct answer is A. RATIONALE: Adjuvant paraaortic lymph node irradiation is the best choice among the options given. Bleomycin, etoposide, and cisplatin (BEP) chemotherapy is typically not used for nonmetastatic seminoma. Mediastinal radiation therapy can result in unacceptable late cardiac toxicity and does not provide a significant benefit. Single-agent carboplatin may be considered, but not cisplatin. While surveillance could be considered, the patient is not the best candidate based on his age, tumor size, and rete testis invasion, and surveillance is not listed as an option. REFERENCE: Warde, et al. Journal of Clinical Oncology (JCO). 2002;20:4448-52. 313. According to the RTOG 8305 trial, what is the expected complete response rate for a

4-cm melanoma treated with definitive radiation therapy? A. 15% B. 25% C. 35% D. 50% Correct answer is B. REFERENCE: Sause WT, et al. International Journal of Radiation Oncology, Biology, Physics (Int J of Radiat Oncol Biol Phys). 1991;20:429. 314. Which of the following histologic subtypes is most common in patients with classic

Hodgkin lymphoma? A. Marginal B. Lymphocyte-predominant C. Mixed cellularity D. Mixed chromosomal Correct answer is C. RATIONALE: Mixed cellularity is a variety of classic Hodgkin disease, whereas lymphocyte predominant is considered a distinct disease by the REAL and WHO classification. REFERENCE: DeVita, et al. Cancer. 2001;2344. 315. Which of the following proteins is involved in the activation of the G2/M cell cycle

checkpoint in response to DNA damage? A. pRb B. p16 C. p21 D. ATM Correct answer is D. RATIONALE: ATM plays a major role in the activation of the G2/M cell cycle checkpoint in response to DNA damage, including that caused by ionizing radiation exposure. The proteins p21, p16 and pRB are involved with the G1/S cell cycle checkpoint.

Page 101: Radiation Oncology ACR 2010 in-Service Exam Rationales

REFERENCE: Choudhury A, Cuddihy A, and Bristow RG. Seminars in Radiation Oncology (Semin. Radiat. Oncol). 2006;16:51-58.

Page 102: Radiation Oncology ACR 2010 in-Service Exam Rationales

316. Which of the following equations correctly describes the beta decay of 137Cs?

A. Cs13755 → Ba137

56 + β01−

+ νv00 + 0.66 MeV

B. Cs13755 → Ba137

54 + β01+ + νv0

0 + 0.66 MeV

C. Cs13755 → Ba137

55 + β00 + νv0

0 + 0.66 MeV

D. Cs13755 → Ba137

54 + β01−

+ νv00 + 0.66 MeV

Correct answer is A. RATIONALE: A beta particle is represented by the symbol β01− . During

beta decay, a neutron in the nucleus is converted into a proton, along with the emission of a beta particle and an anti-neutrino. As such, during beta decay, we would expect the atomic number of the decaying nucleus to increase by one, and the atomic mass to remain constant,

which is shown in option A. Option (B) is an example of positron [ β01+ ] decay, the symbol

shown in (C) is incorrect, and (D) results in an unbalanced equation [55 ≠ 54 + (-1)]. 317. Surgery is preferred in the treatment of patients with Ewing’s sarcoma that involves the: A. rib. B. femur. C. sacrum. D. humerus. Correct answer is A. RATIONALE: There have been no trials directly comparing surgery to radiation for local therapy, thereby causing the local treatment of choice to remain somewhat controversial. Generally, though, surgery is preferred for expendable bones, such as the rib or small bones of the hands and feet. Initially, concerns over dissemination of tumor throughout the bone marrow lead to larger radiation fields; but, with the use of multi-agent chemotherapy, this has fallen out of favor for "tailored fields" involving pre- and post-chemotherapy volumes. 318. Which of the following initial management steps would be most appropriate for a patient

with progressive or relapsed Hodgkin disease? A. Additional chemotherapy B. Consolidative radiation therapy C. Stem cell transplant D. Repeat biopsy Correct answer is D. RATIONALE: With progressive or relapsed Hodgkin disease, a repeat biopsy to confirm histology is necessary. REFERENCE: 2009 Practice Guidelines by the National Comprehensive Cancer Network (NCCN). p 12.

Page 103: Radiation Oncology ACR 2010 in-Service Exam Rationales

319. Which of the following types of ependymoma most commonly occurs in the cauda

equina? A. Subependymoma B. Tanycytic ependymoma C. Myxopapillary ependymoma D. Anaplastic ependymoma Correct answer is C. RATIONALE: Myxopapillary ependymomas are common in the cauda equina and have a better prognosis than other types. 320. Which of the following findings in a 65-year-old patient with transitional cell carcinoma of

the bladder is the best indication for bladder conservation with chemotherapy and radiation therapy?

A. Multifocal muscle invasion and carcinoma in-situ B. Stage T2a tumor after complete TURBT C. Stage T2b tumor and a bladder capacity of 60 minutes D. Stage T3a bladder tumor and hydronephrosis Correct answer is B. RATIONALE: Multifocal disease, carcinoma in-situ (CIS), and hydronephrosis have been shown to be relative negative prognostic factors for patients undergoing bladder conservation with chemoradiation. The condition in option C could be considered for chemoradiation, but the patient’s current bladder function is limited and is not likely to improve (and may worsen). The condition in option C may be better served by a cystectomy. 321. What is the best treatment approach for patients with anaplastic thyroid carcinoma? A. Maximal resection, followed by combined doxorubicin and hyperfractionated

radiation therapy B. Induction chemotherapy with doxorubicin, followed by EBRT C. Total thyroidectomy, followed by radioactive iodine and EBRT D. Thyroid ablation with radioactive iodine, followed by definitive resection Correct answer is A. RATIONALE: Based on a single institutional study, treatment of anaplastic giant and spindle cell carcinoma of the thyroid gland involves combined doxorubicin (Adriamycin) and hyperfractionated radiation therapy. REFERENCE: Kim JH, Leeper RD. A new approach. Cancer. September 15, 1983;52(6):954-7. 322. What is the most common histologic subtype of non-Hodgkin lymphoma? A. Mantle cell B. Diffuse large B-cell C. Extranodal marginal zone D. Follicular Correct answer is B. RATIONALE: Diffuse large B-cell lymphoma is most common and represents about one third of all non-Hodgkin lymphomas.

Page 104: Radiation Oncology ACR 2010 in-Service Exam Rationales

323. Which of the following drugs combined with radiation therapy increases the risk of

radiation pneumonitis? A. Tirapazamine B. Bevacizumab C. Docetaxel D. Cisplatin Correct answer is C. RATIONALE: The administration of taxane chemotherapy (e.g., docetaxel), especially when giving the drug concurrently or sequentially with radiation therapy, significantly increases the risk of radiation pneumonitis. REFERENCE: Taghian, et al. Journal of National Cancer Institute. 2001;93:1806-1811. Hanna, et al. Journal of Clinical Oncology. 2008;26:5755-5760. 324. Proton therapy requires beam energies in the range of: A. 5 to 20 MeV. B. 30 to 50 MeV. C. 75 to 100 MeV. D. 110 to 300 MeV. Correct answer is D. RATIONALE: Electron linear accelerators typically accelerate electrons within a range of 5 MeV and 20 MeV, whereas proton therapy requires reasonably higher energies (110 MeV to 300 MeV). 325. Which of the following factors is considered a risk factor for the development of

osteosarcoma? A. Prior trauma to the bone B. Exposure to alkylating agents C. Hyperparathyroidism D. Osteopetrosis Correct answer is B. RATIONALE: Osteosarcoma is the most common secondary malignancy in patients who received radiation therapy for a pediatric solid tumor. In addition, exposure to alkylating agents has also been implicated as a risk factor in a dose-dependent fashion. Paget's disease, a condition associated with accelerated bone turnover, is a risk factor, particularly in long-standing cases. Prior trauma to the bone has not been commonly associated with the development of osteosarcoma. 326. Which of the following conditions is LEAST likely to benefit from gross total resection? A. Posterior fossa ependymoma B. Third ventricular germinoma C. Suprasellar yolk sac tumor D. Medulloblastoma Correct answer is B. RATIONALE: Patients with CNS germinomas have a greater than 90% overall survival rate with radiation- or chemoradiation-based therapies. Gross total resection is not required.

Page 105: Radiation Oncology ACR 2010 in-Service Exam Rationales

327. Which of the following statements about interpreting study results reported with

confidence intervals is true? A. An observed result has to be within the parameters of the interval to reject the null

hypothesis. B. If the intervals of the studied parameters overlap, the results are more likely to be

significant. C. The smaller the interval, the more reliable are the results. D. The width of the interval is not impacted by sample size. Correct answer is C. RATIONALE: Confidence intervals are used to assess the reliability of an estimate. If an observed result falls outside of the interval, then the null hypothesis can be rejected. The size of the interval bars is directly impacted by the number of subjects in the sample size, i.e., the larger the number, the shorter the interval. The smaller the interval, the more dependable are the results. Non-overlapping confidence intervals on a graph indicate a true difference between study groups and, thus, more significant results. REFERENCE: http://en.wikipedia.org/wiki/Confidence_interval 328. Which of the following foramina at the skull base should be included in the postoperative

radiation field for a patient with adenoid cystic carcinoma of the submandibular gland? A. Ovale B. Jugular C. Rotundum D. Lacerum Correct answer is A. RATIONALE: Following the pathway of the V3 to the skull base leads to the foramina ovale. 329. Which of the following factors is NOT included in the International Prognostic Index for

diffuse large B-cell lymphoma? A. Hemoglobin level B. Number of extranodal sites C. Disease stage D. Age Correct answer is A. RATIONALE: The hemoglobin level is only considered in the follicular lymphoma international prognostic index (FLIPI), not International Prognostic Index (IPI). 330. What is the FIGO stage of an ovarian carcinoma with extension to the fallopian tube

without involvement of other sites in the pelvis or abdomen? A. IC B. IIA C. IIB D. IIC Correct answer is B. RATIONALE: According to the FIGO staging system, involvement of one or more ovaries with extension and/or metastases to the uterus and/or fallopian tubes without other sites of involvement is designated as stage IIA. If there is extension to the pelvis, it is stage IIB. Stage IC patients have disease limited to the ovaries with capsule rupture or ascites. Stage IIC patients have either stage IIA or IIB, but with tumor on the surface of one or both ovaries, or with ascities present containing malignant cells or with positive peritoneal cytology. REFERENCE: Halperin EC, Perez CA, Brady LW, eds. Ovary. Perez and Brady’s: Principles and Practice of Radiation Oncology. 5th edition. Chapter 68. p 1635.